You are on page 1of 193

EMLE Trial Exams October 2020 (1000 Questions) Version (1.

0) 9/12/2020

EMLE Trial Exams


October 2020
Exams 1000 Q
Version (1.0)
9/12/2020
If you have any comment or suggestions don’t hesitate to contact us through:

emle.question@gmail.com

1
EMLE Trial Exams October 2020 (1000 Questions) Version (1.0) 9/12/2020

Contents
Contents ............................................. 2
Exam 1 .................................................... 3
Exam 2 .................................................. 21
Exam 3 .................................................. 42
Exam 4 .................................................. 61
Exam 5 .................................................. 81
Exam 6 ................................................ 100
Exam 7 ................................................ 119
Exam 8 ................................................ 138
Exam 9 ................................................ 155
Exam 10 .............................................. 174

2
EMLE Trial Exams October 2020 (1000 Questions) Version (1.0) 9/12/2020

Exam 1
1) MCGN is characterized by all of the following except:
a) Heavy proteinuria
b) Hypoproteinemia
c) Hyperlipidemia
d) Generalized edema
e) Persistent hypertension
The Answer is : persistant hypertension
2) A 35-year-old woman is referred by her GP with painful nodular swelling of her breasts associated with her
menstrual cycle. She complains of cyclical pain and the feeling of multiple lumps in the breasts, some of which she
feels are growing larger. What is the most likely diagnosis
a) breast Abscess
b) breast cyst
c) fibroadenosis
d) fibroadenomas
e) fat necrosis
Answer is : fibroadenosis ( fibrocystic disease(
3) A 34-year-old man presents to his GP with a lump in his right testicle. This has been present for 6 months but has
started to increase in size causing him some discomfort. He has no lower urinary tract symptoms and is otherwise
well with no past medical history of note. On clinical examination of his external genitalia, there is a lump present
in his right hemiscrotum. It is 6 × 6 cm in size and is fluctuant but not tender. The testis is separate from the lump
and feels normal. You are able to get above the lump and it is not reducible nor does it have a cough impulse. On
attempting to shine a light behind the lump, it is brilliantly transilluminable. From the following list, select the
most likely diagnosis suggested by the history and clinical findings in this vignette .
a) Testicular tumor
b) hydrocele
c) inguinoscrotal hernia
d) epididymal cyst
e) hydatid cyst of Morgagni
Answer is : epididymal cyst
4) A 25-year old with recently diagnosed ulcerative colitis is started on mesalazine after a recent tapering of high
dose steroids. Two weeks later, he develops severe pain in his epigastrium which radiates through to his back.
What is the most likely diagnosis?
a) hepatitis
b) acute pancreatitis
c) primary sclerosing cholangitis
d) duodenal ulcer
e) acute coronary syndrome
Answer is: Acute pancreatitis
5) A 24-old year-old patient came to the outpatient with a 6-week history of fever, bloody diarrhea and weight loss.
On examination, he is clinically anemic and has aphthous ulcer of the mouth and mild tenderness of the
abdomen. Sigmoidoscopy shows transmural inflammation and non-caseating granulomas of the colon and
rectum. If the patient also complains of that lesion on his shins: What is the diagnosis of this skin lesion?
a) erythema nodosum
b) pyoderma gangrenosum
c) eczema
d) atopic dermatitis
e) dermatitis hepetitiformis
Answer is : erythema nodosum
3
EMLE Trial Exams October 2020 (1000 Questions) Version (1.0) 9/12/2020

6) Do you need to wash your hand after wearing gloves?


a) Yes
b) No
c) Only if the gloves are torn
d) Only if have enough time
e) Not every time
Answer is : yes
7) The INCORRECT statement for fungal vulvovaginitis:
a) A)Recurrence after symptomatic improvement is a rare possibility
b) B) there maybe Genitointestinal reservoir
c) C) Reinfection from the husband is a possibility
d) D) reinfection from the patient's Underwear is a possibility
e) E) long term course of oral antifungal may be needed
Answer is :Recurrence after symptomatic improvement is a rare
8) Which factor most associated with an increased risk of going on to have a stroke in a patient presented with TIA?
(Debatable)
a) old age
b) Bp 140/90 mmHg
c) unilateral weakness
d) >60 minutes duration
e) DM
Answer is : >60 minutes duration
9) A 49-year-old man presents with a 4-week history of recurrent head aches occurring at the same time every day,
shortly after going to sleep. It is usually associated with pain around the right eye and lacrimation. There is no
aura and each episode lasts less than an hour. The headache is relieved by movement. what is the cause of this
headache?
a) Giant cell arteritis
b) Subarachnoid hemorrhage
c) Tension headache
d) Cluster headache
e) Migraine
Answer is : cluster headache
10) A 37-year-old woman with right lower extremity edema is evaluated because of the sudden onset of Shortness of
breath and pleuritic chest pain. A diagnosis of pulmonary embolism is made. Which of The following signs, if
present on physical examination, would be the most specific indicator of Pulmonary arterial hypertension in this
patient
a) Increased jugular venous pressure
b) P2 louder than A2
c) Peripheral edema
d) Presence of an S3
e) Pulmonary crackles
Answer is: P2 louder than A2
11) The appropriate type of diuretic to help reaccumilation of ascites in patients with liver cirrhosis
a) Spironolactone
b) Frusemide
c) Thiazides
d) Osmotic diuretics
e) Carbonic anhydrase inhibitors
Answer is: spironelactone
12) Which of the following statements regarding preoperative investigations is false?
4
EMLE Trial Exams October 2020 (1000 Questions) Version (1.0) 9/12/2020

a) Chest X-ray is routinely requested in all patients over 60 years old .


b) A ventricular ejection fraction of less than 35 per cent indicates a high risk of cardiac complications .
c) A body mass index (BMI) <15 is associated with significant hospital mortality .
d) ECG is usually required in patients above 65 years old
e) HIV testing requires patient’s consent.
Answer is: Chest X-ray is routinely requested in all patients over 60 years old.
13) Complications of hemosiderosis include all of the following except:
a) Pubertal delay
b) Cardiomyopathy
c) Hypothyroidism
d) Deafness
e) Diabetes
Answer is: deafness
14) The INCORRECT statement regarding danazol in the treatment of endometriosis..
a) The aim is production of amenorrhea
b) The dose is fixed at 400 mg / day
c) The drug is successful in relieving the symptoms
d) It has androgenic side effects
Answer is: dose is fixed at 400mg/day
15) What of the following combination of diuretics is used in ascites secondary to liver cirrhosis?
a) Chlorthalidone and frusemide
b) Metolazone and frusemide
c) Bumetanide and eplerenone
d) Metolazone and spironolactone
e) Frusemide and metolazone
Answer is: Bumetanide and eplerenone
16) Clue cells are seen in..
a) Bacterial vaginosis
b) Candidiasis
c) Trichomoniasis
d) Chlamydiasis
e) None of the above
Answer is : d) Chlamydiasis
17) A 37-year-old patient complains of headache and abdominal pain a few minutes after red cell Transfusion.
Temperature 38.5°C, pulse 120/min, BP 100/60 mmHg.
a) Iron overload
b) Transfusion-related acute lung injury
c) Hypothermia
d) Non-haemolytic febrile reaction
e) Haemolytic transfusion reaction
Answer is: Haemolytic transfusion reaction
18) A 30-year-old primigravida presents at 34 weeks gestational age with blood pressure of 170/100 mmHg,
headache, epigastric pain, visual abnormalities and +3 proteinuria. Biophysical profile of the fetus is 8/8. Which
one of the following is the immediate response?
a) Start magnesium sulfate intravenously .
b) Perform an emergency C-section .
c) Give betaclomethasone to induce fetal lung maturity
d) Perform an amniocentesis to assess fetal lung maturity
e) Repeat the biophysical profile daily.
Answer is: start magnesium sulfate intravenously
5
EMLE Trial Exams October 2020 (1000 Questions) Version (1.0) 9/12/2020

19) A 43-year-old woman undergoes open cholecystectomy. Intraoperative cholangiogram revealed Multiple stones
in the CBD. Exploration of the CBD was performed to extract gallstones. The CBD was Drained with a #18 T-tube.
After 10 days, a T-tube cholangiogram reveals a retained CBD stone. This Should be treated by which of the
following?
a) Laparotomy and CBD exploration
b) Subcutaneous heparinization
c) Antibiotic therapy for 6 months and then reevaluation
d) Extraction of the stone through the pathway created by the T-tube (after 6 weeks )
e) Ultrasound crushing of the CBD stone
Answer is : extraction of the stone through the pathway created by T tube after 6 weeks
20) Female patient 42 years old with history of gall stones presented to ER by acute right abdominal Pain, fever, rigors
and mild jaundice. Lab results revealed WBCs 15000/dl, s. bilirubin 3.1, ALT 12 Mg/dl. Diagnosis is most likely :
a) Acute cholecystitis .
b) Acute cholangitis .
c) Acute hepatitis .
d) Calcular obstructive jaundice .
e) Biliary colic.
Answer is: acute cholangitis
21) The risk for development of ectopic pregnancy increases with
a) Clamydial salpingitis.
b) Combined oral contraceptive pills.
c) Familial mediranean fever.
d) LNG-intrauterine system.
e) Cu IUD 380-A.
Answer is a) Clamydial salpingitis.
22) Definite final diagnosis of cervical intraepithelial neoplasia Class III (CIN III) is made by:
a) Cervical smear
b) Colposcopy
c) Histopathology
d) Tumormarkers
e) Schiller iodine stain
Answer is : c) Histopathology
23) A 16-year-old primigravida reports that she is experiencing regular menstrual like-cramping every 2 minutes. She
is 32 weeks pregnant with mild vaginal bleeding. The MOST COMMON cause of this bleeding is:
a) A vaginal laceration
b) An endocervical polyp
c) Cervical dilation
d) Placenta previa
e) Placental abruption
Answer is : c) Cervical dilation
24) Diagnosis of ectopic pregnancy can be ruled out in almost all clinically significant cases if
a) Transvaginal sonography showed no abnormalities in the adnexal region.
b) Patient is using the IUCD for contraception.
c) Serum b-hCG was negative.
d) There is no pelvic fluid by transvaginal uterine ultrasound
e) Patient did not miss her period.
Answer is c) Serum b-hCG was negative.
25) Side effects of corticosteroids include:
a) Hypotension
b) Diabetes insipidus
6
EMLE Trial Exams October 2020 (1000 Questions) Version (1.0) 9/12/2020

c) Osteopetrosis
d) Stunted growth
e) Mental retardation
Answer is:- d) Stunted growth
26) Which one of the following is most likely to be found in a patient with Grave’s disease?
a) Raised ESR
b) Anti-TSH receptor stimulating antibodies
c) Anti-thyroid peroxidase antibodies
d) Decreased TSH
e) Co-existing type 2 DM
Answer is :- b) Anti-TSH receptor stimulating antibodies
27) A 33-year-old female is diagnosed with a personality disorder by her community psychiatrist. She has struggled to
hold down a job as an assistant store manager as she often finds her colleagues to be lacking in morals or values
and is reluctant to delegate work to them. She feels that her colleagues are lazy and do not perform their duties
to a sufficiently high standard, as a result, she is often overwhelmed with outstanding tasks that she cannot
complete and ends up staying late to get things right. What personality disorder is she most likely to have been
diagnosed with? (Debatable)
a) Schizoid
b) Narcissistic
c) Borderline
d) Obsessive – compulsive
e) Dependant
Answer is :- d) Obsessive – compulsive
28) A 24-year-old lady presents to the emergency department by acute abdomen. She is married 8 months ago. Her
history is suggestive for recurrent vaginal infections. On examination she is stable. The costovertebral angle is free
but has tender lower abdomen with positive rebound. Ultrasound revealed free fluid in the pelvis. The best next
step to reach diagnosis will be:
a) Urine analysis with culture and sensitivity
b) Laparoscopy
c) Cordocentesis
d) B-HCG
e) Urin pregnant test
Answer is :- d) B-HCG
29) An 18 month-old boy who was previously well, developed fever and irritability for 3 days. He was not coryzal.
Mother tells that she kept giving him antipyretics and the fever subsided spontaneously on the fourth day
followed by the appearance of this widespread maculopapular rash.
What is the most probable diagnosis?
a) Erythema infectiosum
b) Roseola infantum
c) Rubella virus
d) Measles virus
e) Infectious mononucleosis
Answer is :- b) Roseola infantum

30) A 25-year-old female presents with history of recurrent abortions with microthrombi detected on placental
pathology. The most relevant test for identifying cause is
a) Prothrombin time
b) Bleeding time
c) Dilute Russell’s viper venom time
7
EMLE Trial Exams October 2020 (1000 Questions) Version (1.0) 9/12/2020

d) Clot retraction time


e) None of the above
Answer is:- c) Dilute Russell’s viper venom time

31) The following about Candidal vaginal infection is INCORRECT:


a) The infection rate rises with pregnancy
b) Vaginal pH is usually alkaline
c) Vulval itching may occur
d) Vaginal isoconazole or miconazole are effective
e) Diagnosis is usually made without need for culture
Answer is :- b) Vaginal pH is usually alkaline
32) In neonates, clinical manifestations of congenital hypothyroidism include all the following except:
a) Macroglossia
b) Spasticity
c) Umbilical hernia
d) Failure to thrive
e) Constipation
Answer is :- b) Spasticity
33) Cerebral palsy is mainly by definition a:
a) Motor affection
b) Sensory affection
c) Autonomic affection
d) Behavioral problem
e) Mental problem
Answer is :- a) Motor affection
34) In hydrocephalus during the first year of life there is
a) Decrease size of the head
b) Early closure of fontanelles
c) closed sutures
d) no eye movement
e) Shunt is the best treatment
Answer is :- e) Shunt is the best treatment
35) A 25-year-old woman at 33 weeks, gestation is noted to have a placenta previa. Upon cesarean sec- tion, bluish
tissue densely adherent between the uterus and maternal bladder is noted. The most likely added placental
abnormality is: (Debatable)
a) Placenta accreta
b) Placenta increta
c) Placenta percreta
d) Placental polyp
e) Placental hemangioma
answer is:- b) Placenta increta
36) A 34‐year‐old man was involved in a motorcycle accident. On arrival, his initial systolic blood pressure (SBP) was
80/50 mm Hg but subsequently improved to 120/65 mm Hg after 2 L of normal saline bolus. His Glasgow coma
scale (GCS) was 13. He was taken to the computerized scan (CT). His head CT showed depressed skull fracture with
a subdural hematoma. While being transported back to the trauma bay, he again became hypotensive with SBP
85/50 mm Hg. What would be the most appropriate treatment for this patient?
a) Administer 2 L of normal saline and continue to observe
b) Administer 2 units of O‐ and continue to observe
c) Arterial embolization by interventional radiologist
d) Administer 2 units of O‐ and take the patient directly to surgery

8
EMLE Trial Exams October 2020 (1000 Questions) Version (1.0) 9/12/2020

e) Type and cross and wait for radiologist report


Answer is :- d) Administer 2 units of O‐ and take the patient directly to surgery

37) A full term male infant, born 10 hours ago. His mother is blood group O Rh+ve and her membranes ruptured 2
days before delivery. He is breastfeeding and on examination the baby is clinically well but markedly jaundiced.
What investigation should be performed first?
a) Bilirubin level
b) Blood culture
c) Blood group
d) Congenital infection screen
e) Direct antibody test
Answer is :- a) Bilirubin level
38) A 4 year-old has a positive urine dip for nitrites and leucocyte. You are going to start antibiotics and know that
for an upper UTI/pyelonephritis he requires a course of 7-10 days and for a lower UTI/cystitis he will require 3
days which of the following suggests pyelonephrities may be present?
a) dysuria
b) enuresis
c) frequency
d) mild abdomeninal pain
e) temp > 38 degrees
Answer is :- e) temp > 38 degrees
39) A 2 9 -year-old G 1 P 00 1 0 woman with regular menses ( every 2 8 dayslasting 5 days) is currently trying to
conceive. She has not used Chapter 9: Gynecology 1 93 contraception for 6 months . Her ovulation predictor kit
revealed an luteinizing hormone (
a) Cystic teratoma
b) Ectopic pregnancy
c) Follicular cyst
d) Hemorrhagic corpus luteum cyst
e) Serous cystadenoma
Answer is :- c) Follicular cyst
40) A 70-year-old woman presents to her GP complaining of pain over the medial aspect of her right knee. She also
states that she feels lethargic. She has per vaginal bleeding. Examination of the right knee is normal. The GP
examines the abdomen and palpates a mass in the lower abdomen. Which nerve is most likely to be injured?
a) Tibial nerve
b) Obturator nerve
c) Common peroneal nerve
d) Sciatic nerve
e) Femoral nerve
Answer is :- b) Obturator nerve
41) The following are risk factors for development of placental abruption EXCEPT
a) Short umbilical cord
b) Folic acid deficiency
c) Pre-eclampsia
d) History of threatened abortion
e) Previous placental abruption
Answer is: d) History of threatened abortion
42) A 3-day-old infant, has his routine newborn check. you notice blue and black macules on his back and buttocks.
He was born by normal vaginal delivery and has been feeding well since birth. From the following list pick the
diagnosis that fits with the clinical description and picture.
a) Bruising
9
EMLE Trial Exams October 2020 (1000 Questions) Version (1.0) 9/12/2020

b) Capillary hemangioma
c) Erythema toxicum
d) Mongolian blue spots
e) Port wine stain
Answer is : d) Mongolian blue spots
43) A 22-year-old primigravida at 34 weeks’ gestation presents with moderate vaginal bleeding and no uterine
contractions. The following sequence of examinations is the most appropriate:
a) Speculum examination, ultrasound examination, digital examination
b) Ultrasound examination, digital examination, speculum examination
c) Digital examination, ultrasound examination, speculum examination
d) Ultrasound examination, speculum examination, digital examination
e) Digital examination, speculum examination, ultrasound examination
Answer is: d) Ultrasound examination, speculum examination, digital examination
44) What of the following is an absolute contraindication to thrombolytic in stroke? (Debatable)
a) Active pancreatitis
b) Resolving neurological signs
c) Hemorrhagic diathesis
d) Major surgery in preceding 2 weeks
e) Pregnancy
Answer on exam web site is b) Resolving neurological signs
45) A 65-year-old patient presents with a non-tender solid mass in the right vulva that causes some dis- comfort when
walking and during coitus and that is consistent with a diagnosis of swelling of the Bartholin’s gland. What is the
MOST APPROPRIATE procedure?
a) Incision and drainage
b) Observation
c) Surgical excision
d) Administration of antibiotics
e) Marsupialization
Answer is: c) Surgical excision
46) IN the cardio ward round you noticed a fallout patient was crying severely then fall down suddenly what is you
second step to do if Squatting failed? (Debatable)
a) Morphine IV
b) Bronchodilator
c) Squatting position
d) Inderal
e) ICU admission
Answer on exam web site is d) Inderal
47) A patient is admitted to the emergency department following an assault. You note a penetrating wound on the
anterior chest wall. On examination, his blood pressure is 80/65 mmHg, pulse is thready and respiratory rate is 38
breaths/min. His jugular venous pulse is unrecognizable as the neck veins are grossly distended. Breath sounds
are equal bilaterally. During your evaluation the patient’s output becomes undetectable. The next course of
action should be
a) Thoracocentesis
b) Plain chest radiograph
c) Pericardiocentesis
d) Resuscitative thoracotomy
e) Echocardiogram
Answer is: c) Pericardiocentesis

10
EMLE Trial Exams October 2020 (1000 Questions) Version (1.0) 9/12/2020

48) A 12-year-old girl is brought to the GP by her mother having noticed a painless swelling on the left side of her
neck following a recent cold. On examination, there is a smooth, fluctuant, non-tender swelling anterior to the
sternocleidomastoid muscle. What is the most likely diagnosis?
a) Branchial cyst
b) Cervical rib
c) Cystic hygroma
d) Sternocleidomastoid tumour
e) Thyroglossal cyst
Answer is: a) Branchial cyst
49) Which of the following does not need patient consent?
a) History taking
b) Physical examination
c) Performing blood tests
d) Performing surgery
e) None of the above
Answer is: e) None of the above
50) A 62-year-old man presents with a 4-month history of progressive dysphagia, mainly to solids, and weight loss of
3 kg. Investigations show iron deficiency anemia and hypoalbuminemia. What is your diagnosis?
a) Bulbar palsy
b) Esophageal carcinoma
c) GERD
d) cardiac achalasia
e) esophageal candidiasis
Answer is: b) Esophageal carcinoma
51) A 45-year-old woman presents to the emergency department with abdominal pain and vomiting for 2 days. She
has passed flatus but has not passed stools since yesterday. On examination the abdomen is tender with marked
distension. There is a tender swelling above the umbilicus which is cresentric in shape. The swelling is irreducible
dark in colour and no cough impulse is present. The pulse rate is 108/min, BP 90/50 and temperature is 38.0°C.
Immediateline of management:
a) Preoperative detailed investigations to detect the cause.
b) Early fluid resuscitation.
c) Surgical interference.
d) Proper examination of content.
e) Resection of devitalized parts.
Answer is: b) Early fluid resuscitation.
52) A child 8 years had head trauma and has an attack of seizure after that what is your recommendation?
a) Urgent CT
b) Intubate
c) Urgent
d) MRI
e) Reassure
f) U/S
Answer is: a) Urgent CT
53) In attempting to minimize complications during cholecystectomy, the surgeon defines the triangle of Calot during
the operation. The boundaries of the triangle of Calot (modified) are the common hepatic duct medially, the cystic
duct inferiorly, and the liver superiorly. Which structure courses through this triangle?
a) Left hepatic artery
b) Right renal vein
c) Right hepatic artery
d) Cystic artery
e) Superior mesenteric vein
11
EMLE Trial Exams October 2020 (1000 Questions) Version (1.0) 9/12/2020

Answer is: d) Cystic artery


54) Which one of the following skin disorders is least associated with hypothyroidism? (Debatable)
a) Xanthoma
b) Pruritis
c) Pretibial myxedema
d) Eczyma
e) Dry, coarse hair
Answer on exam site is c) Pretibial myxedema
55) Normal daily requirements of vitamin D in normal infant is:
a) 100 IU
b) 400-800 IU
c) 1200 IU
d) 1800 IU
e) 2000 IU
Answer is: b) 400-800 IU
56) A 33-year-old female presents 6 weeks after the birth of her first child with a two-week history of polyarthralgia,
fever and a skin rash. First-line investigations show: ESR High.What is the most specific antibody of limited
scleroderma?
a) Anticentromere antibody
b) Anti-topoisomerase antibody
c) antiJO1 antibody
d) ANA
e) Anti CCP antibody
Answer is: a) Anticentromere antibody
57) A 25-year-old lady presents with a 2-week history of “growths” in the vulvar region. On examination, you find
multiple “cauliflower” verrucous lesions on the labia majora and minora.What is the most likely diagnosis in this
patient?
a) condyloma lata
b) condyloma acuminatum
c) herpes simplex
d) type 1 herpes simplex
e) type 2 genital acrochordon (skin tags)
Answer is: b) condyloma acuminatum
58) A homeless man is brought to A&E by paramedics after being discovered by a Commuter in an unconscious state.
There are no motor responses to pain and eye opening and verbal responses cannot be elicited. Calculate the
Glasgow Coma Score (GCS) of this patient
a) 0
b) 3
c) 4
d) 7
e) 9
Answer is: b) 3
59) Investigators of a study are examining the reliability of a patient questionnaire. Both inter-rater reliability and
test-retest reliability were examined by the investigators.If the study is determined to have poor test-retest
reliability, which of the following can be concluded?
a) This study suffers from low accuracy
b) This study suffers from low precision
c) This study suffers from selection bias
d) This study's poor reliability is due to random error
e) This study's poor reliability is due to systematic error

12
EMLE Trial Exams October 2020 (1000 Questions) Version (1.0) 9/12/2020

Answer is : b) This study suffers from low precision


60) A 38-year-old woman, awaiting a laparoscopic cholecystectomy for known symptomatic gallstones, presents with
severe epigastric pain radiating to her back and profuse vomiting. A CT scan shows bilateral pleural effusions and
ascites. What is the most likely diagnosis?
a) Budd-Chiari syndrome
b) Pancreatitis
c) Ovarian carcinoma
d) Alcoholic liver disease
e) Cardiac failure
Answer is: b) Pancreatitis
61) A 67-year-old man was referred to Vascular Surgery for monitoring of his popliteal artery aneurysm. What is the
most common aetiology of true peripheral artery aneurysms ?
a) Atherosclerosis
b) Medial degeneration of the arterial wall
c) Salmonella infection
d) Trauma
e) Femoropopliteal bypass procedure
Answer is : a) Atherosclerosis
62) A 36-year-old man presents to the emergency department with a severe left- sided headache with pain around
the left eye. He has had several similar episodes over the last 2 weeks, lasting 40-60 minutes each. The headaches
are associated with a runny nose. On examination, there is redness and tearing of his left eye. What is the most
appropriate acute management? (Debatable)
a) Acetazolamide
b) High flow O2
c) Ibuprofen and paracetamol
d) Sumatriptan
e) Urgent CT brain
Answer is : b) High flow O2
63. A 61-year-old man is admitted to the emergency department with sudden onset of a painful, cold, white right leg.
His radial pulse rate is 86bpm and its rhythm follows no discernible pattern throughout 30sec of palpation.
Abdominal examination is normal. No pulses are palpable in the right leg and ankle Doppler signals are absent. An
ECG confirms the arrhythmia but shows no signs of acute ischaemia. Which is the single most likely diagnosis ?
a) Abdominal aortic aneurysm
b) Aorto-iliac dissection
c) Atrial fibrillation
d) Deep vein thrombosis
e) Myocardial infarction
Answer is : c) Atrial fibrillation
64. A 14-year-old boy presents with a 6-hour history of a painful limp. He complains of severe pain in his left groin
and has vomited eight times. On examination, he has an absent cremasteric reflex on the left side.What is the
most likely diagnosis ?
a) Slipped upper femoral epiphysis
b) Testicular torsion
c) Transient synovitis
d) Septic arthritis
e) Perthes’ disease
Answer is : b) Testicular torsion

13
EMLE Trial Exams October 2020 (1000 Questions) Version (1.0) 9/12/2020

65. When a patient is admitted with acute stroke, which out of the following assessments should be completed first ?
a) Abdominal ultrasound
b) Carotid Doppler
c) Cognitive function
d) Echocardiography
e) Swallow safety
Answer is : a) Swallow safety
66. Glucose in oral rehydration solution is added to :
a) Improve the taste of the rehydration solution
b) Meet the child’s energy requirements
c) Increase osmolality of ORS to 400 mOsmol
d) Promote the absorption of sodium
e) Prevent malnutrition
Answer is : d) Promote the absorption of sodium
67. A 36-year-old woman with HIV/AIDS and B-cell lymphoma is hospitalized for Clostridium difficile—associated
diarrhea. Following treatment, the patient is discharged home with a prescription for a 14-day course of oral
vancomycin. She is unable to fill the prescription at her local pharmacy because of a problem with her insurance
coverage. While awaiting coverage approval, she receives no treatment. Her symptoms soon return, prompting an
emergency department visit where she is diagnosed with toxic megacolon. Which of the following should be
addressed in order to bring about changes that improve patient safety ?
a) Prescribing physician
b) Pharmacist
c) Insurance company
d) Patient
e) Discontinuity of care
Answer is : e) Discontinuity of care
68. Elevated total IgE is increased in :
a) G6pd deficiency
b) Hemophilia
c) parasetic infections
d) bacterial infections
e) Iron deficiency anemia
Answer is : c) parasetic infections
69. A 36-year-old woman comes to the physician to discuss contraceptive options. She is currently sexually active
with her husband, and they have not been using any contraception. She has no significant past medical history
and takes no medications. She has smoked one pack of cigarettes daily for 15 years. She is allergic to latex and
copper. A urine pregnancy test is negative. Which of the following contraceptive methods is absolutely
contraindicated in this patient ?
a) Diaphragm with spermicide
b) Progestin-only pill
c) Intrauterine device
d) Condoms
e) Combined oral contraceptive pill
Answer is : e) Combined oral contraceptive pill

14
EMLE Trial Exams October 2020 (1000 Questions) Version (1.0) 9/12/2020

70. A 31-year-old HIV patient presents with a 1-week history of dysphagia and odynophagia, mainly to solids. Barrium
swallow shows multiple esophageal ulcers. What is your diagnosis ?
a) Bulbar palsy
b) Esophageal carcinoma
c) GERD
d) cardiac achalasia
e) esophageal candidiasis
Answer is : e) esophageal candidiasis
71. A 2-year-old child presented with fever, impaired consciousness , a lumber puncture was done for suspected
meningitis; Which of the following is compatible with the diagnosis of bacterial meningitis ?
a) cells 10 proteins 20 glucose 200
b) cells 5 proteins 30 glucose 60
c) cells 3 proteins 40 glucose 80
d) cells 300 proteins 120 glucose 20
e) cells 20 proteins 10 glucose 90
Answer is: d) cells 300 proteins 120 glucose 20
72. A 63-year-old woman complains of a severe right-sided pleuritic chest pain 2 days after an elective total hip
replacement. She is hypoxic on air and has a sinus tachycardia on ECG. You suspect a pulmonary embolus, which
is confirmed on CT pulmonary angiogram. What is the most appropriate treatment ?
a) Low molecular weight heparin
b) Aspirin
c) Clopidogrel
d) Intravenous thrombolysis
e) Surgical thrombectomy
Answer is : a) Low molecular weight heparin
73. A 56-year-old man presents with painless jaundice, weight loss and anorexia. On examination there is a palpable
gallbladder. An ultrasound shows a dilated gallbladder with intrahepatic and extrahepatic duct dilatation. There is
a suspicious mixed echogenic lesion in the head of the pancreas. It is decided that he requires an endoscopic
retrograde cholangiopancreatography (ERCP) to obtain brushings from the pancreatic duct. His clotting studies
are prothrombin time 21 sec, activated partial prothrombin time 36 sec. You are informed that his prothrombin
time needs to be reduced to 13 sec for his ERCP. What is the most appropriate pharmacotherapy to correct the
prothrombin time ?
f) Activated factor VII
g) Platelet transfusion
h) Tranexamic acid
i) Warfarin
j) Vitamin K
Answer is : a) Vitamin K
74. You review a 24-year-old woman with a history of asthma in the Emergency Department. She has been admitted
with acute shortness of breath associated with tongue tingling and an urticarial rash after eating a meal
containing shellfish. Her symptoms settle with nebulised salbutamol and intravenous hydrocortisone what is the
diagnostic test of asthma severity ?
a) Arterial blood gases
b) Spirometry
c) Serum IgE
d) Serum tryptase

15
EMLE Trial Exams October 2020 (1000 Questions) Version (1.0) 9/12/2020

e) Eosinophil count
Answer is: b) Spirometry
75. What is the antibiotic regimen of choice for a patient on long‐standing proton pump inhibitor therapy with
suspected esophageal perforation from balloon dilation of a benign esophageal stricture ?
a) Cefazolin, piperacillin/tazobactam
b) Cefazolin, fluconazole
c) Vancomycin, piperacillin/tazobactam
d) Piperacillin/tazobactam, fluconazole
e) Vancomycin, piperacillin/tazobactam, fluconazole
Answer is: d) Piperacillin/tazobactam, fluconazole
76. What is the preferred treatment for an anaphylactic shock ?
a) Epinephrine
b) Placing the patient in sitting position and administer oxygen
c) Preventing the reaction from occurring through patient teaching
d) Placing a bag of ice on the area, administer antihistaminic and corticosteroids
e) Antibiotic therapy
Answer is : a) Epinephrine
77. A 19-year-old female is seen in the ER complaining of lower abdominal pain, which has been going on-off for 24
hours. She states that she had similar attacks in the last 6 months. She denies fevers. She has regular cycles
strictly every 28 days with considerable premenstrual tension syndrome symptoms. Her last menstrual period was
two weeks ago. Her vital signs are stable. On examination, she is tender in the lower abdominal area. She has no
rebound or guarding. Her WBC count is 8,500/cmm with normal differential. She most likely has :
a) Ectopic pregnancy.
b) Premenstrual tension pains.
c) Mittelschmerz ovulation pain.
d) Pelvic inflammatory disease.
e) Monilial infection
Answer is: c) Mittelschmerz ovulation pain.
78. The INCORRECT statement regarding intrauterine devices :
a) They neither affect ovulation nor steroidogenesis
b) Not suitable for lactating mothers
c) Perforation of the uterus is rare
d) When intrauterine pregnancy occurs on top of IUCD the risk of spontaneous abortion increases
e) If a pregnancy occurs in IUCD user, the risk of it being ectopic is higher than in women using other contraceptive
methods
Answer is : b) Not suitable for lactating mothers
79. Concerning rhesus hemolytic disease, select the most appropriate statement .
a) It can present as hydrops fetalis.
b) It cannot occur in firstborn children
c) It is an indication for elective caesarean section
d) It occurs as a consequence of the transplacental passage of IgM
e) The most common antibody type is anti-C
Answer is : a) It can present as hydrops fetalis.
80. Which is INCORRECT statement for nipple discharge :
a) It is common complaint
b) Galactorrhea is usually unilateral
c) It is usually physiologic
d) Duct ectasia is a benign cause for the discharge
e) Mammogram should be a part of the evaluation with bloody discharge
16
EMLE Trial Exams October 2020 (1000 Questions) Version (1.0) 9/12/2020

Answer is : b) Galactorrhea is usually unilateral


81. An 85-year-old male patient with a history of chronic constipation presents with acute severe colicky abdominal
pain and absolute constipation. Plain abdominal film shows a grossly dilated oval of large bowel arising from the
left lower quadrant. A diagnosis of sigmoid volvulus is made. The next step in management is:
a) Laparotomy
b) Sigmoidoscopy with flatus tube insertion
c) Sigmoid colectomy with colostomy
d) Barium swallow
e) Computed tomography
The answer is b) Sigmoidoscopy with flatus tube insertion
82. Initial metastatic deposits in the liver are best indicated by:
a) Increased bilirubin
b) Decreased serum albumin
c) Increased alkaline phosphatase
d) Increased gamma globulins
e) Increased AST (ALT)
The answer is c) Increased alkaline phosphatase
83. Which of the following is not a cause of gangrene?
a) Buerger’s disease
b) Infection
c) Intra-arterial drug injection
d) Frostbite
e) Deep vein thrombosis insufficiency.
The answer is e) Deep vein thrombosis insufficiency.
84. The investigation of choice in case of post-menopausal bleeding after Trans-vaginal Ultrasound is:
a) Pap smear
b) Fractional curettage
c) CA-125
d) Breast Ultrasound
e) MRI
The answer is b) Fractional curettage
85. 70-year-old man complains of reduced vision. Fundoscopy reveals the following: What is the most likely diagnosis
?
a) Primary open-angle glaucoma
b) Hypertensive retinopathy
c) Optic neuritis
d) Age-related macular degeneration
e) Diabetic retinopathy
The answer is d) Age-related macular degeneration

86. Which of the following about blood transfusion are false?


a) A hemoglobin level of 6 g/dL or less is now considered a typical indication.
b) Fresh frozen plasma (FFP) is considered as the first-line therapy in coagulopathic hemorrhage.
c) Cryoprecipitate is useful in low fibrinogen states and in factor VIII deficiency.
d) Platelets have a shelf life of 3 weeks.
e) Patients can pre-donate blood up to 3 weeks before surgery for autologous transfusion
The answer is d) Platelets have a shelf life of 3 weeks.

17
EMLE Trial Exams October 2020 (1000 Questions) Version (1.0) 9/12/2020

87. A 48-year-old builder presents following a fall from a second floor . He is assessted with a primary survey, and
found to has a respiratory rate of 32 breaths/min, a deviated trachea, distended neck veins and right-sided hyper
resonance on percussion. What is the most appropriate management option?
a) Chest X-ray
b) Chest drain
c) Laparotomy
d) CT head
e) CT neck
The answer is b) Chest drain
88. A 24-year-old woman gravida 2, para 1 at 24 weeks' gestation comes to the physician for a prenatal visit. She
feels well. Her earlier pregnancy was uncomplicated. This is her 4th prenatal visit. She had an ultrasound scan 2
weeks ago that showed a live intrauterine pregnancy consistent with a 22-week gestation with no anomalies. She
had a normal Pap smear 2 years ago. Vital signs are within normal limits. Pelvic examination shows a uterus
consistent in size with a 24-week gestation. Her blood group and type is B positive. Which of the following is the
most appropriate next step in management?
a) Serum PAPP-A and HCG level
b) Oral glucose challenge test
c) Cardiotocography
d) Rh antibody testing
e) Swab for GBS culture
The answer is Oral glucose challenge test
89. Child 2 years old his weight is 12 kg what is the maintenance fluid requirement for him if he is severely
dehydrated?
a) 550ml
b) 750 ml
c) 1100 ml
d) 1500 ml
e) 2000 ml
The answer is c) 1100 ml
90. A child 3 years old open his eye to pain only what Glasgow coma scale score you will give for that?
a) 2
b) 3
c) 4
d) 5
e) 6
answer is 2
91. The MOST SPECIFIC fetal malformation to diabetes mellitus is:
a) Ventricularseptaldefect
b) Coarctation of the aorta
c) Spinabifida
d) Sacral agenesis
e) None of the above
The answer is d) Sacral agenesis
92. You are called to review a 55-year-old man on the wards who started fitting around 5 minutes ago. He was
admitted three days ago following an acute coronary syndrome. His past medical history includes tonic-clonic
epilepsy which is generally well controlled on sodium valproate. On your arrival he is still fitting. Oxygen
18
EMLE Trial Exams October 2020 (1000 Questions) Version (1.0) 9/12/2020

saturations are 99% on 100% oxygen and his pulse rate is 96/min. Intravenous access is already in-situ. What is
the most appropriate next step?
a) Observation
b) IV lorazepam
c) IV phenytoin
d) Buccal midazolam
e) Call anaesthetic doctor
The answer is b) IV lorazepam
93. Hyperprolactinemia may be associated with all the following EXCEPT:
a) Chronic renal failure
b) Hypothyroidism
c) Pituitary adenoma
d) Methyldopatherapy
e) Adrenogenital syndrome
The answer is Adrenogenital syndrome
94. The risk factors for an ectopic pregnancy include the following EXCEPT:
a) History of pelvic inflammatory disease
b) Previous ectopic pregnancy
c) Use of combined oral contraceptives
d) Assisted reproductive techniques
e) Previous tubal surgery
The answer is c) Use of combined oral contraceptives
95. A 36-year-old man presents to the emergency department with a severe left-sided headache with pain around
the left eye. He has had several similar episodes over the last 2 weeks, lasting 40-60 minutes each. The headaches
are associated with a runny nose. On examination, there is redness and tearing of his left eye.What is the first line
long-term preventative management of cluster headaches?
a) Verapamil
b) Carbamazepine
c) Lamotrigine
d) Phenytoin
e) sumatriptan
the answer is a) Verapamil
96. As regards Chlamydial infection, the following statements are correct EXCEPT
a) The organisms are obligatory intracellular organisms
b) It may cause sterile pyuria
c) Chlamydia and gonorrhea are the main causes of PID
d) The husband should be treated
e) Azithromycin 2 g in single dose is an effective treatment
The answer is e) Azithromycin 2 g in single dose is an effective treatment
97. A newborn girl presents with puffy feet. The mother did not have any scans performed prior to delivery. On
examaniation, there are lymphoedematous feet, widely spaced nipples, a systolic murmur & absent femoral
pulses.
a) Angel man syndrome
b) DiGeorge syndrome
c) Edwards syndrome

19
EMLE Trial Exams October 2020 (1000 Questions) Version (1.0) 9/12/2020

d) Turner syndrome
e) Williams syndrome
The answer is d) Turner syndrome
98. A 2 3-year-old.woman who is 10 weeks pregnant with a 3-week history of hyperemesis gravidarum is brought to
the emergency room after she collapsed and started fiting. what is your diagnosis?
a) Tonic-clonic seizures
b) Hyponatremia
c) Absence seizures
d) Hypomagnesemia
e) Hypocalcemia
THE ANSWER IS d) Hypomagnesemia
99. Which of the following statements concerning hematuria in children is TRUE:
a) Urine in glomerular hematuria is usually red
b) Glomerular hematuria occurs at the beginning of urinary stream
c) UTI is the most common cause of hematuria
d) Lower urinary tract hematuria is usually accompanied by proteinuria
e) Renal biopsy should be done to all cases
The answer is c) UTI is the most common cause of hematuria
100. A 47-year-old alcoholic presents with a 24-hour history of severe epi¬ gastric pain radiating to the back, nausea
and vomiting. O/E: epigastric tenderness and rigidity. Investigations show macrocytic anaemia, leukocytosis,
hyperglycaemia and prolonged clotting .what is the cause of his abdominal pain?
a) Acute appendicitis
b) Acute pancreatitis
c) Ascending cholangitis
d) Acute MI
e) Ruptured abdominal aortic aneurysm
The answer is Acute pancreatitis

20
EMLE Trial Exams October 2020 (1000 Questions) Version (1.0) 9/12/2020

Exam 2
(1) A 24-year-old handball player presents to the fracture clinic with a comminuted, displaced fracture of the clavicle.
What is the most appropriate treatment option? (Debatable)
a) Hanging cast
b) External fixation
c) Manipulation and cast application
d) Plating
e) Dynamic hip screw
The answer is: Hanging cast
(2) Complications of evacuation and curettage of the uterus DO NOT include?
a) Uterine perforation.
b) Hemorrhage.
c) Infection.
d) Asherman’s syndrome
e) Amniotic band syndrome.
The answer is: Amniotic band syndrome
(3) A 70-year-old woman presents to an accident and emergency with sudden onset pain and swelling in the right
knee. Her past medical history includes hypertension and hypercholesterolaemia. She is currently taking aspirin,
ramipril and simvastatin. On examination, she is apyrexial and the right knee is swollen. There is reduced range of
movement in the knee due to swelling and pain. X-ray of the right knee shows chondrocalcinosis. What is the
most likely diagnosis?
a) Gout
b) Pseudogout
c) Septic arthritis
d) Reactive arthritis
e) Osteoarthritis
The answer is: pseudogout
(4) Which of the following is not a problem associated with surgery in obese patients?
a) Myocardial infarction.
b) Aspiration.
c) Deep venous thrombosis (DVT)/embolism
d) Pressure sore.
e) Pain control.
The answer is: pain control
5) A 25-year-old lady presents with a 2-week history of “growths” in the vulvar region. On examination, you find
multiple “cauliflower” verrucous lesions on the labia majora and minora.What is the most likely diagnosis in this
patient?
a) condyloma lata
b) condyloma acuminatum
c) herpes simplex type 1
d) herpes simplex type 2
21
EMLE Trial Exams October 2020 (1000 Questions) Version (1.0) 9/12/2020

e) genital acrochordon (skin tags)


The answer is: condyloma acuminatum
6) In bronchial asthma , symptoms are all of the following except:
a) Wheezing
b) Increased eosinophils in blood
c) Paroxysmal nocturnal attacks
d) Gross hemoptysis
e) Dry cough
The answer is: Gross hemoptysis
7) A 23-year-old man with very severe Crohn's disease has been put on complete bowel rest for an indefinite period
of time whilst he continues his in-patient recovery. He has been receiving total parenteral nutrition for the past
few weeks. Recently he has noticed his skin becoming dry and scaly with mouth ulcers and lip swelling. What is
the likely cause of these changes?
a) Iron deficiency
b) Zinc deficiency
c) Vitamin B12 deficiency
d) Folate deficiency
e) Selenium deficiency
The answer is: Zinc deficiency
8) You are called to review a 55-year-old man on the wards who started fitting around 5 minutes ago. He was
admitted three days ago following an acute coronary syndrome. His past medical history includes tonic-clonic
epilepsy which is generally well controlled on sodium valproate. On your arrival he is still fitting. Oxygen
saturations are 99% on 100% oxygen and his pulse rate is 96/min. Intravenous access is already in-situ. What is
the most appropriate next step?
a) Observation
b) IV lorazepam
c) IV phenytoin
d) Buccal midazolam
e) Call anaesthetic doctor
The answer is: IV lorazepam
9) Neonate presents with bilirubin 16, you have examined him seems he is good except for the jaundice After
excluding all the causes you have diagnosed him as breast milk jaundice, What is your next step?
a) Phototherapy
b) Exchange transfusion
c) Stop breastfeeding
d) Reassure
e) Phenobarbitone
The answer is: Reassure

22
EMLE Trial Exams October 2020 (1000 Questions) Version (1.0) 9/12/2020

10) Investigators of a study are examining the reliability of a patient questionnaire. Both inter-rater reliability and
test-retest reliability were examined by the investigators.If the study is determined to have poor test-retest
reliability, which of the following can be concluded ?
a) This study suffers from low accuracy
b) This study suffers from low precision
c) This study suffers from selection bias
d) This study's poor reliability is due to random error
e) This study's poor reliability is due to systematic error
The answer is: This study suffers from low precision
11) A 28‐year‐old man presents to the emergency department with a complaint of severe anal pain of 2 days duration.
Surgery is consulted with a referring diagnosis of “hemorrhoids.”The patient is very uncomfortable and cannot sit.
He relates no prior episodes or prior history of anorectal problems. His past medical history reveals a childhood
appendectomy and a history of asthma that is infrequent. On examination there is a small round bluish mass,
about 8 mm in diameter in a left‐lateral position at the anal verge. This is very tender on palpation. The remainder
of the examination is unremarkable. What is the best management of this condition?
a) Excision under local anesthesia
b) Observation, Sitz baths, oral analgesics
c) Hemorrhoid banding
d) Examination under general anesthesia, formal high ligation and hemorrhoid excision.
e) Incision and expression of hematoma.
The answer is: Excision under local anesthesia
12) A 72-year-old retired clerk who has smoked 40 cigarettes per day for 50 years has cramp- like pain in his left
calves, thighs, and buttocks which stops him walking after 60 meters on the flat ground. choose the single most
likely diagnosis
a) Aorto-iliac arterial disease
b) Cellulitis
c) Deep vein thrombosis
d) Lipodermatosclerosis
e) Superficial femoral artery occlusion
The answer is: Aorto-iliac arterial disease
13) A 90-year-old lady presents with a painful lump in the groin. It is tender and non-reducible, and appears to be
arising below and lateral to the pubic tubercle. What type of hernia is being described?
a) Direct inguinal hernia
b) Indirect inguinal hernia
c) Femoral hernia
d) Richter’s hernia
e) Spigelian hernia
The answer is: femoral hernia
14) A 42-year-old woman presents with an upper midline mass that has been present for over a year. She has a
history of a partial gastrectomy for a perforated ulcer. On examination, the mass is 5 cm in size, soft, non-tender
and reducible. Which of the following is the most likely diagnosis?
a) Direct inguinal hernia
b) Epigastric hernia
23
EMLE Trial Exams October 2020 (1000 Questions) Version (1.0) 9/12/2020

c) Hiatus hernia
d) Incisional hernia
e) Paraumbilical hernia
The answer is: incisional hernia
15) A 40-year-old man underwent laparoscopic cholecystectomy 2 years earlier. He remains asymptomatic until 1
week before admission, when he complains of RUQ pain and jaundice. He develops a fever and has several rigor
attacks on the day of admission. An ultrasound confirms the presence of gallstones in the distal CBD. The patient
is given antibiotics. Which of the following should be undertaken as the next step in therapy?
a) Should be discharged home under observation
b) Should be observed in the hospital
c) Undergo surgical exploration of the CBD
d) ERCP with sphincterotomy and stone removal
e) Anticoagulants
The answer is: ERCP with sphincterotomy and stone removal
16) What is the most appropriate antibiotic to use in cholera?
a) Erythromycin
b) Metronidazole
c) Doxycycline
d) Penicillin V
e) Trimethoprim
The answer is: Doxycycline
17) Which statement regarding esophageal perforation is most accurate?
a) In spite of recent advances in diagnosis and treatment of esophageal perforation, recent series demonstrate mortality
as high as 80%
b) Iatrogenic injury accounts for the majority of esophageal perforations
c) High mortality rates seen in patients with penetrating esophageal injuries are directly related to severe mediastinitis
d) In hemodynamically normal patients, free perforations of the thoracic esophagus can be successfully managed
with chest drainage and antibiotics alone
e) Esophageal perforation from food bolus impedance is frequently associated with untreated achalasia
The answer is: Iatrogenic injury accounts for the majority of esophageal perforations
18) A 28 -year-old G 1 PO presents to the clinic for her 42 -week appointment. She states that she is tired all the time,
and her feet swell when she stands for too long. When her cervix is checked by the physician, it is noted that it is
posterior, closed, thick, and very firm. As discussion begins regarding the possibility of induction, the patient asks
what can be done to improve her chances of having her baby vaginally. What do you tell her as her care provider?
a) Cesarean section is the best option for the baby at this point
b) The cervix must ripen on its own
c) Induction with artificial ripening agents is an option
d) Sexual intercourse always ripens the cervix
e) A transvaginal ultrasound (U /S) will tell us if your cervix is ready to deliver.
The answer is: Induction with artificial ripening agents is an option

24
EMLE Trial Exams October 2020 (1000 Questions) Version (1.0) 9/12/2020

19) As regards Chlamydial infection, the following statements are correct EXCEPT
a) The organisms are obligate intracellular organisms
b) It may cause sterile pyuria
c) Chlamydia and gonorrhea are the main causes of PID
d) The husband should be treated
e) Azithromycin 2 g in single dose is an effective treatment
The answer is: Azithromycin 2 g in single dose is an effective treatment
20) Male patient 60-years-old presented with progressive dysphagia more for food than for fluids for 4 months. On
examination the patient is cachectic pulse rate was 100 B/min., B.P. 100/65 mmHg. What is the best
investigation?
a) Upper endoscopy
b) PH monitoring study
c) Barium swallow
d) CT scan abdomen
e) Manometry
The answer is: upper endoscopy
21) A 16-year-old girl has been referred on to you by her gynaecologist. She has been
complaining of amenorrhea for 5 months, although no gynaecological abnormality has been found. She feels well and
is very active but her weight has decreased from 61 / 43kg in the last 6 months. her height is 168 cm. on
examination her BP is 90/60 mmHg, heart rate was 60 pbm . What is the most likely diagnosis?
a) Conn's syndrome
b) Crohn's disease
c) Anorexia nervosa
d) Hyperthyroidism
e) Diabetes mellitus
answer is ( C ) anorexia nervosa
22) A 35-year-old man is admitted after severing his arm on industrial machinery. His airway is patent and there is no
identifiable hindrance to breathing. His pulse is 110 beats/min, blood pressure is 130/105 mmHg, and respiratory
rate is 25 breaths/min. Which stage of shock is in this patient ?
a) Class I
b) Class II
c) Class III
d) Class IV
e) Impossible to say from given information
the answer is (B) class II
23) A 22-year-old pedestrian is brought to the Emergency Department following a road traffic collision with a car
travelling at 60 kmph. She was witnessed to have ‘bullseye’ the windscreen of the car. On presentation she has a
primary survey which revealed a Glasgow Coma Score of 9. What is the most appropriate management option ?
a) Trauma CT
b) Chest drain
c) Laparotomy
d) CT head
25
EMLE Trial Exams October 2020 (1000 Questions) Version (1.0) 9/12/2020

e) CT neck
the answer is (D) CT head
24 ) All are true statements concerning acute idiopathic thrombocytopenic purpura (ITP) except
a) It may follow chickenpox or EBV infection
b)Spleen is not enlarged
c) Intravenous immunoglobulin (IVIG) may be used in severe cases
d) Platelet count is less than 100,000 mm ³
e) Platelet transfusion is necessary in all cases
the answer is (E) platelets transfusion is necessary in all cases
25) A 5 months baby came with tachypnea, fever, mild intercostal and subcostal retractions, dry cough, and wheezing.
chest radiograph shows lung hyperinflation, peribronchial thickening “cuffing”, and increased interstitial
markings. How would you manage her?
a)Oxygen supplementation and fluid support
b) Glucocorticoids
c) Epinephrine
d) Theophylline
e) Penicillin
the answer is ( A ) oxygen supplementation and fluid support
26) Which one of the following is most likely to be a contraindication for tocolysis at 28 weeks gestation ?
a) Suspected abruption
b) Asymptomatic bacteriuria
c) Group B streptococcal bacteriuria
d) Uterine fibroid
e) Contracted pelvis
the answer is (A) suspected abruption
27) A 22-year-old primigravida at 34 weeks’ gestation presents with moderate vaginal bleeding and no uterine
contractions. The following sequence of examinations is the most appropriate
a) Speculum examination, ultrasound examination, digital examination
b) Ultrasound examination, digital examination, speculum examination
c) Digital examination, ultrasound examination, speculum examination
d) Ultrasound examination, speculum examination, digital examination
e) Digital examination, speculum examination, ultrasound examination
The answer is (D). Ultrasound, speculum, digital examination
28) Child 2 years old and his weight is 12 kg .What is the maintenance fluid requirement for him if he is severely
dehydrated?
a) 550 ml
b) 750ml
c) 1100 ml
d) 1500 ml
e) 2000 ml
The answer is (c) 1100

26
EMLE Trial Exams October 2020 (1000 Questions) Version (1.0) 9/12/2020

29) A 10-year-old girl presents with symptoms of an upper respiratory tract infection. She is incidentally noted to be
hypertensive on examination. Her systolic blood pressure (BP) is 116\75 mmHg, which is at the 95th centile for
her weight & height. What is the next best step in management?
a) Admission to hospital for anti-hypertensive treatment
b) None, as hypertension in childhood is defined as a BP>120 mmhg
c) BP measurement using the smallest cuff available
d) Renal biopsy
e) Ambulatory BP measurement
The answer is (E). Ambulatory Bp measurement.
30) A 26-year-old G 1 PO at 3 7 weeks presents with regular painful contractions for the past 6 hours of increasing
intensity. Upon examination, there is a bulging bag with cervical dilation of 7 em. No fetal part is detected. A
bedside ultrasound (U/S) reveals a footling breech presentation. Fetal heart tracing reveals baseline of 1 30s with
accelerations and no decelerations noted categorized as category 1 . The patient is taken for an immediate
Cesarean section. What would be the greatest risk for a vaginal delivery ?
a) Apnea associated due to prematurity
b) Cord prolapse
c) Entrapment of the aftercoming head
d) Meconium aspiration
e) Need for emergency Cesarean section
The answer is(B) cord prolapse.
31) Which of the following statements about ischaemia-reperfusion syndrome is correct?
a) This refers to the cellular injury because of the direct effects of tissue hypoxia
b) It is seen after the normal circulation is restored to the tissues following an episode of hypoperfusion
c) The increased sodium load can lead to myocardial depression
d) This is not influenced by the duration and extent of tissue hypoperfusion
e) It usually does not cause death
The answer is (B) it's seen after the normal circulation is restored to tissue following an episode of hypoperfusion
32) Child 11 years old. He has eaten from a streetcar and now he is having jaundice and abdominal pain which
infection do you suspect?
a) Hepatitis A
b)Shigella
c) Hepatitis B
d) Staphylococcus
e) Streptococcus
The answer is (A) Hepatitis A
33) What will you request to confirm streptococcal infection?
a) ASOT
b) ESR
c) CRP
d) CBC
e) X ray chest
The answer is(A) ASOT
27
EMLE Trial Exams October 2020 (1000 Questions) Version (1.0) 9/12/2020

34) Which of the following is not included in Standard Infection Control Precautions?
a) Management of blood and bodily fluid spillage
b) Patient’s personal hygiene
c) Safe handling of linen
d) Cleanliness of care equipment
e) Sharps safety
The answer is (B) Patient personal hygiene
35) You review a 24-year-old woman with a history of asthma in the Emergency Department. She has been admitted
with acute shortness of breath associated with tongue tingling and an urticarial rash after eating a meal
containing shellfish. Her symptoms settle with nebulized salbutamol and intravenous hydrocortisone . What is the
diagnostic test of asthma severity ?
a) Arterial blood gases
b) Spirometry
c) Serum IgE
d) Serum tryptase
e) Eosinophil count
The answer is (B) spirometry
36) A 29 -year-old G 1 P 00 1 0 woman with regular menses ( every 28 days lasting 5 days) is currently trying to
conceive. She has not used contraception for 6 months Her ovulation predictor kit revealed an luteinizing
hormone …. ) the question is incomplete from the source((Debatable)
a) Cystic teratoma
b) Ectopic pregnancy
c) Follicular cyst
d) Hemorrhagic corpus luteum cyst
e) Serous cystadenoma
The answer is (C) Follicular cyst
37) A 50-year-old hypertensive man who had a CVA before 6 months, presents with severe chest pain, vomiting and
sweating for the last four hours. ECG shows ST elevation in leads V1 - V4. Cardiac troponin (T) is 24 pg/mL (14.5
pg/mL) & troponin I is 1.2 ng/ml. (below 0.04 ng/ml). What is the best line of therapy?
a) Alteplase
b) Aspirin and clopidogrel
c) Heparin
d) Nitrate
e) Percutaneous Coronary Intervention
The answer is (E) percutaneous coronary intervention
38) What are the physiological daily sodium requirements of an average 70 Kg man ?
a) 20 mmol
b) 40 mmol
c) 80 mmol
d) 1 mmol
e) 140 mmol
The answer is. (C) 80mmol
28
EMLE Trial Exams October 2020 (1000 Questions) Version (1.0) 9/12/2020

39) Which one of the following is the main benefit of erythropoietin in patients with end stage renal disease?
a) Reduces proteinuria
b) Corrects anemia and improve exercise intolerance
c) Reduces blood pressure
d) Improves renal functions
e) Reduces long-term all cases mortality
The answer is (B) correct anemia and improve exercise intolerance
40) To measure obesity the best indicator is...
a) Measure the body weight
b) Measure the weight for height
c) Calculate body mass index (BMI
d) Determine growth velocity
e) Assists lipid profile
The answer is (C) Calculate the body mass index
41) For a child 15 kg with severe dehydration how many cc fluids will you give in the maintenance phase?
a) 1000 cc
b) 1250cc
c) 1500 cc
d) 2000 cc
e) none of the above
The answer is : 1250 cc
42) A 48-year-old smoker, who was diagnosed with COPD 10 years ago, is experiencing shortness of breath and a
productive cough with purulent sputum. These episodes have become more frequent within the last few years.
What is the most common causative agent of these exacerbations?
a) Staph Aureus
b) TB
c) Haemophilus influenzae
d) Pneumococcus
e) Moraxella catarrhalis
The answer is : Haemophilus influenzae
43) A 49-year-old man presents with a 4-week history of recurrent headaches occurring at the same time every day,
shortly after going to sleep. It is usually associated with pain around the right eye and lacrimation. There is no
aura and each episode lasts less than an hour. The headache is relieved by movement . What is the cause of this
headache?
a) Giant cell arteritis
b) Subarachnoid hemorrhage
c) Tension headache
d) Cluster headache
e) Migraine
The answer is : Cluster headache

29
EMLE Trial Exams October 2020 (1000 Questions) Version (1.0) 9/12/2020

44) A 43-year-old woman undergoes open cholecystectomy. Intraoperative cholangiogram revealed multiple stones
in the CBD. Exploration of the CBD was performed to extract gallstones. The CBD was drained with a #18 T-tube.
After 10 days, a T-tube cholangiogram reveals a retained CBD stone. This should be treated by which of the
following?
a) Laparotomy and CBD exploration
b) Subcutaneous heparinization
c) Antibiotic therapy for 6 months and then reevaluation
d) Extraction of the stone through the pathway created by the T-tube (after 6 weeks
e) Ultrasound crushing of the CBD stone
The answer is : Extraction of the stone through the pathway created by the T-tube (after 6 weeks)
45) A 61-year-old nulliparous woman comes to the physician for a follow-up examination. Her last Pap smear 3 years
ago showed atypical squamous cells of undetermined significance. HPV testing was negative at that time. On
questioning, she has had fatigue and an increase in abdominal girth despite a 5-kg (11.0-lb) weight loss over the
past 6 months. She has gastroesophageal reflux disease and Hashimoto's thyroiditis. Menarche was at the age of
10 years and her last menstrual period was 2 years ago. Current medications include omeprazole and
levothyroxine. Abdominal examination shows shifting dullness. There is tenderness to palpation of the right lower
quadrant but no guarding or rebound.Bimanual palpation shows a small uterus and a right adnexal mass. Further
evaluation of this patient is most likely to show which of the following findings?
a) Proliferation of endometrial glands
b) Elevated serum CA-125 level
c) Elevated serum beta-hCG level
d) Prolonged prothrombin time
e) Chocolate cyst of the right ovary
The answer is : Elevated serum CA-125 level
46) A 75-year-old ex-smoker presents to the vascular outpatient clinic with a claudication distance of several yards
and rest pain. A CT angiogram reveals multi-level atherosclerotic disease, including a flush short segment
occlusion of the proximal superficial femoral artery, an occluded proximal popliteal artery. There appears to be
adequate run-off below the knee. What is the most appropriate management?
a) Above knee amputation
b) Angioplasty
c) Femoral distal bypass
d) Embolectomy
e) Endovascular (abdominal) aneurysm repair (EVAR
The answer is : Femoral distal bypass
47) After an unexpected 3-hour delay in the operating room due to a problem in the electrical system, an operating
room team rushes to get started in order to complete the scheduled elective procedures. The team elects not to
perform the mandatory sponge count at the end of the first surgery in order to get the next case started sooner.
The patient returns 2 weeks later with abdominal pain and is found on x- ray to have a retained foreign object (a
sponge) in the abdominal cavity. Which bias caused this adverse event?
a) Violation bias
b) Anchoring bias
c) Confirmation bias
d) Premature closure
e) Availability bias
30
EMLE Trial Exams October 2020 (1000 Questions) Version (1.0) 9/12/2020

The answer is : Violation bias


48) Which one of the following is most likely to be found in a patient with Grave’s disease?
a) Raised ESR
b) Anti-TSH receptor stimulating antibodies
c) Anti-thyroid peroxidase antibodies
d) Decreased TSH
e) Co-existing type 2 DM
The answer is : Anti-TSH receptor stimulating antibodies
49) A patient refuses to undergo a necessary surgery. This is a demonstration of which of the following?
a) Autonomy
b) Confidentiality
c) Justice
d) Equity
e) beneficence
The answer is : Autonomy
50) A 4-year-old boy has a runny nose and fever for 3 days. He has now developed a cough and difficulty breathing.
On examination his temperature is 39° C. His breaths are rapid (RR is 55/min) but shallow with some mild
substernal recession. There is no wheeze on auscultation but some crackles at the right base. His oxygen
saturation is 91% in air. Select the most probable diagnosis for his respiratory symptoms?
a) Acute asthma
b) Bronchiolitis
c) Chronic asthma
d) Laryngotracheobronchitis (croup
e) Pneumonia
The answer is : pneumonia

51) A 47-year-old alcoholic presents with a 24-hour history of severe epigastric pain radiating to the back, nausea
and vomiting. O/E: epigastric tenderness and rigidity. Investigations show macrocytic anaemia, leukocytosis,
hyperglycaemia and prolonged clotting .What is the cause of his abdominal pain?
a) Acute appendicitis
b) Acute pancreatitis
c) Ascending cholangitis
d) Acute MI
e) Ruptured abdominal aortic aneurysm
The answer is : Acute pancreatitis
52) Which of the following IS NOT a risk factor for the development of pregnancy-induced hypertension (PIH)?
a) Multiple gestations
b) Chronic hypertension
c) Genetic predisposition
d) Iron deficiency anemia
e) Diabetes mellitus

31
EMLE Trial Exams October 2020 (1000 Questions) Version (1.0) 9/12/2020

The answer is : Iron deficiency anemia


53) Features characteristically associated with atopic dermatitis include all of the following except
a) Allergic rhinitis /asthma
b) Elevated serum IgE level
c) Peripheral blood eosinophilia
d) Lymphopenia
e) Itching
The answer is : Lymphopenia
54) In neonates, clinical manifestations of congenital hypothyroidism include all the following except
a) Macroglossia
b) Spasticity
c) Umbilical hernia
d) Failure to thrive
e) Constipation
The answer is : Spasticity
55) A 64-year-old woman presents with a 2-day history of increasing left-sided abdominal pain with fever. On
examination, she has localized peritonism in the left iliac fossa. Her blood tests reveal a raised white cell count
and C-reactive protein. Which of the following is the most likely diagnosis?
a) Constipation
b) Diverticular disease
c) Diverticulitis
d) Diverticulosis
e) Irritable bowel syndrome
The answer is : Diverticulitis
56) Which of the following is NOT a concerning finding for aortic injury on the CXR of a 27‐year old motorcyclist who
drove into a tree at highway speed?
a) Widened mediastinum
b) Depression of the left main bronchus
c) Apical (pleural cap)
d) Multiple posterior rib fractures at the costovertebral junction
e) Large left haemothorax
The answer is : Multiple posterior rib fractures at the costovertebral junction
57) The increased risk of postpartum hemorrhage during delivery of diabetic woman can be attributed to
a) Polyhydramnios
b) Macrosomia
c) Prolonged labor
d) Obstructed labor
e) All of the above
The answer is : All of the above

32
EMLE Trial Exams October 2020 (1000 Questions) Version (1.0) 9/12/2020

58) A 25-year- old woman, gravida 2, para 1, at 33 weeks of gestation presents with abdominal pain and minimal
vaginal bleeding. Her vital signs are Temp = 36.9, Pulse = 120, BP = 160/100. Fetal heart rate baseline is in the 160s
with minimal variability. Her uterus is tender with hypertonus. The best management for her condition is ...
)Debatable)
a) Urgent CS now
b) Start Induction of labour
c) Give steroids, magnesium sulphate, labetalol and deliver 48 hours later
d) Give steroids, labetalol and conserve till 34 weeks
e) Observe
The answer is : Urgent CS now
59) A 70-year-old woman presents with a 2-month history of anal pain and itching. More recently she had been
having some fresh bleeding and mucous discharge per rectum. On examination, there is an irregular tender
ulceration at the anal margin which appears to be extending into the anal canal. What is the most likely
diagnosis? )Debatable(
a) Anal carcinoma
b) Anal fissure
c) Anal warts
d) Fistula-in-ano
e) Primary syphilis
The answer is : Anal carcinoma
60) What is the level of BP at which thrombolytic can be given to stroke patients?
a) <200/100
b) <240/120
c) <185/110
d) <140/90
e) <120/90
The answer is : <185/110
61) A 43-year-old man has a work-up for hypertension. He was found to have blood + on a urine dipstick of a freshly
voided sample. Which one of the following may account for this finding?
a) Smoking
b) UTI
c) Obesity
d) Ramipril
e) Eating red meat the previous day
Answer is : b
62) A 41-year-old lawyer presents with a 3-month history of recurrent headaches which feel like a tight band around
the head.Which of the following is considered a dangerous sign of headache?
a) Morning headache and vomiting
b) Being unilateral
c) Being bilateral
d) Being in the morning
e) Associated with aura
33
EMLE Trial Exams October 2020 (1000 Questions) Version (1.0) 9/12/2020

Answer is a
63) Atopic dermatitis is characterized by:
a) never affect infants
b) Itchy
c) not related to food allergy
d) caused by bacterial infection
e) treated by antibiotics
Answer is b
64) A 54-year-old man presents with a 2-day history of upper abdominal and shoulder tip pain on the right side. He
also complains of difficulty breathing and episodes of fever and sweats. He underwent a laparotomy 2 weeks ago
following a perforated appendix. What is the most likely diagnosis?
a) Appendix abscess
b) Cholecystitis
c) Diverticular abscess
d) Polycystic liver disease
e) Subphrenic abscess
Answer is: subphrenic abscess
65) Which of the following drugs is contraindicated in asthmatic patients?
a) Valsartan
b) Propranolol
c) Enalapril
d) Atorvastatin
e) none of the above
Answer is Propranolol
66) Mother came complaining that her child Mona who is 13 years old has low ability to concentrate at school
recently she had her period, she looks slightly pale and usually feels tired what the first investigation you will do
is?
a) Chest X ray
b) ESR
c) CBC
d) Urine analysis
e) Nothing is needed
Answer is CBC.
67) The correct statement for hormonal contraception:
a) Ethinyl Estradiol (EE) is the estrogen present in the combined oral contraceptive pills (COCS).
b) Progestins are synthetic compounds that mimic the structure of natural progesterone
c) Failure rate is about 1/HWY with COCS and 5/HWY with POPs
d) Combined oral contraceptive pills increases the risk of anemia
e) History of hepatic adenoma is not a contraindication of COC pills
Answer is: a) Ethinyl Estradiol (EE) is the estrogen present in the combined oral contraceptive pills (COCS).

34
EMLE Trial Exams October 2020 (1000 Questions) Version (1.0) 9/12/2020

68) A full-term baby boy, born 16 hours ago. His mother is blood group O rhesus positive. The baby is breastfeeding
well but he looks jaundiced. On examination the baby is clinically well. His bilirubin was 9 mg/dl at 10 hours, and
six hours later his bilirubin is 15.5 mg/dl. His blood group was identified as group A rhesus positive. Choose the
most likely cause of the baby’s jaundice:
a) ABO incompatibility
b) Physiologic jaundice
c) Congenital hypothyroidism
d) Congenital infection
e) Rhesus hemolytic disease
Answer is ABO incompatibility
69) 22 years old pt. presented to the ER after RTA, O/E pt. was hypotensive, absent breath sounds, engorged neck
veins in the left chest, what is the next step?
a) Pericardiocentesis
b) Chest tube
c) Chest x ray
d) Thoracotomy
e) Needle decompression
Answer is needle decompression
70) The INCORRECT statement regarding endometriosis : )debatable(
a) There is no evidence of spread with lymphatics.
b) Women with primary relatives with endometriosis have a greater risk of having the disease.
c) Endometriosis is most often at the dependent portions of the pelvis
d) There is evidence to support immunological etiological factors.
e) It is an estrogen-dependent disease.
Answer is d d) There is evidence to support immunological etiological factors.
71) A 36-year-old man presents to the emergency department with a severe left-sided headache with pain around the
left eye. He has had several similar episodes over the last 2 weeks, lasting 40-60 minutes each. The headaches are
associated with a runny nose. On examination, there is redness and tearing of his left eye. What is the most
appropriate acute management?
a) Acetazolamide
b) High flow O2
c) Ibuprofen and paracetamol
d) Sumatriptan
e) Urgent CT brain
Answer: b) High flow O2
72) You review a 24-year-old woman with a history of asthma in the Emergency Department. She has been admitted
with acute shortness of breath associated with tongue tingling and an urticarial rash after eating a meal
containing shellfish. Her symptoms settle with nebulised salbutamol and intravenous hydrocortisone. What is the
diagnostic test of asthma severity?
a) Arterial blood gases
b) Spirometry
c) Serum IgE

35
EMLE Trial Exams October 2020 (1000 Questions) Version (1.0) 9/12/2020

d) Serum tryptase
e) Eosinophil count
Answer is spirometry
73) A 69-year-old man presents with a swelling in his right groin. He otherwise feels well. On examination, he has a
medial lying, minimally tender pulsatile lump. He tells you he recently suffered from a heart attack and had an
angioplasty. What is the most likely diagnosis?
a) False aneurysm
b) Femoral hernia
c) Groin abscess
d) Inguinal hernia
e) Saphena varix
Answer is a) False aneurysm
74) What of the following investigations are specific for autoimmune hepatitis?
a) ALT & AST
b) Alkaline phosphatase
c) GGT
d) ASMA
e) ANA
Answer is d) ASMA
75) Termination of pregnancy in cases of preterm premature rupture of membranes occur in the following conditions
EXCEPT:
a) The patient goes into active labor.
b) There are uterine contractions 1/15 minutes.
c) Fetal distress occurred.
d) Presence of fever after excluding extrauterine causes of fever
e) Cervix is 5 cm dilated.
Answer is b) There are uterine contractions 1/15 minutes.
76) A 35–year-old banker presents with intermittent lower abdominal pain, gaseous distension with a tendency
toward diarrhea whenever he is “stressed out”. He is a heavy cigarette smoker. Physical examination shows only
diffuse abdominal distension. The most important criteria of abdominal pain of this disease is:
a) Associated with a diarrhea
b) Association with rectal bleeding
c) Relieved with defecation
d) Relieved with urination
e) Relieved with vomiting
Answer is c) Relieved with defecation
77) A 23-year-old man presents with 6 weeks of increasingly frequent stools with blood-stained mucus mixed with
diarrhoea. There is generalized mild abdominal tenderness but no guarding. Temperature is 37.6°C. Which is the
single most likely diagnosis?
a) Crohn’s disease
b) Diverticular disease
c) Haemorrhoids
36
EMLE Trial Exams October 2020 (1000 Questions) Version (1.0) 9/12/2020

d) Rectal cancer
e) Ulcerative colitis
Answer is e) Ulcerative colitis
78) Patient suffering from duodenal ulcer was brought to the hospital in a state of shock indicating severe internal
hemorrhage, the blood vessel that could be eroded is:
a) Splenic artery
b) Portal vein
c) Gastroduodenal artery
d) Hepatic artery
e) Left gastroepiploic artery
Answer is c) Gastroduodenal artery
79) A primigravida at 26 weeks gestation and her CBC showing HB of 9.4 g/dL; MCV of 77fL and TIBC saturation <
15%. What is the INCORRECT statement regarding her management?
a) The recommended oral dose of elemental iron is 150 – 300 mg/day.
b) Oral iron therapy may cause constipation.
c) Cyanocobalamin is essential for normal erythropoiesis.
d) Parenteral iron therapy might induce teratogenic effects.
e) Adding folic acid to the iron formula is better than iron formula alone.
Answer is d) Parenteral iron therapy might induce teratogenic effects.
80) A 73-year-old woman has had a sudden onset of acute profuse rectal bleeding 4h ago. She has had no abdominal
pain. The pulse rate is 105bpm, blood pressure of 95/50mmHg, capillary refill time is 5sec and oxygen saturations
are 96% on supplemental oxygen via a non-re-breathing reservoir. Which is the single most appropriate first step
in her management?
a) Catheterize the patient
b) Insert two large bore cannula
c) Organize flexible sigmoidoscopy
d) Organize rigid sigmoidoscopy
e) Organize mesenteric angiogram
Answer is b) Insert two large bore cannula
81) Which of the following describes the patient’s right to participate in decisions about the diagnostic and treatment
recommendation
a) Autonomy
b) Beneficence
c) Non-maleficence
d) Justice
e) maleficence
The Answer is: a) Autonomy
82) A 78-year-old woman is found unresponsive to pain or speech on the ward by a nurse 3 days after carotid
endarterectomy. On examination there is no speech, no eye-opening to pain, and there is abnormal rigid
extension of her arms and legs. Calculate the Glasgow Coma Score (GCS) of this patient:
a) 0
b) 3
37
EMLE Trial Exams October 2020 (1000 Questions) Version (1.0) 9/12/2020

c) 4
d) 7
e) 9
The Answer is: C) 4
83) In the metabolism of iron, the following statements are correct EXCEPT:
a) Absorption occurs mainly in the duodenum and upper jejunum.
b) Absorption is increased when stores are depleted.
c) Traces of copper are essential for hematopoiesis.
d) Transferrin is increased in pregnancy.
e) Ferric salts are more readily absorbed than ferrous salts
The Answer is: e) Ferric salts are more readily absorbed than ferrous salts
84) A 27-year-old man who was the driver of a car involved in a high speed collision with a truck is brought into the
resuscitation room. On arrival, he is complaining of severe left-sided chest pain. You note that he is breathless,
tachycardic and hypotensive. He has reduced air entry on the left side of the chest and the trachea is deviated to
the right. What would you do next?
a) Insert a chest drain into the fifth intercostal space
b) Insert a wide bore cannula into the second intercostal space
c) Request an urgent chest X-ray
d) Request an urgent ECG
e) Perform a pericardiocentesis
The Answer is: b) Insert a wide bore cannula into the second intercostal space
85) Folic acid deficiency during pregnancy is associated with the following EXCEPT
a) Hypersegmented neutrophils
b) Megaloblastic erythropoiesis
c) Abruptio Placentae
d) Increased rate of systemic infections
e) Megaloblastic anemia
The Answer is: d) Increased rate of systemic infections
86) A 2 year old girl develops a rash on her legs. The next day she is brought to surgery, by which time the rash has
spread to the rest of her body . What is the most likely diagnosis ?
a) Erythema multiforme
b) Erytema chronica migrans
c) Erythema nodosum
d) Urticarial
e) Dermatitis artefacta
The Answer is: a) Erythema multiforme
87) Which of the following values may cause neonatal seizures:
a) Serum bicarbonate level 22mmol/L
b) Serum calcium is 5.5mg/dl
c) Random blood sugar 50 mg/dl
d) Serum sodium 138meq/l

38
EMLE Trial Exams October 2020 (1000 Questions) Version (1.0) 9/12/2020

e) Serum potassium 3.5 meq/L


The Answer is: b) Serum calcium is 5.5mg/dl
88) Indications of therapeutic abortion DO NOT include:
a) Advanced hypertensive vascular disease.
b) Missedabortion
c) Active pulmonary T.B.
d) Advanced chronic renal disease.
e) All cases of uncontrolled diabetes mellitus
The answer e) All cases of uncontrolled diabetes mellitus
89) What of the following is an absolute contraindication to thrombolytic in stroke? (Debatable)
a) Active pancreatitis
b) Esophageal varices
c) Hemorrhagic diathesis
d) Major surgery in preceding 2 weeks
e) pregnancy
The Answer is: b) Esophageal varices
90) Which is correct about vaginal discharge:
a) Cervical ectopy might induce vaginal discharge.
b) Normally the vagina is dry.
c) Progesterone causes a proliferation of the vaginal epithelium.
d) Atrophic vaginitis is uncommon in postmenopausal women.
e) Excessive vaginal discharge must indicate local infection
The answer is: Cervical ectopy might induce vaginal discharge.
91) Risk factors of breast cancer DO NOT include:
a) Genetic predisposition
b) Early menarche
c) Late menopause
d) Multiparity
e) Longevity
The Answer is : d) Multiparity
92) Intracranial calcifications on skull X-ray are seen in:
a) Infantile rickets
b) Congenital toxoplasmosis
c) Β-thalassemia major
d) Patau syndrome
e) Congenital rubella syndrome
The Answer is : b) Congenital toxoplasmosis
93) Which of the following IS NOT included in the biophysical profile for the risk assessment of a post- term fetus?
a) Fetal Breathing
b) Amniotic fluid volume
c) Fetal tone
39
EMLE Trial Exams October 2020 (1000 Questions) Version (1.0) 9/12/2020

d) Contraction stress test (CST)


e) Fetal motion
The answer is : Contraction stress test (CST) REVISE
94) About Mongolian spot in neonate:
a) Need urgent referral
b) Need surgery
c) Reassure
d) Antibiotics
e) Cortisone
The answer is : Reassure
95)The tocolytic agent that enhances the toxicity of magnesium to produce neuromuscular blockade is:
a) Nifedipine
b) Ritodrine
c) Indomethacin
d) Isopropranolol
e) Ethanol
The answer is: a) Nifedipine
96) A 50-year-old woman presents with tiredness, dyspnea and paraesthesia. On examination she showed extensor
planters, brisk knee jerks and absent ankle jerks. Her blood film shows macrocytic anemia. What is the most
probable cause?
a) Hemolytic anemia
b) Iron deficiency anemia
c) Pernicious anemia
d) Sickle-cell anemia
e) Sideroblastic anemia
The answer is: Pernicious anemia
97) A 30-year-old type 1 diabetic man is brought to the emergency department by his boyfriend. The boyfriend says
that the patient has been acting confused and drowsy after recently 'coming down with the flu'. A simple blood
test reveals marked hyperglycemia. A urinalysis shows the presence of ketones ++.What is the initial
management?
a) IV insulin
b) SC insulin
c) Buccal glucose gel
d) Potassium supplement
e) IV fluids
The answer is: IV fluids
98) What is the preferred treatment for an anaphylactic shock?
a) Epinephrine
b) Placing the patient in sitting position and administer oxygen
c) Preventing the reaction from occurring through patient teaching
d) Placing a bag of ice on the area, administer antihistamines and corticosteroids

40
EMLE Trial Exams October 2020 (1000 Questions) Version (1.0) 9/12/2020

e) Antibiotic therapy
The answer is : a) Epinephrine
99) A primigravida at 26 weeks gestation and her CBC showing HB of 9.4 g/dL; MCV of 77fL and TIBC saturation < 15%.
What is the INCORRECT statement regarding her management?
a) The recommended oral dose of elemental iron is 150 – 300 mg/day.
b) Oral iron therapy may cause constipation.
c) Parenteral iron therapy might induce teratogenic effects.
d) Adding folic acid to the iron formula is better than iron formula alone.
e) Cyanocobalamin is essential for normal erythropoiesis
The answer is : Parenteral iron therapy might induce teratogenic effects.
100) A 50-year-old woman with known gallstones presents to the emergency department with severe epigastric pain
radiating to the back together with nausea and vomiting. Examination reveals localized epigastric peritonitis and
investigations reveal an amylase of 650 μ/L.Which of the following would indicate a poor prognosis in this
condition?
a) Amylase of 650 μ/L
b) Arterial pO2 of 53 mmHg
c) Patient age of 50 years
d) Pyrexia of 38.5°C
e) White cell count of 10 × 103/μL
The answer is : Arterial pO2 of 53 mmHg

41
EMLE Trial Exams October 2020 (1000 Questions) Version (1.0) 9/12/2020

Exam 3
1) The INCORRECT STATEMENT REGARDING THE IUCDS:
a) COPPER IUCDS ARE EFFECTIVE FOR AT LEAST FIVE YEARS
b) IUCDS REDUCE THE CAPACITY OF SPERMS TO FERTILIZE THE EGG
c) THE RISK OF PELVIC INFLAMMATORY DISEASE IS SLIGHTLY INCREASED
d) THEY ARE CONTRAINDICATED IN WOMEN WITH SUBSEPTATE UTERUS
e) LEVONORGESTREL RELEASING DEVICES ARE ASSOCIATED WITH A SIGNIFICANT INCREASE IN BLOOD LOSS
DURING MENSTRUATION
The answer is: LEVONORGESTREL RELEASING DEVICES ARE ASSOCIATED WITH A SIGNIFICANT INCREASE IN BLOOD LOSS
DURING MENSTRUATION
2) A MUM ENTERING THE ER WITH HER INFANT 8 MONTH OLD WITH SWELLING 6 CM ON HIS HEAD AFTER HISTORY
OF HEAD TRAUMA WHAT IS THE RECOMMENDATION?
a) MRI
b) X RAY
c) URGENT CT
d) OBSERVE
e) REASSURE
The answer is: URGENT CT
3) A 55-YEAR-OLD MAN COMPLAINS OF ANOREXIA, WEIGHT LOSS, AND FATIGUE. AUGI STUDY DEMONSTRATES AN
ULCERATED LESION AT THE INCISURA. WHERE IS THE INCISURA?
a) CARDIA
b) FUNDUS
c) GREATER CURVATURE
d) LESSER CURVATURE
e) GASTROCOLIC LIGAMENT
The answer is: LESSER CURVATURE
4) A 55-YEAR-OLD MALE WITH HISTORY OF DIABETES AND HYPERTENSION PRESENTS TO THE EMERGENCY
DEPARTMENT WITH CHEST PAIN THAT STARTED 1 HOUR AGO AND DIDN’T RESOLVE WITH NITRATES. ECG
SHOWED ST DEPRESSION. TROPONIN IS ELEVATED. WHICH IS THE MOST LIKELY DIAGNOSIS?
a) STABLE ANGINA
b) UNSTABLE ANGINA
c) STEMI
d) NSTEMI
e) PERICARDITIS
The answer is: NSTEMI
5) YOU REVIEW A 24-YEAR-OLD WOMAN WITH A HISTORY OF ASTHMA IN THE EMERGENCY DEPARTMENT. SHE HAS
BEEN ADMITTED WITH ACUTE SHORTNESS OF BREATH ASSOCIATED WITH TONGUE TINGLING AND AN URTICARIAL
RASH AFTER EATING A MEAL CONTAINING SHELLFISH. HER SYMPTOMS SETTLE WITH NEBULIZED SALBUTAMOL
AND INTRAVENOUS HYDROCORTISONE . WHAT IS THE DIAGNOSTIC TEST OF ASTHMA SEVERITY?
a) ARTERIAL BLOOD GASES
b) SPIROMETRY
c) SERUM IGE
d) SERUM TRYPTASE
e) EOSINOPHIL COUNT
The answer is: SPIROMETRY

42
EMLE Trial Exams October 2020 (1000 Questions) Version (1.0) 9/12/2020

6) A 30-YEAR-OLD G3P2 IS 14 WEEKS PREGNANT. SHE HAD TWO PAINLESS DELIVERIES AT 16 WEEKS EARLIER. NEXT
LINE OF MANAGEMENT IS:
a) CERVICAL CERCLAGE
b) EVALUATION FOR DIABETES MELLITUS AND THYROID DISORDERS
c) CERVICAL LENGTH ASSESSMENT
d) TOCOLYTICS
e) NONE OF THE ABOVE
The answer is: CERVICAL LENGTH ASSESSMENT
7) A 35-YEAR-OLD WOMAN PRESENTS WITH A 2-MONTH HISTORY OF CHEST PAIN, DYSPHAGIA AND
REGURGITATION OF SOLIDS AND FLUIDS. BARIUM SWALLOW SHOWS BEAK-LIKE TAPERING OF THE LOWER THIRD
OF ESOPHAGUS. WHAT IS YOUR DIAGNOSIS?
a) BULBAR PALSY
b) ESOPHAGEAL CARCINOMA
c) GERD
d) CARDIAC ACHALASIA
e) ESOPHAGEAL CANDIDIASIS
The answer is: CARDIAC ACHALASIA
8) A 34‐YEAR‐OLD MAN WAS INVOLVED IN A MOTORCYCLE ACCIDENT. ON ARRIVAL, HIS INITIAL SYSTOLIC BLOOD
PRESSURE (SBP) WAS 80/50 MM HG BUT SUBSEQUENTLY IMPROVED TO 120/65 MM HG AFTER 2 L OF NORMAL
SALINE BOLUS. HIS GLASGOW COMA SCALE (GCS) WAS 13. HE WAS TAKEN TO THE COMPUTERIZED SCAN (CT). HIS
HEAD CT SHOWED DEPRESSED SKULL FRACTURE WITH A SUBDURAL HEMATOMA. WHILE BEING TRANSPORTED
BACK TO THE TRAUMA BAY, HE AGAIN BECAME HYPOTENSIVE WITH SBP 85/50 MM HG. WHAT WOULD BE THE
MOST APPROPRIATE TREATMENT FOR THIS PATIENT?
a) ADMINISTER 2 L OF NORMAL SALINE AND CONTINUE TO OBSERVE
b) ADMINISTER 2 UNITS OF O‐ AND CONTINUE TO OBSERVE
c) ARTERIAL EMBOLIZATION BY INTERVENTIONAL RADIOLOGIST
d) ADMINISTER 2 UNITS OF O‐ AND TAKE THE PATIENT DIRECTLY TO SURGERY
e) TYPE AND CROSS AND WAIT FOR RADIOLOGIST REPORT
The answer is: ADMINISTER 2 UNITS OF O‐ AND TAKE THE PATIENT DIRECTLY TO SURGERY
9) A 50-YEAR-OLD WOMAN PRESENTS WITH TIREDNESS, DYSPNEA AND PARAESTHESIA. ON EXAMINATION SHE
SHOWED EXTENSOR PLANTERS, BRISK KNEE JERKS AND ABSENT ANKLE JERKS. HER BLOOD FILM SHOWS
MACROCYTIC ANEMIA. WHAT IS THE MOST PROBABLE CAUSE?
a) HEMOLYTIC ANEMIA
b) IRON DEFICIENCY ANEMIA
c) PERNICIOUS ANEMIA.
d) SICKLE-CELL ANEMIA
e) SIDEROBLASTIC ANEMIA
The answer is: PERNICIOUS ANEMIA.
10) MANAGEMENT OF BARTHOLIN ABSCESS DOES NOT INCLUDE:
a) INCISION AND DRAINAGE.
b) MARSUPIALIZATION.
c) EXCISION OF CHRONIC ABSCESS
d) SAUCERIZATION OPERATION.
e) WOOD CATHETER
The answer is: SAUCERIZATION OPERATION.

43
EMLE Trial Exams October 2020 (1000 Questions) Version (1.0) 9/12/2020

11) AS PART OF A RESEARCH PROJECT YOU ARE TRYING TO ASCERTAIN WHETHER THE USE OF DUMMIES IN INFANTS
IS LINKED TO SUDDEN INFANT DEATH SYNDROME. WHAT IS THE MOST APPROPRIATE FORM OF STUDY DESIGN?
a) COHORT
b) RCT
c) META-ANALYSIS
d) CROSS-SECTIONAL
e) CASE-CONTROL
The answer is: CASE-CONTROL
12) THE CORRECT STATEMENT FOR VASA PREVIA:
a) IT IS A COMMON SAFE ANOMALY OF THE PLACENTAL CIRCULATION
b) IT OCCURS WITH CIRCUMVALLATE PLACENTA
c) BLEEDING IS FROM THE FETAL CIRCULATION
d) THE FETAL MORTALITY WITH VAGINAL DELIVERY IS LOWER THAN WITH CESAREAN SECTION
e) COMPLICATIONS ARE MAINLY MATERNAL RATHER THAN FETAL
The answer is: BLEEDING IS FROM THE FETAL CIRCULATION
13) BREAST SELF EXAMINATION (BSE) INVOLVE ALL OF THE FOLLOWING EXCEPT:
a) ADVISED TO BE DONE ONCE PER MONTH TO ALL WOMEN ABOVE 20 YEARS OF AGE
b) THE BEST TIME IS POSTMENSTRUAL
c) THE AIM IS DETECTION OF THE SMALLEST LESIONS
d) IT IS TO BE CARRIED OUT IN MORE THAN ONE POSITION TO DEMONSTRATE LESIONS EMBEDDED WITHIN THE
FAT
e) MAMMOGRAPHY IS PERFORMED ONLY IF THERE IS POSITIVE FINDINGS ON BSE
The answer is: MAMMOGRAPHY IS PERFORMED ONLY IF THERE IS POSITIVE FINDINGS ON BSE
14) WHICH STATEMENT IS TRUE ABOUT URTICARIA/ANGIOEDEMA?
a) IT IS A RARE ALLERGIC DISORDER IN CHILDREN
b) NOT CAUSED BY LOCAL CONTACT WITH ALLERGEN
c) FOOD ALLERGY IS COMMON
d) EDEMA OCCURS INDEPENDENT PARTS OF THE BODY
e) TOPICAL CORTICOSTEROIDS ARE THE MAINSTAY OF THERAPY
The answer is: FOOD ALLERGY IS COMMON
15) A 37-YEAR-OLD MAN PRESENTED TO THE UROLOGY OUTPATIENT DEPARTMENT HAVING BEEN REFERRED FROM
THE FERTILITY SERVICES. HE HAS HAD MULTIPLE FAILED ATTEMPTS TO CONCEIVE A CHILD WITH HIS PARTNER. OF
NOTE, HE EXPERIENCES A DRAGGING SENSATION WITHIN HIS RIGHT TESTICLE WHICH OFTEN CAUSES HIM
DISCOMFORT. ON EXAMINATION OF THE SCROTUM, THE TESTICLE AND EPIDIDYMIS APPEARED NORMAL.
HOWEVER, THE CORD STRUCTURES WERE BOGGY AND FELT LIKE A ‘BAG OF WORMS.’ THIS DID NOT
TRANSILLUMINATE NOR IS IT TENDER. THERE IS AN ASSOCIATED COUGH IMPULSE.FROM THE FOLLOWING LIST,
SELECT THE NEXT MOST APPROPRIATE DIAGNOSIS ASSOCIATED WITH THESE FINDINGS.
a) VARICOCELES
b) EPIDIDYMAL CYST
c) HYDROCOELE
d) DIRECT INGUINAL HERNIA
e) INDIRECT INGUINAL HERNIA
The answer is: VARICOCELES

44
EMLE Trial Exams October 2020 (1000 Questions) Version (1.0) 9/12/2020

16) A 55-YEAR-OLD WOMAN PRESENTS WITH A NECROTIC, GANGRENOUS ULCER ON THE TIP OF HER LEFT SECOND
FINGER. SHE IS A NON-SMOKER AND GIVES A PAST HISTORY OF COLD PERIPHERIES, GASTRO-OESOPHAGEAL
REFLUX AND POOR WOUND HEALING. ON EXAMINATION, SHE HAS A GOOD VOLUME RADIAL AND ULNA PULSE
AND LONG, SPINDLY FINGERS. CRP IS 5 MG/L, AN ECG AND CHEST X-RAY ARE NORMAL. WHAT IS THE MOST
LIKELY CAUSE?
a) CERVICAL RIB
b) VASCULITIS
c) THROMBUS
d) EMBOLISM
e) RAYNAUD’S SYNDROME
The answer is: RAYNAUD’S SYNDROME
17) THE MEDIAL BOUNDARY OF THE HASSELL BACH TRIANGLE IS
a) LINA ALBA
b) LINA SEMILUNARIS
c) INFERIOR EPIGASTRIC ARTERY
d) INGUINAL LIGAMENT
e) INTERNAL RING
The answer is: LINA SEMILUNARIS
18) A 9-DAY-OLD BOY HAD A CIRCUMCISION YESTERDAY, BUT THE WOUND DID NOT STOP BLEEDING. ON
EXAMINATION HE IS PALE AND HAS TACHYCARDIA. THERE IS OOZING OF BLOOD AROUND THE CIRCUMCISION
WOUND. A CANNULA IS INSERTED AND A BLOOD CROSS-MATCH SENT. THERE IS NOW OOZING AROUND THE
CANNULA SITE. INVESTIGATION REVEALS: HB: 8.4 G/DL, NORMAL WBC AND PLATELET COUNTS, PROTHROMBIN
TIME: 15 SECONDS (CONTROL:12–15 S) ACTIVATED PARTIAL THROMBOPLASTIN TIME: >120 SECONDS (CONTROL:
25–35 S). WHAT IS THE MOST LIKELY DIAGNOSIS?
a) HEMOPHILIA A
b) IMMUNE THROMBOCYTOPENIC PURPURA
c) LIVER DISEASE
d) VITAMIN K DEFICIENCY
e) VON WILLEBRAND DISEASE
The answer is: HEMOPHILIA A
19) WHAT IS THE TREATMENT OF ACUTE ATTACKS OF MENIERE’S DISEASE?
a) IV IBUPROFEN
b) BETAHISTIDINE
c) BUCCAL OR INTRAMUSCULAR PROCHLORPERAZINE
d) IV MORPHINE
e) IV TRAMADOL
The answer is: BUCCAL OR INTRAMUSCULAR PROCHLORPERAZINE
20) A 47-YEAR-OLD ALCOHOLIC PRESENTS WITH A 24-HOUR HISTORY OF SEVERE EPI¬ GASTRIC PAIN RADIATING TO
THE BACK, NAUSEA AND VOMITING. O/E: EPIGASTRIC TENDERNESS AND RIGIDITY. INVESTIGATIONS SHOW
MACROCYTIC ANAEMIA, LEUKOCYTOSIS, HYPERGLYCAEMIA AND PROLONGED CLOTTING .WHAT IS THE CAUSE OF
HIS ABDOMINAL PAIN?
a) ACUTE APPENDICITIS
b) ACUTE PANCREATITIS
c) ASCENDING CHOLANGITIS
d) ACUTE MI
e) RUPTURED ABDOMINAL AORTIC ANEURYSM
The answer is: ACUTE PANCREATITIS
45
EMLE Trial Exams October 2020 (1000 Questions) Version (1.0) 9/12/2020

21) RIGHT SIDED SENSORY LOSS AFFECTING ARMS MORE THAN THE LEGS TOGETHER WITH RIGHT SIDED
HOMONYMOUS HEMIANOPIA. WHAT AREA IS THE STROKE MOST LIKELY TO BE AFFECTED?
a) ACA
b) MCA
c) PCA
d) ASA
e) AICA
The answer is: MCA
22) WHICH ONE OF THE FOLLOWING IS LEAST LIKELY TO PRECIPITATE HAEMOLYSIS IN A PATIENT WITH G6PD
DEFICIENCY?
a) BEANS
b) SEPSIS
c) CIPROFLOXACIN
d) PRIMAQUINE
e) PENICILLIN
The answer is: PENICILLIN
23) A 65-YEAR-OLD MAN COLLAPSES IN THE STREET. ON EXAMINATION HE HAS AN UMBILICAL MASS THAT IS
EXPANSILE AND PULSATILE. WHAT IS THE MOST LIKELY DIAGNOSIS?
a) HEPATOCELLULAR CARCINOMA
b) PSEUDO PANCREATIC CYST
c) PSOAS ABSCESS
d) ABDOMINAL AORTIC ANEURYSM
e) DIVERTICULOSIS
The answer is: ABDOMINAL AORTIC ANEURYSM
24) A CASE OF PREGESTATIONAL DM IS USUALLY CONTROLLED DURING PREGNANCY BY
a) DIET ONLY.
b) DIET AND ORAL HYPOGLYCEMICS
c) ORAL HYPOGLYCEMICS ONLY
d) DIET AND INSULIN THERAPY
e) STOPPAGE OF TREATMENT ONCE PREGNANCY OCCURS
The answer is: DIET AND INSULIN THERAPY
25) COMPLICATIONS OF EVACUATION AND CURETTAGE OF THE UTERUS DO NOT INCLUDE:
a) UTERINE PERFORATION.
b) HEMORRHAGE.
c) INFECTION.
d) ASHERMAN’S SYNDROME.
e) AMNIOTIC BAND SYNDROME.
The answer is: AMNIOTIC BAND SYNDROME.
26) AN 86-YEAR-OLD MAN IS BROUGHT TO A&E WITH A SUSPECTED SUBARACHNOID HAEMORRHAGE. ON
EXAMINATION HE IS MOANING INCOMPREHENSIBLY WITH HIS EYES CLOSED. NAIL-BED PRESSURE CAUSES LIMB
FLEXION AND EYE-OPENING. CALCULATE THE GLASGOW COMA SCORE (GCS) OF THIS PATIENT
a) 0
b) 3
c) 4
d) 7
e) 9
The answer is: 7
46
EMLE Trial Exams October 2020 (1000 Questions) Version (1.0) 9/12/2020

27) A 32-YEAR-OLD MAN HAS HAD SHARP AND PERSISTENT ANAL PAIN ASSOCIATED WITH DEFECATION FOR 4 WEEKS
. OCCASIONALLY, THERE ARE SMALL AMOUNTS OF FRESH BRIGHT BLOOD ON THE TOILET PAPER. THERE IS A
FISSURE PRESENT IN THE ANAL CANAL AT THE 6 O’CLOCK POSITION (POSTERIOR ANAL FISSURE). WHICH IS THE
SINGLE MOST APPROPRIATE TREATMENT FOR HER?
a) ANAL DILATATION UNDER GENERAL ANESTHESIA
b) BOTOX INJECTION
c) DILTIAZEM CREAM (CALCIUM CHANNEL BLOCKER)
d) LATERAL INTERNAL SPHINCTEROTOMY
e) PREDNISOLONE SUPPOSITORIES
The answer is: DILTIAZEM CREAM (CALCIUM CHANNEL BLOCKER)
28) A 25-YEAR-OLD LADY PRESENTS WITH A 2-WEEK HISTORY OF “GROWTHS” IN THE VULVAR REGION. ON
EXAMINATION, YOU FIND MULTIPLE “CAULIFLOWER” VERRUCOUS LESIONS ON THE LABIA MAJORA AND
MINORA.WHAT IS THE MOST LIKELY DIAGNOSIS IN THIS PATIENT? (Debatable)
a) CONDYLOMA LATA
b) CONDYLOMA ACUMINATUM
c) HERPES SIMPLEX TYPE 1
d) HERPES SIMPLEX TYPE 2
e) GENITAL ACROCHORDON (SKIN TAGS)
The answer is: CONDYLOMA ACUMINATUM
29) WHICH OF THE FOLLOWING CAN BE WORNG ON HANDS DURING PATIENT CARE?
a) A RING WITH A PLAIN BAND
b) FALSE NAILS
c) DISCIPLINE: INFECTION CONTROL
d) SUBJECT: HAND WASHING
e) DIFFICULTY LEVEL: EASY
The answer is: A RING WITH A PLAIN BAND
30) URTICARIA MAY BE OCCUR MOST COMMONLY IN AN INFANT 3 MONTH OLD FROM?
a) COW MILK
b) SCORPION BITE
c) ANIMAL DANDER
d) PARACETAMOL
e) PEANUTS
The answer is: COW MILK
31) DIAGNOSIS OF ECTOPIC PREGNANCY CAN BE RULED OUT IN ALMOST ALL CLINICALLY SIGNIFICANT CASES IF
a) TRANSVAGINAL SONOGRAPHY SHOWED NO ABNORMALITIES IN THE ADNEXAL REGION.
b) PATIENT IS USING THE IUCD FOR CONTRACEPTION.
c) SERUMB-HCG WAS NEGATIVE.
d) THERE IS NO PELVIC FLUID BY TRANSVAGINAL UTERINE ULTRASOUND.
e) PATIENT DID NOT MISS HER PERIOD.
The answer is: SERUMB-HCG WAS NEGATIVE.
32) THE CAUSATIVE ORGANISM TO CAUSE BRONCHIOLITIS IS:
a) RSV
b) E COLI
c) ADENO VIRUS
d) SARS
e) PARA INFLUENZA
The answer is: RSV
47
EMLE Trial Exams October 2020 (1000 Questions) Version (1.0) 9/12/2020

33) CEREBRAL PALSY IS MAINLY BY DEFINITION A:


a) MOTOR AFFECTION
b) SENSORY AFFECTION
c) AUTONOMIC AFFECTION
d) BEHAVIORAL PROBLEM
e) MENTAL PROBLEM
The answer is: MOTOR AFFECTION
34) A 64-YEAR-OLD WOMAN PRESENTS WITH A 2-DAY HISTORY OF INCREASING LEFT-SIDED ABDOMINAL PAIN WITH
FEVER. ON EXAMINATION, SHE HAS LOCALIZED PERITONISM IN THE LEFT ILIAC FOSSA. HER BLOOD TESTS REVEAL
A RAISED WHITE CELL COUNT AND C-REACTIVE PROTEIN. WHICH OF THE FOLLOWING IS THE MOST LIKELY
DIAGNOSIS?
a) CONSTIPATION
b) DIVERTICULAR DISEASE
c) DIVERTICULITIS
d) DIVERTICULOSIS
e) IRRITABLE BOWEL SYNDROME
The answer is: DIVERTICULITIS
35) ALL OF THE FOLLOWING ARE RECOGNIZED FEATURES OF HENOCH-SCHӦNLEIN PURPURA EXCEPT:
a) A MACULOPAPULAR SKIN RASH OVER THE BUTTOCKS
b) ABDOMINAL COLIC
c) GENIALIZED LYMPHADENOPATHY
d) HEMATURIA
e) GLOMERULONEPHRITIS
The answer is: GENIALIZED LYMPHADENOPATHY
36) ACCORDING TO GLASGOW COMA SCALE FOR A CHILD 3 YEARS OLD WHAT SCORE WILL YOU GIVE FOR A CHILD
WITH EYE OPENING SPONTANEOUS (FOR HIS EYE?)
a) 6
b) 4
c) 8
d) 10
e) 2
The answer is:4
37) A 6 YEARS OLD GIRL WITH OFFENSIVE VAGINAL BLEEDING IS BROUGHT BY HER MOTHER, THE BEST NEXT STEP IS:
a) LOCAL ESTROGEN.
b) EXAMINATION UNDER ANESTHESIA TO EXCLUDE RETAINED FOREIGN BODY.
c) PARENTERAL AMPICILLIN FOR INFECTION.
d) METASTATIC WORK-UP TO EXCLUDE MALIGNANCY.
e) MRI
The answer is: EXAMINATION UNDER ANESTHESIA TO EXCLUDE RETAINED FOREIGN BODY.
38) WHICH OF THE FOLLOWING IS NOT A COMPLICATION OF MASSIVE BLOOD TRANSFUSIONS?
a) HYPERCALCEMIA
b) HYPERKALEMIA
c) AIR EMBOLISM
d) HYPOTHERMIA
e) NON OF THE ABOVE
The answer is: HYPERCALCEMIA

48
EMLE Trial Exams October 2020 (1000 Questions) Version (1.0) 9/12/2020

39) WHICH IS NOT A COMPONENT OF SYSTEMIC INFLAMMATORY RESPONSE SYNDROME (SIRS)


a) TEMPERATURE.
b) WHITE BLOOD CELL (WBC) COUNT.
c) BLOOD PRESSURE.
d) HEART RATE.
e) PA CO2.
The answer is: BLOOD PRESSURE.
40) CLINICAL EVIDENCES OF CERVICAL CANCER MAY INCLUDE ALL OF THE FOLLOWING EXCEPT:
a) PERIMENOPAUSAL BLEEDING
b) UREMIA
c) POSTMENOPAUSAL BLEEDING
d) GASTRIC UPSET
e) SEVERE PELVIC PAIN
The answer is: GASTRIC UPSET
41) WHICH OF THE FOLLOWING STATEMENTS IS TRUE REGARDING PATIENTS WITH REBOUND TENDERNESS AND FREE
AIR SEEN ON PLAIN RADIOGRAPHS? (Debatable)
a) PARENTERAL STEROIDS ARE HELPFUL IN TREATING PERITONEAL INFLAMMATION.
b) CT SCANNING IS IMPORTANT IN SURGICAL PLANNING.
c) NARCOTIC ANALGESICS ARE CONTRAINDICATED BECAUSE THEY MAY MASK CHANGES IN THE ABDOMINAL
EXAMINATION
d) ABDOMINAL ULTRASOUND IS SUPERIOR TO CT SCANNING IN LOCALIZING AN INTRA-ABDOMINAL ABSCESS
e) NO FURTHER IMAGING STUDIES ARE INDICATED
The answer is: NARCOTIC ANALGESICS ARE CONTRAINDICATED BECAUSE THEY MAY MASK CHANGES IN THE
ABDOMINAL EXAMINATION
42) WHAT OF THE FOLLOWING IS AN ABSOLUTE CONTRAINDICATION TO THROMBOLYTIC IN STROKE? (Debatable)
a) ACTIVE PANCREATITIS
b) BP >200/120 MMHG
c) HEMORRHAGIC DIATHESIS
d) MAJOR SURGERY IN PRECEDING 2 WEEKS
e) PREGNANCY
The answer is: c) HEMORRHAGIC DIATHESIS
43) THE FOLLOWING STATEMENT IS TRUE ABOUT CHILDHOOD ALLERGIC RHINITIS:
a) SYMPTOMS ARE INTERMITTENT AND NEVER PERSISTENT
b) SYMPTOMS INCLUDE PAROXYSMAL SNEEZING AND NASAL ITCHING
c) DARK CIRCLES UNDER THE EYES HAVE NO RELATION TO ITS PATHOLOGY
d) THE CHILD WITH ALLERGIC RHINITIS RARELY DEVELOPS ASTHMA
e) SYSTEMIC STEROIDS ARE THE MAIN LINE OF THERAPY
The answer is: SYMPTOMS INCLUDE PAROXYSMAL SNEEZING AND NASAL ITCHING
44) A 35-YEAR-OLD WOMAN IS REFERRED BY HER GP WITH PAINFUL NODULAR SWELLING OF HER BREASTS
ASSOCIATED WITH HER MENSTRUAL CYCLE. SHE COMPLAINS OF CYCLICAL PAIN AND THE FEELING OF MULTIPLE
LUMPS IN THE BREASTS, SOME OF WHICH SHE FEELS ARE GROWING LARGER. WHAT IS THE MOST LIKELY
DIAGNOSIS?
a) BREAST ABSCESS
b) BREAST CYST
c) FIBROADENOMA
d) FIBROADENOSIS
e) FAT NECROSIS
49
EMLE Trial Exams October 2020 (1000 Questions) Version (1.0) 9/12/2020

The answer is: FIBROADENOSIS


45) A 59-YEAR-OLD MAN WITH A HISTORY OF TYPE 2 DIABETES IS DIAGNOSED WITH DIABETIC RETINOPATHY AND
REFERRED TO OPHTHALMOLOGY FOR ADDITIONAL MANAGEMENT. THE PATIENT'S PRIMARY CARE PHYSICIAN IS
INTERESTED IN REDUCING THE NUMBER OF PATIENTS IN THE PRACTICE WHO DEVELOP SIMILAR LONG-TERM
COMPLICATIONS FROM TYPE 2 DIABETES MELLITUS. WHICH ONE OF THE FOLLOWING IS THE MOST IMPORTANT
NEXT STEP?
a) DEVELOP AN INTERVENTION TO MONITOR BLOOD GLUCOSE LEVELS FOR ALL PATIENTS IN THE PRACTICE
b) UTILIZE THE PATIENT REGISTRY TO IDENTIFY HIGH-RISK PATIENTS COMPRISING THE TARGET POPULATION
c) TRAIN STAFF IN THE CLINIC TO IDENTIFY EARLY SIGNS OF RETINOPATHY
d) REQUEST TO HAVE AN OPHTHALMOLOGIST PERFORM FUNDUSCOPIC EXAMS ON ALL PATIENTS IN THE PRACTICE
e) PLACE A SIGN IN THE OFFICE DEPICTING THE DANGERS OF DIABETES
The answer is: UTILIZE THE PATIENT REGISTRY TO IDENTIFY HIGH-RISK PATIENTS COMPRISING THE TARGET
POPULATION
46) WHICH ONE OF THE FOLLOWING IS MOST LIKELY TO BE FOUND IN A PATIENT WITH DEQUERVAIN THYROIDITIS?
a) RAISED ESR
b) ANTI-TSH RECEPTOR STIMULATING ANTIBODIES
c) ANTI-THYROID PEROXIDASE ANTIBODIES
d) DECREASED TSH
e) CO-EXISTING TYPE 2 DM
The answer is: RAISED ESR
47) CHILD 5 YEARS IS CHOCKING IN A TOY IN FRONT OF YOU AND HE IS CONSCIOUS AND COUGHING EFFECTIVELY?
a) STARTS CPR
b) HEIMLICH MANEUVER
c) ENCOURAGE COUGHING
d) REASSURE
e) SEND FOR HOSPITAL ADMISSION
The answer is: ENCOURAGE COUGHING
48) MESALAZINE IS USED AS A TREATMENT AND MAINTENANCE OF REMISSION OF WHICH OF THE FOLLOWING
DISEASES?
a) BEHCET’S DISEASE
b) CELIAC DISEASE
c) PRIMARY DYSMENORRHEA
d) ULCERATIVE COLITIS
e) WHIPPLE’S DISEASE
The answer is: ULCERATIVE COLITIS
49) WHICH OF THE FOLLOWING DESCRIBES THE PATIENT’S RIGHT TO PARTICIPATE IN DECISIONS ABOUT THE
DIAGNOSTIC AND TREATMENT RECOMMENDATION?
a) AUTONOMY
b) BENEFICENCE
c) NON-MALEFICENCE
d) JUSTICE
e) MALEFICENCE
The answer is: AUTONOMY
50) WHICH OF THE FOLLOWING IS NOT A PROBLEM ASSOCIATED WITH SURGERY IN THE JAUNDICED PATIENT?
a) CLOTTING DISORDERS.
b) HEPATORENAL SYNDROME.
c) INFECTION.
50
EMLE Trial Exams October 2020 (1000 Questions) Version (1.0) 9/12/2020

d) POOR WOUND HEALING.


e) MYOCARDIAL INFARCTION.
The answer is: MYOCARDIAL INFARCTION.
51) YOU PERFORM A ROUTINE NEWBORN EXAMINATION ON A BABY WHO IS 72 HOURS OLD. WHICH ONE OF THE
FOLLOWING FEATURES REQUIRES FURTHER IMMEDIATE ASSESSMENT?
a) ACROCYANOSIS (CYANOSIS OF THE HANDS AND FEET)
b) A HEART MURMUR
c) WHITE VAGINAL DISCHARGE
d) BREAST ENLARGEMENT
e) SUBCONJUNCTIVAL HEMORRHAGES
The answer is: A HEART MURMUR
52) A 40-YEAR-OLD WOMAN PRESENTS WITH RECURRENT EPISODE OF VERTIGO ASSOCIATED WITH A FEELING OR
'FULLNESS' AND 'PRESSURE' IN HER EARS. SHE THINKS HER HEARING IS WORSE DURING THE ATTACKS. CLINICAL
EXAMINATION IS UNREMARKABLE. WHAT IS THE MOST LIKELY DIAGNOSIS?
a) MENIERE’S DISEASE
b) BENIGN PAROXYSMAL POSITIONAL VERTIGO
c) ACOUSTIC NEUROMA
d) CHOLESTEATOMA
e) SOMATIZATION
The answer is: MENIERE’S DISEASE
53) CAUSES OF DELAYED PUBERTY INCLUDE ALL OF THE FOLLOWINGS EXCEPT:
a) HYPOTHYROIDISM
b) PITUTARY ADENOMA
c) EXTENSIVE PHYSICAL TRAINING AS ATHLETES
d) MALNUTRITION
e) FUNCTIONING OVARIAN CYSTS
The answer is: FUNCTIONING OVARIAN CYSTS
54) THE SHAPE OF CERVIX (ON USG) WHICH INDICATES A COMPETENT OS IS:
a) T-SHAPED
b) Y-SHAPED
c) V-SHAPED
d) U-SHAPED
e) NONE OF THE ABOVE
The answer is: T-SHAPED
55) INITIAL METASTATIC DEPOSITS IN THE LIVER ARE BEST INDICATED BY:
a) INCREASED BILIRUBIN
b) DECREASED SERUM ALBUMIN
c) INCREASED ALKALINE PHOSPHATASE
d) INCREASED GAMMA GLOBULINS
e) INCREASED AST (ALT)
The answer is: INCREASED ALKALINE PHOSPHATASE
56) DELAYED BONE AGE MAY OCCUR IN ALL THE FOLLOWING EXCEPT
a) HYPOTHYROIDISM
b) FAMILIAL SHORT STATURE
c) ISOLATED GROWTH HORMONE DEFICIENCY
d) PSYCHOLOGICAL DEPRIVATION

51
EMLE Trial Exams October 2020 (1000 Questions) Version (1.0) 9/12/2020

e) CONSTITUTIONAL DELAY OF GROWTH & PUBERTY


The answer is: FAMILIAL SHORT STATURE
57) IN A CASE OF RECURRENT SPONTANEOUS ABORTION, THE FOLLOWING INVESTIGATION IS UNWANTED
a) HYSTEROSCOPY
b) TESTING FOR ANTIPHOSPHOLIPID ANTIBODIES
c) THYROID FUNCTION TESTS
d) TESTING FOR TORCH INFECTIONS
e) NONE OF THE ABOVE
The answer is: TESTING FOR TORCH INFECTIONS
58) WHICH OF THE FOLLOWING IS ASSOCIATED WITH FINGER CLUBBING?
a) ACHALASIA
b) GASTRO-OESOPHAGEAL REFLUX DISEASE
c) CARCINOMA HEAD OF PANCREAS
d) COELIAC DISEASE
e) CHRONIC PANCREATITIS
The answer is: COELIAC DISEASE
59) PERVAGINAL BLOODY DISCHARGE AND HEMOPTYSIS WITH RECENT COMPLETE HISTORY OF ABORTION RAISES
THE SUSPICION OF:
a) HYDATIFORM MOLE.
b) CHORIOCARCINOMA.
c) KRUKENBERG TUMOR.
d) ENDOMETRIALCANCER.
e) INCOMPLETE ABORTION.
The answer is: CHORIOCARCINOMA.
60) A 30-YEAR-OLD WOMAN, GRAVIDA 2, PARA 1, AT 28 WEEKS' GESTATION COMES TO THE PHYSICIAN FOR A
PRENATAL VISIT. SHE FEELS WELL. PREGNANCY AND DELIVERY OF HER FIRST CHILD WERE UNCOMPLICATED. SHE
HAS A HISTORY OF BIPOLAR DISORDER AND HYPOTHYROIDISM. SHE USES COCAINE ONCE A MONTH AND HAS A
HISTORY OF DRINKING ALCOHOL EXCESSIVELY, BUT HAS NOT CONSUMED ALCOHOL FOR THE PAST 5 YEARS.
MEDICATIONS INCLUDE QUETIAPINE, LEVOTHYROXINE, FOLIC ACID, AND A MULTIVITAMIN. HER TEMPERATURE IS
37.1°C (98.8°F), PULSE IS 88/MIN, AND BLOOD PRESSURE IS 115/75 MM HG. PELVIC EXAMINATION SHOWS A
UTERUS CONSISTENT IN SIZE WITH A 28-WEEK GESTATION. SERUM STUDIES SHOW A HEMOGLOBIN
CONCENTRATION OF 11.2 G/DL AND THYROID-STIMULATING HORMONE LEVEL OF 3.5 µU/ML. HER FETUS IS AT
GREATEST RISK OF DEVELOPING WHICH OF THE FOLLOWING COMPLICATIONS?
a) APLASIA CUTIS CONGENITA
b) CRETINISM
c) CHROMOSOMAL ABNORMALITY
d) NEURAL TUBE DEFECT
e) PREMATURE PLACENTAL SEPARATION
The answer is: NEURAL TUBE DEFECT
61) DETECTION OF MATERNAL ANTIBODIES IN THE FETAL BLOOD THAT HAD BEEN ATTACHED TO FETAL CELLS IS
ACCOMPLISHED BY:
a) INDIRECTCOOMBS’TEST
b) DIRECT COOMBS’ TEST
c) RADIOIMMUNOASSAY
d) ROSETTE TEST
e) ENZYME-LINKED ANTIGLOBULIN TEST
The answer is: DIRECT COOMBS’ TEST
52
EMLE Trial Exams October 2020 (1000 Questions) Version (1.0) 9/12/2020

62) THE MOST RELEVANT FEATURE OF ECTOPIC PREGNANCY IS:


a) PELVIC PAIN.
b) AMENORRHOEA
c) FAINTING.
d) VAGINALBLEEDING.
e) PELVIC MASS.
The answer is: PELVIC PAIN
63) WHICH ONE OF THE FOLLOWING STATEMENTS IS CORRECT REGARDING THE DEFINITION OF NEONATAL DEATHS?
a) NEONATAL DEATHS OCCUR ONLY UP TO THE FIRST 24 HOURS OF LIFE
b) NEONATAL DEATHS OCCUR ONLY UP TO THE FIRST 72 HOURS OF LIFE
c) NEONATAL DEATHS OCCUR ONLY UP TO THE FIRST 7 DAYS OF LIFE
d) NEONATAL DEATHS OCCUR ONLY UP TO THE FIRST 21 DAYS OF LIFE
e) NEONATAL DEATHS OCCUR ONLY UP TO THE FIRST 28 DAYS OF LIFE
The answer is: NEONATAL DEATHS OCCUR ONLY UP TO THE FIRST 28 DAYS OF LIFE
64) A 64-YEAR-OLD WOMAN PRESENTS WITH SEVERE EPIGASTRIC PAIN RADIATING TO HER BACK AND PROFUSE
VOMITING. HER SERUM AMYLASE IS 1800 U/L. HER INITIAL GLASGOW PANCREATITIS SCORE IS 2. AN INITIAL
ULTRASOUND SCAN SHOWS GALLSTONES IN THE GALLBLADDER AND A DILATED COMMON BILE DUCT. HER
RESPIRATORY RATE INCREASES TO 30 BREATHS/MIN, AND HER OXYGEN REQUIREMENTS INCREASE TO 40% FIO2.
A CHEST X-RAY SHOWS BILATERAL ALVEOLAR INFILTRATES. WHAT IS THE MOST LIKELY COMPLICATION OF ACUTE
PANCREATITIS?
a) ARDS
b) PARALYTIC ILEUS
c) PANCREATIC ABSCESS
d) PANCREATIC PSEUDOCYST
e) DISSEMINATED INTRAVASCULAR COAGULATION (DIC)
The answer is: ARDS
65) YOU ARE ASKED TO ASSIST THE LEAD SURGEON WITH A MIDLINE LAPAROTOMY IN THEATRE. THE PATIENT HAS
SMALL BOWEL OBSTRUCTION CONFIRMED BY CT IMAGING. BEFORE THE START OF THE OPERATION, YOU ARE
ASKED WHAT LAYERS, FROM SUPERFICIAL TO DEEP, WOULD BE CUT THROUGH DURING A MIDLINE LAPAROTOMY
INCISION. WHICH OF THE FOLLOWING IS THE MOST LIKELY ANSWER?
a) SKIN, SUBCUTANEOUS FAT, SCARPA'S FASCIA, EXTERNAL OBLIQUE, INTERNAL OBLIQUE, TRANSVERSALIS FASCIA,
EXTRAPERITONEAL FAT AND PERITONEUM
b) SCARPA'S FASCIA, SKIN, LINEA ALBA, TRANSVERSALIS FASCIA, EXTRAPERITONEAL FAT, SUBCUTANEOUS FAT AND
PERITONEUM
c) SKIN, SCARPA'S FASCIA, LINEA ALBA, TRANSVERSALIS FASCIA, EXTRAPERITONEAL FAT, SUBCUTANEOUS FAT AND
PERITONEUM
d) LINEA ALBA, SCARPA'S FASCIA, SKIN, EXTERNAL OBLIQUE, INTERNAL OBLIQUE, TRANSVERSALIS FASCIA,
EXTRAPERITONEAL FAT, SUBCUTANEOUS FAT AND PERITONEUM
e) SKIN, SUBCUTANEOUS FAT, SCARPA'S FASCIA, LINEA ALBA, TRANSVERSALIS FASCIA, EXTRAPERITONEAL FAT AND
PERITONEUM
The answer is: SKIN, SUBCUTANEOUS FAT, SCARPA'S FASCIA, LINEA ALBA, TRANSVERSALIS FASCIA, EXTRAPERITONEAL
FAT AND PERITONEUM
66) WHAT IS THE MOST SUITABLE THERAPY TO ABORT MIGRAINE ATTACKS?
a) PARACETAMOL
b) PROPRANOLOL
c) AMITRIPTYLINE
d) BISOPROLOL

53
EMLE Trial Exams October 2020 (1000 Questions) Version (1.0) 9/12/2020

e) PIZOTIFEN
The answer is: PARACETAMOL
67) A 25-YEAR-OLD FEMALE PRESENTS WITH HISTORY OF RECURRENT ABORTIONS WITH MICROTHROMBI DETECTED
ON PLACENTAL PATHOLOGY. THE MOST RELEVANT TEST FOR IDENTIFYING CAUSE IS
a) PROTHROMBIN TIME
b) BLEEDING TIME
c) DILUTE RUSSELL’S VIPER VENOM TIME
d) CLOT RETRACTION TIME
e) NONE OF THE ABOVE
The answer is: DILUTE RUSSELL’S VIPER VENOM TIME
68) A 24-YEAR-OLD WOMAN GRAVIDA 2, PARA 1 AT 24 WEEKS' GESTATION COMES TO THE PHYSICIAN FOR A
PRENATAL VISIT. SHE FEELS WELL. HER EARLIER PREGNANCY WAS UNCOMPLICATED. THIS IS HER 4TH PRENATAL
VISIT. SHE HAD AN ULTRASOUND SCAN 2 WEEKS AGO THAT SHOWED A LIVE INTRAUTERINE PREGNANCY
CONSISTENT WITH A 22-WEEK GESTATION WITH NO ANOMALIES. SHE HAD A NORMAL PAP SMEAR 2 YEARS AGO.
VITAL SIGNS ARE WITHIN NORMAL LIMITS. PELVIC EXAMINATION SHOWS A UTERUS CONSISTENT IN SIZE WITH A
24-WEEK GESTATION. HER BLOOD GROUP AND TYPE IS B POSITIVE. WHICH OF THE FOLLOWING IS THE MOST
APPROPRIATE NEXT STEP IN MANAGEMENT?
a) SERUM PAPP-A AND HCG LEVEL
b) ORAL GLUCOSE CHALLENGE TEST
c) CARDIOTOCOGRAPHY
d) RH ANTIBODY TESTING
e) SWAB FOR GBS CULTURE
The answer is: ORAL GLUCOSE CHALLENGE TEST
69) AN 8- YEAR- OLD GIRL PRESENTED WITH LOW GRADE FEVER AND DIFFUSE MACULOPAPULAR RASH. ON
EXAMINATION HER PHYSICIAN NOTED MILD TENDERNESS & SWELLING OF HER CERVICAL & OCCIPITAL LYMPH
NODES. THREE DAYS AFTER THE ONSET OF ILLNESS, THE RASH HAS VANISHED. WHAT IS THE MOST LIKELY
DIAGNOSIS?
a) MEASLES.
b) GERMAN MEASLES
c) SCARLET FEVER.
d) INFECTIOUS MONONUCLEOSIS
e) MUMPS
The answer is: GERMAN MEASLES
70) CHILD 8 YEARS HAS SHOCKED IN A FOOD IN FRONT OF YOU CANNOT COUGH BUT HE IS CONSCIOUS WHAT WILL
YOU DO?
a) HEIMLICH MANEUVER
b) CPR
c) ENCOURAGE COUGHING
d) SEND TO ER
e) CALL AMBULANCE
The answer is: HEIMLICH MANEUVER
71) WHICH ONE OF THE FOLLOWING IS THE MOST APPROPRIATE DIAGNOSTIC INVESTIGATION IN A YOUNG ADULT
COMPLAINING OF EPISODIC SHORTNESS OF BREATH AND WHEEZES?
a) PATCH TESTING
b) CT CHEST
c) SERIAL PEAK FLOW MEASUREMENT AT HOME AND AT WORK
d) IGE MEASUREMENT
54
EMLE Trial Exams October 2020 (1000 Questions) Version (1.0) 9/12/2020

e) SKIN PRICK TEST


The answer is: SERIAL PEAK FLOW MEASUREMENT AT HOME AND AT WORK
72) THE FOLLOWING STATEMENTS, ABOUT VAGINAL DISCHARGE, ARE CORRECT EXCEPT:
a) LEUCORRHEA MEANS EXCESSIVE AMOUNT OF NORMAL VAGINAL DISCHARGE.
b) ESTROGEN THERAPY INCREASES THE AMOUNT OF VAGINAL DISCHARGE
c) COLUMNAR EPITHELIUM OF THE CERVIX CONTRIBUTES TO THE NORMAL VAGINAL DISCHARGE.
d) THE AMOUNT OF VAGINAL DISCHARGE INCREASES ONLY DURING INFECTION
e) ATROPHIC VAGINITIS IS RELATIVELY COMMON IN POSTMENOPAUSAL WOMEN.
The answer is: THE AMOUNT OF VAGINAL DISCHARGE INCREASES ONLY DURING INFECTION
73) A 71-YEAR-OLD MAN CAME TO THE OUTPATIENT CLINIC COMPLAINING OF DIFFICULTY IN WALKING AND SHAKY
HANDS ONLY AT REST 3 MONTHS AGO. THERE WAS NO HISTORY OF TRAUMA OR PAST ILLNESS. GENERAL
EXAMINATION REVEALED EXPRESSIONLESS FACE, SLOW AND SLURRED SPEECH, HANDS TREMORS AT REST. NO
SENSORY ABNORMALITY BUT THE MOTOR SYSTEM SHOWED BRISK REFLEXES AND HYPERTONIA. THE GAIT WAS
QUICK, SHORT STEPS WITH DIMINISHED ASSOCIATED MOVEMENTS. WHICH OF THE FOLLOWING IS THE SITE OF
LESION IN THIS DISEASE?
a) CEREBELLUM
b) FRONTAL LOBE
c) LATERAL VENTRICLE
d) PARIETAL LOBE
e) SUBSTANTIA NIGRA
The answer is: SUBSTANTIA NIGRA
74) ALL OF THE FOLLOWING STATEMENTS WITH REGARD TO THE COPPER IUCD ARE CORRECT EXCEPT:
a) MODERN COPPER LUCDS ARE CLINICALLY EFFECTIVE AND SAFE FOR AT LEAST FIVE YEARS.
b) IT REDUCES THE NUMBER OF SPERM REACHING THE FALLOPIAN TUBE AND THEIR CAPACITY TO FERTILIZE THE
EGG
c) IT IS ASSOCIATED WITH A SLIGHT INCREASED RISK OF PID
d) IT IS CONTRAINDICATED IN WOMEN WITH IRREGULAR UTERINE BLEEDING
e) LEVONORGESTREL RELEASING DEVICES ARE ASSOCIATED WITH HEAVY MENSTRUAL BLEEDING
The answer is: LEVONORGESTREL RELEASING DEVICES ARE ASSOCIATED WITH HEAVY MENSTRUAL BLEEDING
75) A 2 9 -YEAR-OLD G 1 P 00 1 0 WOMAN WITH REGULAR MENSES ( EVERY 2 8 DAYSLASTING 5 DAYS) IS CURRENTLY
TRYING TO CONCEIVE. SHE HAS NOT USED CONTRACEPTION FOR 6 MONTHS . HER OVULATION PREDICTOR KIT
REVEALED AN LUTEINIZING HORMONE(
a) CYSTIC TERATOMA
b) ECTOPIC PREGNANCY
c) FOLLICULAR CYST
d) HEMORRHAGIC CORPUS LUTEUM CYST
e) SEROUS CYSTADENOMA
The answer is: FOLLICULAR CYST
76) WHAT IS THE MOST SPECIFIC AND SENSITIVE INVESTIGATION OF TIA?
a) CT BRAIN
b) MRI BRAIN
c) CRANIAL US
d) DUPLEX US ON NECK
e) DIAGNOSTIC ANGIOGRAPHY
The answer is: MRI BRAIN

55
EMLE Trial Exams October 2020 (1000 Questions) Version (1.0) 9/12/2020

77) A 3-DAY-OLD INFANT HAS HIS ROUTINE NEWBORN CHECK. YOU NOTICE BLUE AND BLACK MACULES ON HIS BACK
AND BUTTOCKS. HE WAS BORN BY NORMAL VAGINAL DELIVERY
AND HAS BEEN FEEDING WELL SINCE BIRTH. FROM THE
FOLLOWING LIST PICK THE DIAGNOSIS THAT FITS WITH THE
CLINICAL DESCRIPTION AND PICTURE .
a) BRUISING
b) CAPILLARY HEMANGIOMA
c) ERYTHEMA TOXICUM
d) MONGOLIAN BLUE SPOTS
e) PORT WINE STAIN
The answer is: MONGOLIAN BLUE SPOTS
78) A PRIMIGRAVIDA AT 26 WEEKS GESTATION AND HER CBC
SHOWING HB OF 9.4 G/DL; MCV OF 77FL AND TIBC SATURATION < 15%. WHAT IS THE INCORRECT STATEMENT
REGARDING HER MANAGEMENT?
a) THE RECOMMENDED ORAL DOSE OF ELEMENTAL IRON IS 150 – 300 MG/DAY
b) ORAL IRON THERAPY MAY CAUSE CONSTIPATION
c) PARENTERAL IRON THERAPY MIGHT INDUCE TERATOGENIC EFFECTS
d) ADDING FOLIC ACID TO THE IRON FORMULA IS BETTER THAN IRON FORMULA ALONE
e) CYANOCOBALAMIN IS ESSENTIAL FOR NORMAL ERYTHROPOIESIS
The answer is: PARENTERAL IRON THERAPY MIGHT INDUCE TERATOGENIC EFFECTS
79) AN 80-YEAR-OLD MALE PRESENTS TO THE EMERGENCY DEPARTMENT EXPERIENCING NUMBNESS IN HIS RIGHT
ARM WHICH RESOLVES AFTER 6 HOURS UPON BEING ADMITTED TO THE GENERAL MEDICAL WARD. HE HAS HAD
THREE EPISODES OF SIMILAR SYMPTOMS WITHIN THE LAST WEEK. HIS PAST MEDICAL HISTORY INCLUDES
HYPERTENSION, TYPE 2 DIABETES MELLITUS, STAGE 2 CHRONIC KIDNEY DISEASE, AND HAS SUFFERED A
MYOCARDIAL INFARCTION 10 YEARS AGO. ON CLINICAL EXAMINATION YOU DETECT A LEFT CAROTID BRUIT AND
SUSPECT HE IS EMBOLIZING FROM A PROBABLE ATHEROMATOUS PLAQUE. FROM THE FOLLOWING LIST, WHAT IS
THE MOST APPROPRIATE FIRST LINE INVESTIGATION TO EXCLUDE CAROTID ARTERY STENOSIS?
a) MAGNETIC RESONANCE ANGIOGRAPHY
b) PLAIN X-RAY NECK
c) CAROTID ANGIOGRAPHY
d) ULTRASOUND SCAN OF NECK
e) CAROTID DUPLEX SCAN
The answer is: CAROTID DUPLEX SCAN
80) A 60-YEAR-OLD MAN WITH IDIOPATHIC THROMBOCYTOPAENIC PURPURA ATTENDS THE PREOPERATIVE
ADMISSION CLINIC PRIOR TO ELECTIVE SPLENECTOMY. WHICH OF THE FOLLOWING IMMUNIZATIONS ARE GIVEN
ROUTINELY PRIOR TO SPLENECTOMY?
a) HEPATITIS B
b) PNEUMOCOCCUS
c) MEASLES/MUMPS/RUBELLA (MMR)
d) POLIO/DIPTHERIA/TYPHOID
e) TETANUS
The answer is: PNEUMOCOCCUS
81) HYPERTONIA IN AN INFANT MAY BE A MANIFESTATION OF:
a) NUTRITIONAL RICKETS
b) POLIOMYELITIS
c) DOWN SYNDROME
d) WARDING HOFFMAN DISEASE

56
EMLE Trial Exams October 2020 (1000 Questions) Version (1.0) 9/12/2020

e) CP (COMMENEST TYPE)
The answer is: CP (COMMENEST TYPE)
82) PRETERM IS DEFINED AS BEEN DELIVERED:
a) BEFORE 37 WEEKS
b) AFTER 39 WEEKS
c) AFTER 40 WEEKS
d) BEFORE 16 WEEKS
e) BEFORE 10 WEEKS
The answer is: BEFORE 37 WEEKS
83) AN 8-YEARS-OLD BOY BROUGHT TO THE ER AFTER ACCIDENTALLY TOUCHING A HOT IRON WITH FOREARM. ON
EXAMINATION, THE BURNED AREA HAS WEEPING BLISTERS CONTAINING CLEAR FLUID AND SEVERELY TENDER.
WHAT IS THE BURN DEPTH?
a) FIRST DEGREE.
b) SECOND DEGREE.
c) THIRD DEGREE.
d) FOURTH DEGREE.
e) SCALD.
The answer is: SECOND DEGREE.
84) A 35-YEAR-OLD FEMALE PRESENTS TO THE EMERGENCY DEPARTMENT FOLLOWING A DELIBERATE PARACETAMOL
OVERDOSE. WHICH ONE OF THE FOLLOWING FEATURES IS MOST INDICATIVE OF A CONTINUING HIGH RISK OF
SUICIDE?
a) STAGGERED OVERDOSE
b) MIXED OVERDOSE INVOLVING OTHER DRUGS
c) FEMALE GENDER
d) MADE EFFORTS TO AVOID HERSELF BEING FOUND BY FRIENDS AND FAMILY
e) CONSUMED 50G OF PARACETAMOL
The answer is: CONSUMED 50G OF PARACETAMOL
85) WHICH OF THE FOLLOWING PHYSICAL FINDINGS WOULD BE LEAST LIKELY ON EXAMINATION OF A CHILD WITH
MODERATE TO SEVERE ASTHMA?
a) TACHYPNEA
b) WHEEZING
c) CLUBBING
d) DECREASED AIR EXCHANGE OVER THE RIGHT MIDDLE LOBE
e) AN INCREASED ANTERIOR-POSTERIOR DIAMETER OF THE CHEST
The answer is: CLUBBING
86) PREGNANCY DOES NOT HAVE WHICH OF THE FOLLOWING EFFECTS ON DIABETIC WOMEN:
a) TENDENCY TOWARD KETOSIS IN THE FIRST TRIMESTER
b) TENDENCY TOWARD HYPOGLYCEMIA DURING EARLY PREGNANCY
c) INCREASE IN INSULIN REQUIREMENT DURING EARLY PREGNANCY
d) INCREASE IN INSULIN REQUIREMENT DURING LATE PREGNANCY
e) WOMEN WITH PREEXISTING BUT UNDETECTED OVERT DM, MAY BE INITIALLY DIAGNOSED DURING PREGNANCY.
The answer is: INCREASE IN INSULIN REQUIREMENT DURING EARLY PREGNANCY
87) DURING A REPAIR OF A PRIMARY INGUINAL HERNIA, YOU ARE ASKED TO NAME THE NERVE THAT IS LOCATED
WITHIN THE SPERMATIC CORD. WHICH OF THE FOLLOWING IS THE NERVE THAT IS FOUND OUTSIDE THE
SPERMATIC CORD AND WITHIN THE INGUINAL CANAL?
a) ILIOINGUINAL NERVE

57
EMLE Trial Exams October 2020 (1000 Questions) Version (1.0) 9/12/2020

b) GENITOFEMORAL NERVE
c) GENITAL BRANCH OF THE GENITOFEMORAL NERVE
d) ILIOHYPOGASTRIC NERVE
e) LATERAL FEMORAL CUTANEOUS NERVE
The answer is: ILIOINGUINAL NERVE
88) AN 85-YEAR-OLD MALE PATIENT WITH A HISTORY OF CHRONIC CONSTIPATION PRESENTS WITH ACUTE SEVERE
COLICKY ABDOMINAL PAIN AND ABSOLUTE CONSTIPATION. PLAIN ABDOMINAL FILM SHOWS A GROSSLY DILATED
OVAL OF LARGE BOWEL ARISING FROM THE LEFT LOWER QUADRANT. A DIAGNOSIS OF SIGMOID VOLVULUS IS
MADE. THE NEXT STEP IN MANAGEMENT IS:
a) LAPAROTOMY
b) SIGMOIDOSCOPY WITH FLATUS TUBE INSERTION
c) SIGMOID COLECTOMY WITH COLOSTOMY
d) BARIUM SWALLOW
e) COMPUTED TOMOGRAPHY
The answer is: SIGMOIDOSCOPY WITH FLATUS TUBE INSERTION
89) IN TRAUMA IMAGING, WHICH OF THE FOLLOWING STATEMENT IS FALSE?
a) IN A MULTIPLY INJURED PATIENT, CT OF HEAD AND SPINE SHOULD BE THE FIRST LINE OF IMAGING.
b) FOCUSED ASSESSMENT WITH SONOGRAPHY FOR TRAUMA HELPS IN DETECTING INTRAPERITONEAL FLUID AND
CARDIAC TAMPONADE.
c) CT SHOULD NOT BE USED WHEN A PATIENT IS UNSTABLE.
d) U/S IS USEFUL FOR DIAGNOSING OCCULT PNEUMOTHORAX.
e) CT IS THE MAIN IMAGING METHOD FOR INTRACRANIAL, INTRA-ABDOMINAL AND VERTEBRAL INJURIES.
The answer is: IN A MULTIPLY INJURED PATIENT, CT OF HEAD AND SPINE SHOULD BE THE FIRST LINE OF IMAGING.
90) A 30-YEAR-OLD PRIMIGRAVIDA PRESENTS AT 34 WEEKS GESTATIONAL AGE WITH BLOOD PRESSURE OF 170/100
MMHG, HEADACHE, EPIGASTRIC PAIN, VISUAL ABNORMALITIES AND +3 PROTEINURIA. BIOPHYSICAL PROFILE OF
THE FETUS IS 8/8. WHICH ONE OF THE FOLLOWING IS THE IMMEDIATE RESPONSE?
a) START MAGNESIUM SULFATE INTRAVENOUSLY.
b) PERFORM AN EMERGENCY C-SECTION.
c) GIVE BETACLOMETHASONE TO INDUCE FETAL LUNG MATURITY
d) PERFORM AN AMNIOCENTESIS TO ASSESS FETAL LUNG MATURITY
e) REPEAT THE BIOPHYSICAL PROFILE DAILY.
The answer is: START MAGNESIUM SULFATE INTRAVENOUSLY.
91) THE PATIENT’S NURSE PLACES A PERIPHERAL INTRAVENOUS CATHETER IN A 15-KG, 4-YEAR-OLD MALE AND
REQUESTS ORDERS FOR FLUID TYPE AND RATE. WHICH OF THE FOLLOWING IS THE MOST APPROPRIATE INITIAL
CHOICE (BOTH FLUID AND AMOUNT) FOR VOLUME RESUSCITATION IN THIS PATIENT?
a) PACKED RED BLOOD CELLS 150 ML
b) LACTATED RINGERS 30 ML
c) NORMAL SALINE (0.9% SODIUM CHLORIDE SOLUTION) 300 ML
d) ½ NORMAL SALINE (0.045% SODIUM CHLORIDE SOLUTION) 300 ML
e) 3% NORMAL SALINE 45 ML
The answer is: NORMAL SALINE (0.9% SODIUM CHLORIDE SOLUTION) 300 ML
92) A 10-YEAR-OLD GIRL PRESENTS WITH SYMPTOMS OF AN UPPER RESPIRATORY TRACT INFECTION. SHE IS
INCIDENTALLY NOTED TO BE HYPERTENSIVE ON EXAMINATION. HER SYSTOLIC BLOOD PRESSURE (BP) IS 116\75
MMHG, WHICH IS AT THE 95TH CENTILE FOR HER WEIGHT & HEIGHT. WHAT IS THE NEXT BEST STEP IN
MANAGEMENT?
a) ADMISSION TO HOSPITAL FOR ANTI-HYPERTENSIVE TREATMENT
b) NONE, AS HYPERTENSION IN CHILDHOOD IS DEFINED AS A BP>120 MMHG
58
EMLE Trial Exams October 2020 (1000 Questions) Version (1.0) 9/12/2020

c) BP MEASUREMENT USING THE SMALLEST CUFF AVAILABLE


d) RENAL BIOPSY
e) AMBULATORY BP MEASUREMENT
The answer is: AMBULATORY BP MEASUREMENT
93) CONCERNING DIABETES IN CHILDREN WHICH IS TRUE:
a) IS ALWAYS INSULIN INDEPENDENT
b) FRACTOSAMIN ASSAY IS A ROUTINE BIOCHEMICAL SCREENING
c) MORE COMMON IN MALES
d) INCREASED INCIDENCE IN CHILDREN WITH CONGENITAL RUBELLA
e) HYPOGLYCEMIA IS A RARE COMPLICATION
The answer is: INCREASED INCIDENCE IN CHILDREN WITH CONGENITAL RUBELLA
94) THE FIRST STEP IN THE ASSESSMENT OF THE POSTTERM GESTATION IS:
a) ULTRASOUND EXAMINATION
b) DETERMINATION OF THE TRUE LENGTH OF GESTATION
c) MEASUREMENT OF FETAL HEART RATE (FHR)
d) DETERMINATION OF AMNIOTIC FLUID VOLUME
e) CONTRACTION STRESS TEST
The answer is: DETERMINATION OF THE TRUE LENGTH OF GESTATION
95) WHICH OF THE FOLLOWING IS NOT A REASON FOR BREAKING PATIENT’S CONFIDENTIALITY?
a) SUICIDE
b) HOMICIDE
c) TB
d) HIV
e) INFLUENZA
The answer is: INFLUENZA
96) A 27-YEAR-OLD MAN WHO WAS THE DRIVER OF A CAR INVOLVED IN A HIGH SPEED COL¬LISION WITH A TRUCK IS
BROUGHT INTO THE RESUSCITATION ROOM. ON ARRIVAL, HE IS COMPLAINING OF SEVERE LEFT-SIDED CHEST
PAIN. YOU NOTE THAT HE IS BREATHLESS, TACHYCARDIC AND HYPOTENSIVE. HE HAS REDUCED AIR ENTRY ON
THE LEFT SIDE OF THE CHEST AND THE TRACHEA IS DEVIATED TO THE RIGHT. WHAT WOULD YOU DO NEXT?
a) INSERT A CHEST DRAIN INTO THE FIFTH INTERCOSTAL SPACE
b) INSERT A WIDE BORE CANNULA INTO THE SECOND INTERCOSTAL SPACE
c) REQUEST AN URGENT CHEST X-RAY
d) REQUEST AN URGENT ECG
e) PERFORM A PERICARDIOCENTESIS
The answer is: INSERT A WIDE BORE CANNULA INTO THE SECOND INTERCOSTAL SPACE
97) A 27-YEAR-OLD G 1 PO AT 1 5 WEEK'S GESTATION HAS A HISTORY OF CHRONIC HYPERTENSION. SHE PRESENTS
FOR HER FIRST PRENATAL VISIT. VITAL SIGNS: T 98°F, P 66 BEATS/MIN, R 1 6 BREATHS/MIN, BP 1 46/98 MMHG.
SHE STOPPED TAKING HER BP MEDICATION 6 WEEKS AGO WHEN SHE FOUND OUT THAT SHE WAS PREGNANT. OF
THE FOLLOWING MEDICATIONS, WHICH IS THE BEST CHOICE FOR TREATING CHRONIC HYPERTENSION IN
PREGNANCY?
a) HYDRALAZINE
b) HYDROCHLOROTHIAZIDE
c) LABETALOL
d) LISINOPRIL
e) ATENOLOL
The answer is: LABETALOL

59
EMLE Trial Exams October 2020 (1000 Questions) Version (1.0) 9/12/2020

98) A 36-YEAR-OLD MAN PRESENTS TO THE EMERGENCY DEPARTMENT WITH A SEVERE LEFT-SIDED HEADACHE WITH
PAIN AROUND THE LEFT EYE. HE HAS HAD SEVERAL SIMILAR EPISODES OVER THE LAST 2 WEEKS, LASTING 40-60
MINUTES EACH. THE HEADACHES ARE ASSOCIATED WITH A RUNNY NOSE. ON EXAMINATION, THERE IS REDNESS
AND TEARING OF HIS LEFT EYE. WHAT IS THE MOST APPROPRIATE ACUTE MANAGEMENT?
a) ACETAZOLAMIDE
b) HIGH FLOW O2
c) IBUPROFEN AND PARACETAMOL
d) SUMATRIPTAN
e) URGENT CT BRAIN
The answer is: HIGH FLOW O2
99) IRON DEFICIENCY ANEMIA IS CHARACTERIZED BY THE FOLLOWING EXCEPT:
a) MICROCYTIC HYPOCHROMIC ANEMIA
b) DECREASED IRON BINDING CAPACITY
c) DECREASED SERUM FERRITIN
d) PICA
e) DECREASED SERUM IRON
The answer is: DECREASED IRON BINDING CAPACITY
100) IN PRETERM THE CALORIC INTAKE SHOULD BE:
a) 60CAL/KG/DAY
b) 80CAL/KG/DAY
c) 120CAL/KG/DAY
d) 170CAL/KG/DAY
e) 200CAL/KG/DAY
The answer is: 120 CAL/KG/DAY

60
EMLE Trial Exams October 2020 (1000 Questions) Version (1.0) 9/12/2020

Exam 4
1) A 23-year-old.woman who is 10 weeks pregnant with a 3-week history of hyperemesis gravidarum is brought
to the emergency room after she collapsed and started fiting. what is your diagnosis?
a) Tonic-clonic seizures
b) Hyponatremia
c) Absence seizures
d) Hypomagnesemia
e) Hypocalcemia
The answer is: Hypomagnesemia
2) An otherwise well 2-year-old presents with extreme pallor. Growth & development are normal. The child is
pale but there is no lymphadenopathy or splenomegaly. Full blood count shows haemoglobin 4g\dl (40 g\l).
MCV 59. Other indicies are normal. What is the most appropiate management option?
a) Bone marrow aspirate
b) Chelation therapy for lead
c) Dietary review
d) Small bowel technetium scan
e) Start gluten-free diet
The answer is: Dietary review

3) All can cause delayed puberty EXCEPT:


a) Gonadal dysgenesis
b) Polycystic ovarian disease
c) Constitutional factor
d) Neoplasms:aspituitaryadenomas
e) Malnutrition as in Starvation & anorexia nervosa
The answer is: Polycystic ovarian disease
4) A 60-year-old smoker complains of a throbbing lump in his right groin. Examination reveals an expansile
pulsation in the mass.
a) maldescended testis
b) femoral aneurysm
c) hydrocoele
d) lipoma of the cord
e) inguinal hernia
The answer is: femoral aneurysm
5) A 70-year-old man comes into his doctor's office for a routine check-up. His family medical history is significant
for a father who died at 48 years old from colon cancer. He practices safe habits and always wears his seat belt
while driving. His health has been "great" for the past few years, although he is concerned about his wife
because she recently suffered a mild stroke. He denies any visual loss or motor or sensory weakness. The
patient’s physical examination is unremarkable. What is the leading cause of death in the patient's age group?
a) Heart disease
b) Malignancy
c) Motor vehicle crashes D. Stroke
d) Suicide
61
EMLE Trial Exams October 2020 (1000 Questions) Version (1.0) 9/12/2020

e) non of the above


The answer is: Heart disease
6) A 25-year-old man is brought into the emergency department following an assault. On arrival he has multiple
bruises over the left side of his chest and upper abdomen. On examination, he has abdominal tenderness
with guarding throughout. His observations include a heart rate of 132/min and blood pressure 86/42 mmHg.
A chest X-ray shows a lower rib fracture and elevated diaphragm on the left. Which of the following is the
most likely cause of his symptoms?
a) Hepatic injury
b) Left-sided haemothorax
c) Left-sided pneumothorax
d) Ruptured diaphragm
e) Splenic injury
The answer is: Splenic injury
7) The MOST SPECIFIC fetal malformation to diabetes mellitus is:
a) Ventricular septal defect.
b) Coarctation of the aorta.
c) Spina bifida
d) Sacral agenesis
e) non of the above
The answer is: Sacral agenesis
8) A healthy primigravida is admitted at 26 weeks' gestation with spontaneous rupture of the membranes.
The following statement is correct:
a) Fetal pulmonary hypoplasia may occur due to oligohydramnios
b) Prophylactic antibiotics are contraindicated
c) Corticosteroids are contraindicated because of the risk of infection
d) Active management results in a better perinatal outcome than is achieved by expectant management
e) The risk of preterm prelabor rupture of the membranes in subsequent pregnancies is 70%.
The answer is: Fetal pulmonary hypoplasia may occur due to oligohydramnios
9) Test not useful in case of tubal pregnancy is:
a) Pelvic examination
b) Ultrasonography
c) HCG levels
d) Hysterosalpingography (HSG)
e) non of the above
The answer is: Hysterosalpingography (HSG)
10) Which of the following are not immediately life-threatening injuries?
a) Tension pneumothorax
b) Cardiac tamponade
c) Flail chest
d) Open pneumothorax
e) Liver injury
The answer is: Liver injury

62
EMLE Trial Exams October 2020 (1000 Questions) Version (1.0) 9/12/2020

11) You are seeing a 28-year-old woman (gravida 3, para 2) with suspected UTI. To obtain a urine specimen, which
of the following should you order?
a) Clean-void midstream urine
b) Catheterization
c) Suprapubic tap
d) 24 hour urine
e) First morning void
The answer is: Clean-void midstream urine
12) A 6 years old girl with offensive vaginal bleeding is brought by her mother, the best next step is:
a) Local Estrogen.
b) Examination under anesthesia to exclude retained foreign body.
c) Parenteral ampicillin for infection.
d) Metastatic work-up to exclude malignancy.
e) MRI
The answer is: Examination under anesthesia to exclude retained foreign body.
13) A 65year-old man develops a cold, painful, left leg. No pulses are palpable throughout the left leg. He has
several risk factors for atherosclerotic disease. What is the best to do to differentiate between acute and
chronic ischemia clinically:
a) History of 40 pack-years of smoking
b) History of intermittent claudication
c) History of thrombotic stroke
d) Presence of femoro-popliteal bypass scar in the left leg
e) Presence of foot pulses in the right leg
The answer is: Presence of foot pulses in the right leg
14) Which of the following features would be associated with a lower risk of malignant change in a colonic
adenoma found on surveillance colonoscopy?
a) Size > 2 cm
b) Flat
c) Pedunculated
d) Villous architecture
e) Multiple polyps
The answer is: Pedunculated
15) The INCORRECT statement regarding missed abortion is:
a) It means retained dead embryo/ fetus.
b) Ultrasound is helpful in its diagnosis.
c) The uterus is usually felt smaller than the duration of pregnancy.
d) There may be brownish vaginal discharge.
e) There are exaggerated pregnancy symptoms.
The answer is: There are exaggerated pregnancy symptoms
16) A 64-year-old woman presents with severe epigastric pain radiating to her back and profuse vomiting. Her
serum amylase is 1800 U/L. Her initial Glasgow pancreatitis score is 2. An initial ultrasound scan shows
gallstones in the gallbladder and a dilated common bile duct. Her respiratory rate increases to 30 breaths/min,
and her oxygen requirements increase to 40% FiO2. A chest X-ray shows bilateral alveolar infiltrates. What is

63
EMLE Trial Exams October 2020 (1000 Questions) Version (1.0) 9/12/2020

the most likely complication of acute pancreatitis?


a) ARDS
b) Paralytic ileus
c) Pancreatic abscess
d) Pancreatic pseudocyst
e) Disseminated intravascular coagulation (DIC)
The answer is: ARDS
17) A case of pregestational DM is usually controlled during pregnancy by
a) Diet only
b) Diet and oral hypoglycaemics
c) Oral Hypoglycemics Only
d) Diet and insulin therapy
e) Stoppage of treatment once pregnancy occurs
The answer is: Diet and insulin therapy
18) A 25-year old with recently diagnosed ulcerative colitis is started on mesalazine after a recent tapering of
high dose steroids. Two weeks later, he develops severe pain in his epigastrium which radiates through to
his back. What is the most likely diagnosis?
a) Hepatitis
b) Acute pancreatitis
c) Primary sclerosing cholangitis
d) Duodenal ulceration
e) Acute coronary syndrome
The answer is: Acute pancreatitis
19) A 25-year-old female presents with history of recurrent abortions with microthrombi detected on
placental pathology. The most relevant test for identifying cause is
a) Prothrombin time
b) Bleeding time
c) Dilute Russell’s viper venom time
d) Clot retraction time
e) None of the above
The answer is: Dilute Russell’s viper venom time
20) A 36-year-old woman with HIV/AIDS and B-cell lymphoma is hospitalized for Clostridium difficile—associated
diarrhea. Following treatment, the patient is discharged home with a prescription for a 14-day course of oral
vancomycin. She is unable to fill the prescription at her local pharmacy because of a problem with her
insurance coverage. While awaiting coverage approval, she receives no treatment. Her symptoms soon return,
prompting an emergency department visit where she is diagnosed with toxic megacolon. Which of the
following should be addressed in order to bring about changes that improve patient safety?
a) Prescribing physician
b) Pharmacist
c) Insurance company
d) Patient
e) Discontinuity of care
The answer is: Discontinuity of care

64
EMLE Trial Exams October 2020 (1000 Questions) Version (1.0) 9/12/2020

21) A 50-year-old man is taking 75 mg aspirin and 75 mg of clopidogrel daily following a non-ST elevation
myocardial infarction 9 months ago. His blood pressure and cholesterol are well controlled. A recent exercise
ECG showed no evidence of ischaemia. He requires a laparoscopic cholecystetomy as an elective procedure
during the next month. Which of the following advice should be given regarding his haemostatic system?
a) Aspirin and clopidogrel can both be continued at this low dose
b) Both aspirin and clopidogrel should be discontinued as they are no longer indicated
c) Both aspirin and clopidogrel should be stopped 3 days prior to surgery with platelet function testing
performed prior to surgery
d) Both aspirin and clopidogrel should be stopped 7–10 days prior to surgery
e) Clopidogrel should be stopped 5 days prior to surgery and aspirin continued
The answer is: Both aspirin and clopidogrel should be stopped 7–10 days prior to surgery
22) Schistosomiasis causes which type of splenomegaly?
a) Immune hyperplasia
b) RES hyperplasia
c) Congestion
d) Infiltration
e) Neoplastic
The answer is: Congestion
23) 33 year-old G2 P1 stillborn infant. She had a screening test for diabetes mellitus at 10 weeks pregnancy. One-
hour post-prandial glucose test = 145 mg/dl. Follow-up should include which of the following?
f) No further testing and no need for treatment
g) A diet regimen with daily 2000-calories
h) Standard glucose tolerance test
i) Serial follow up with domiciliary glucose urine testing
j) Serial follow up with domiciliary blood sugar testing
The answer is: Standard glucose tolerance test
24) A 67-year-old man is diagnosed with a mid transverse colon tumour following a colonoscopy to investigate
rectal bleeding. What is the most appropriate surgical management?
a) Hartmann’s procedure
b) Sub-total colectomy
c) Anterior resection
d) Extended right hemicolectomy
e) Right hemicolectomy
The answer is: Extended right hemicolectomy
25) A 50-year-old patient presents with jaundice 5 days after receiving a red cell transfusion.
a) air embolus
b) delayed haemolytic transfusion reaction
c) hypocalcaemia
d) viral infection
e) circulatory overload
The answer is: delayed haemolytic transfusion reaction

65
EMLE Trial Exams October 2020 (1000 Questions) Version (1.0) 9/12/2020

26) A 61-year-old nulliparous woman comes to the physician for a follow-up examination. Her last Pap smear 3
years ago showed atypical squamous cells of undetermined significance. HPV testing was negative at that time.
On questioning, she has had fatigue and an increase in abdominal girth despite a 5-kg (11.0-lb) weight loss over
the past 6 months. She has gastroesophageal reflux disease and Hashimoto's thyroiditis. Menarche was at the
age of 10 years and her last menstrual period was 2 years ago. Current medications include omeprazole and
levothyroxine. Abdominal examination shows shifting dullness. There is tenderness to palpation of the right
lower quadrant but no guarding or rebound. Bimanual palpation shows a small uterus and a right adnexal
mass. Further evaluation of this patient is most likely to show which of the following findings?
f) Proliferation of endometrial glands
g) Elevated serum CA-125 level
h) Elevated serum beta-hCG level
i) Prolonged prothrombin time
j) Chocolate cyst of the right ovary
The answer is: Elevated serum CA-125 level
27) A 28-year-old woman presents with sudden onset severe right upper quadrant pain, jaundice and abdominal
distension. She has no other medical conditions and only takes the oral contraceptive pill. On examination
she is icteric, has a 3 cm palpable liver edge and shifting dullness. What is the most likely diagnosis?
a) Budd-Chiari syndrome
b) Pancreatitis
c) Ovarian carcinoma
d) Alcoholic liver disease
e) Cardiac failure
The answer is: Budd-Chiari syndrome
28) A 27-year-old athlete presents with a sudden onset of severe headache associated with nausea and
vomiting. O/E: GCS is 8/15 and he has marked neck stiffness. He is apyrexial. Fundoscopy reveals
subhyaloid. What is the cause of his headache?
a) Giant cell arteritis
b) Subarachinoid hemorrhage
c) Tension headache
d) Cluster headache
e) Migraine
The answer is: Subarachinoid hemorrhage
29) A 55-year-old lady presenting to outpatient department with postmenopausal bleeding for 3 months has a 1
× 1 cm nodule on the anterior lip of cervix. The most appropriate investigation to be done subsequently is:
a) Pap smear
b) Punch biopsy
c) Endocervical curettage
d) Colposcopy
e) Non of the above
The answer is: Punch biopsy
30) A 62-year-old man presents with a 3 week progressive jaundice, dark urine, and pale stool. Investigations
show high direct bilirubin. What is the most probable diagnosis?
a) Acute pancreatitis
b) Chronic pancreatitis
66
EMLE Trial Exams October 2020 (1000 Questions) Version (1.0) 9/12/2020

c) Pancreatic cancer
d) Gall stones
e) Cholecystitis
The answer is: Pancreatic cancer
31) Normal infant can sit unsupported at age of
a) 5 - 4 months
b) 6 - 8 months
c) 9 - 10 months
d) 3 - 5 months
e) 3 - 7 months
The answer is: 6 - 8 months
32) The risk factors for an ectopic pregnancy include the following EXCEPT:
k) History of pelvic inflammatory disease
l) Previous ectopic pregnancy
m) Use of combined oral contraceptives
n) Assisted reproductive techniques
o) Previous tubal surgery
The answer is: Use of combined oral contraceptives
33) The most relevant feature of ectopic pregnancy is:
a) Pelvic pain.
b) Amenorrhoea
c) Fainting.
d) Vaginal bleeding.
e) Pelvic mass.
The answer is: Pelvic pain.
34) Urticaria may be occur most commonly in an infant 3 month old from?
a) Cow milk
b) Scorpion bite
c) Animal dander
d) Paracetamol
e) Peanuts
The answer is: Cow milk
35) Atopic dermatitis is characterized by:
a) never affect infants
b) Itchy
c) not related to food allergy
d) caused by bacterial infection
e) treated by antibiotics
The answer is: Itchy

67
EMLE Trial Exams October 2020 (1000 Questions) Version (1.0) 9/12/2020

36) Precautions to minimize bleeding during myomectomy DO NOT include:


a) Preoperative LH-RH analogue
b) Timing of the operation in the premenstrual phase
c) Single rather than multiple incisions
d) Midline rather than lateral incisions
e) Preoperative antibiotics
The answer is: Timing of the operation in the premenstrual phase
37) Which of the following statements regarding preoperative investigations is false?
a) Chest X-ray is routinely requested in all patients over 60 years old .
b) A ventricular ejection fraction of less than 35 per cent indicates a high risk of cardiac complications.
c) A body mass index (BMI) <15 is associated with significant hospital mortality.
d) ECG is usually required in patients above 65 years old
e) HIV testing requires patient’s consent.
The answer is: Chest X-ray is routinely requested in all patients over 60 years old .
38) Which one of the following is most likely to precipitate haemolysis in a patient with G6PD deficiency?
a) Penicillin
b) Paracetamol
c) Erythromycin
d) Ciprofloxacin
e) Ceftriaxone
The answer is: Ciprofloxacin
39) Chorea movements are associated with lesions in:
p) Basal ganglia
q) Anterior horn cells
r) Peripheral nerves
s) Neuromascular junction
t) Mascle
The answer is: Basal ganglia
40) A 35-year-old woman is referred by her GP with painful nodular swelling of her breasts associated with her
menstrual cycle. She complains of cyclical pain and the feeling of multiple lumps in the breasts, some of which
she feels are growing larger. What is the most likely diagnosis?
a) Breast abscess
b) Breast cyst
c) Fibroadenoma
d) Fibroadenosis
e) Fat necrosis
The answer is: Fibroadenosis
41) In a newborn suspected of having Down syndrome, which of the following investigations are routinely
indicated in all patients with Down syndrome regardless of the symptoms?
a) Chest X ray
b) Echocardiography
c) Head ultrasound
d) Renal ultrasound
68
EMLE Trial Exams October 2020 (1000 Questions) Version (1.0) 9/12/2020

e) Upper gastrointestinal (GI) series


The answer is: Echocardiography
42) A 45-year-old man collapses at home and is brought to accident and emergency. He has a fever at 39.5°C and
blood pressure is 90/60 mmHg, although he is in a lucid state. Bruises can be seen on his skin which he
remembers being present before he fell. Blood tests show the patient to have a normocytic anaemia with a
low platelet count and increased fibrin split products. The most likely diagnosis is:
a) Warm autoimmune haemolytic anaemia
b) Cold autoimmune haemolytic anaemia
c) Paroxysmal nocturnal haemoglobinuria
d) Disseminated intravascular coagulation
e) Thalassaemia minor
The answer is: Disseminated intravascular coagulation
43) Child 2 years old his weight is 12 kg what is the maintenance fluid requirement for him if he is
severely dehydrated?
a) 550 ml
b) 750 ml
c) 1100 ml
d) 1500 ml
e) 2000 ml
The answer is: 1100 ml
44) Which one of the following is the most important cause of jaundice presenting in the first 24 hours of life?
a) Prematurity.
b) Hemolytic disease of the newborn.
c) Breast milk jaundice.
d) Early-onset sepsis.
e) Physiologic jaundice
The answer is: Hemolytic disease of the newborn.
45) An 18 month-old boy who was previously well, developed fever and irritability for 3 days. He was not
coryzal. Mother tells that she kept giving him antipyretics and the fever subsided spontaneously on the
fourth day followed by the appearance of this widespread maculopapular rash. What is the most probable
diagnosis?
a) Erythema infectiosum
b) Roseola infantum
c) Rubella virus
d) Measles virus
e) Infectious mononucleosis
The answer is: Roseola infantum
46) 30-year-old primigravida presents at 34 weeks gestational age with blood pressure of 170/100 mmHg,
headache, epigastric pain, visual abnormalities and +3 proteinuria. Biophysical profile of the fetus is 8/8.
Which one of the following is the immediate response?
a) Start magnesium sulfate intravenously.
b) Perform an emergency C-section.
c) Give betaclomethasone to induce fetal lung maturity

69
EMLE Trial Exams October 2020 (1000 Questions) Version (1.0) 9/12/2020

d) Perform an amniocentesis to assess fetal lung maturity


e) Repeat the biophysical profile daily.
The answer is: Start magnesium sulfate intravenously.
47) A 16-year-old girl has an enlarging mass on the right side of her neck for the last six weeks. She has no
other symptoms. She has a two x two cm lymph node in the right anterior triangle of the neck and
several smaller cervical lymph nodes. Her tonsils are enlarged and covered by a grey membrane. What is
the most likely diagnosis?
a) Infectious mononucleosis
b) Leukaemia
c) Lymphoma
d) Sarcoidosis
e) Tuberculosis
The answer is: Infectious mononucleosis
48) Normal findings in a newborn baby includes all of the following except:
a) Vernix caseosa
b) Mongolian spot
c) A strawberry nevus
d) Vagina bleeding
e) Breast enlargement
The answer is: A strawberry nevus
49) A 56-year-old woman is brought into the resuscitation room. Three days ago she suffered an insect bite to
the abdomen which has now spread, causing redness across the whole of her abdomen. On examination,
she is confused, with a temperature of 38.3°C, heart rate 106/min and blood pressure 100/56 mmHg.
Which of the following is specific to the treatment of this type of shock?
a) Antibiotics
b) Antihistamines
c) Atropine
d) Fluids
e) Inotropes
The answer is: Antibiotics
50) A 39-year-old builder presents to the Emergency Department following a fall from a fourth floor scaffold with a
presumed head injury. His initial Glasgow Coma Score is 13 on admission, which falls 20 min later to 7/15.
What is the diagnosis?
a) Subdural haematoma
b) Subarachnoid haemorrhage
c) Hydrocephalus
d) Extradural haematoma
e) Intracerebral haemorrhage
The answer is: Extradural haematoma
51) A 33-year-old female presents 6 weeks after the birth of her first child with a two-week history of
polyarthralgia, fever and a skin rash. First-line investigations show: A 33-year-old female presents 6 weeks
after the birth of her first child with a two-week history of polyarthralgia, fever and a skin rash. First-line
investigations show: ESR High. What is the most likely diagnosis?
a) Antiphospholipid syndrome
70
EMLE Trial Exams October 2020 (1000 Questions) Version (1.0) 9/12/2020

b) Systemic sclerosis
c) Systemic lupus
d) Fibromyalgia
e) Rheumatoid arthritis
The answer is: Systemic lupus
52) The COMMONEST site of occurrence ectopic pregnancy is:
a) Isthmic or interstitial portion of the Fallopian tube
b) Ovary
c) Corneal end of the tube
d) Fimbrial end of the Fallopian tube
e) Ampullary portion of the fallopian tube
The answer is: Ampullary portion of the fallopian tube
53) Which of the following conditions is most likely to be associated with a vaginal pH of 4?
a) Atrophic vaginitis
b) Candidalvaginitis
c) Trichomonas vaginitis
d) Gardnerellavaginitis
e) Non of the above
The answer is: Candidalvaginitis
54) A 56-year-old man presents with painless jaundice, weight loss and anorexia. On examination there is a
palpable gallbladder. An ultrasound shows a dilated gallbladder with intrahepatic and extrahepatic duct
dilatation. There is a suspicious mixed echogenic lesion in the head of the pancreas. It is decided that he
requires an endoscopic retrograde cholangiopancreatography (ERCP) to obtain brushings from the pancreatic
duct. His clotting studies are prothrombin time 21 sec, activated partial prothrombin time 36 sec. You are
informed that his prothrombin time needs to be reduced to 13 sec for his ERCP. What is the most appropriate
pharmacotherapy to correct the prothrombin time?
a) Activated factor VII
b) Platelet transfusion
c) Tranexamic acid
d) Warfarin
e) Vitamin K
The answer is: Vitamin K
55) Which of the following statements regarding management of the acute wound is not correct?
a) A bleeding wound should be elevated and a pressure pad applied.
b) Clamps may sometimes need to be put on bleeding vessels blindly.
c) Anaesthesia may be required in the assessment of wounds.
d) A thorough debridement is essential.
e) Repair of all damaged structures may be attempted in a tidy wound.
The answer is: Clamps may sometimes need to be put on bleeding vessels blindly.
56) Diagnosis of ectopic pregnancy can be ruled out in almost all clinically significant cases if
a) Transvaginal sonography showed no abnormalities in the adnexal region.
b) Patient is using the IUCD for contraception.
c) Serum b-hCG was negative.

71
EMLE Trial Exams October 2020 (1000 Questions) Version (1.0) 9/12/2020

d) There is no pelvic fluid by transvaginal uterine ultrasound.


e) Patient did not miss her period.
The answer is: Serum b-hCG was negative
57) You are the FY1 in pre-assessment clinic and examining a 56-year-old woman with an ejection systolic
murmur you are suspicious is aortic stenosis. Which of the following features would suggest severe
disease?
a) A left ventricular heave
b) A quiet second heart sound
c) Radiation to the carotids
d) Loudest during expiration
e) Loudest at the left sternal edge
The answer is: A quiet second heart sound
58) The following statements, about vaginal discharge, are correct EXCEPT:
a) Leucorrhea means excessive amount of normal vaginal discharge.
b) Estrogen therapy increases the amount of vaginal discharge
c) Columnar epithelium of the cervix contributes to the normal vaginal discharge.
d) The amount of vaginal discharge increases only during infection
e) Atrophic vaginitis is relatively common in postmenopausal women.
The answer is: The amount of vaginal discharge increases only during infection
59) A 2 3-year-old.woman who is 10 weeks pregnant with a 3-week history of hyperemesis gravidarum is brought
to the emergency room after she collapsed and started fiting.What is the best drug to treat absence epilepsy?
a) Lamotrigine
b) Haloperidol
c) Midazolam
d) Ethusiximide
e) Phenytoin
The answer is: Ethusiximide
60) A 42-year-old woman presents with an upper midline mass that has been present for over a year. She has a
history of a partial gastrectomy for a per¬forated ulcer. On examination, the mass is 5 cm in size, soft, non-
tender and reducible. Which of the following is the most likely diagnosis?
a) Direct inguinal hernia
b) Epigastric hernia
c) Hiatus hernia
d) Incisional hernia
e) Paraumbilical hernia
The answer is: Incisional hernia
61) Allergic rhinitis may cause all of the following except?
a) Paroxysmal sneezing
b) Dark circles under the eyes
c) Itchy sensation in the pharynx
d) Snoring during sleep
e) purulent nasal discharge
The answer is: purulent nasal discharge
72
EMLE Trial Exams October 2020 (1000 Questions) Version (1.0) 9/12/2020

62) Child 3 years in the ER talks normally and interact you judgment for speech in Glasgow coma scale, what
score you will give?
a) 1
b) 2
c) 3
d) 4
e) 5
The answer is: 5
63) During a total thyroidectomy, the lead surgeon tells you that he must first identify and then preserve a nerve
that is situated near the thyroid gland. He also tells you that this nerve is at risk of injury during thyroidectomy
procedures. Which of the following nerves is he referring to?
a) External laryngeal nerve
b) Vagus nerve
c) Phrenic nerve
d) Recurrent laryngeal nerve
e) None of the above
The answer is: Recurrent laryngeal nerve
64) A 61-year-old man is admitted to the emergency department with sudden onset of a painful, cold, white right
leg. His radial pulse rate is 86bpm and its rhythm follows no discernible pattern throughout 30sec of palpation.
Abdominal examination is normal. No pulses are palpable in the right leg and ankle Doppler signals are absent.
An ECG confirms the arrhythmia but shows no signs of acute ischemia. Which is the single most likely diagnosis?
a) Abdominal aortic aneurysm
b) Aorto-iliac dissection
c) Atrial fibrillation
d) Deep vein thrombosis
e) Myocardial infarction
The answer is: Atrial fibrillation
65) Variable features of kwashiorkor include all of the following except:
a) Anemia
b) Ectodermal changes
c) GI manifestations
d) Growth retardation
e) Multiple vitamin and mineral deficiencies
The answer is: Growth retardation
66) An unsensitized Rh-negative woman in her second pregnancy is seen in her thirty-sixth week. She complains
of edema in her legs and some tingling in her left hand. The most appropriate action at this time is
a) Analysis of the husband’s blood type
b) Intramuscular Rh (anti-D) immune globulin
c) Assessment for possible preeclampsia
d) Rh antibody titer
e) Amniocentesis
The answer is: Assessment for possible preeclampsia

73
EMLE Trial Exams October 2020 (1000 Questions) Version (1.0) 9/12/2020

67) A 48-year-old smoker, who was diagnosed with COPD 10 years ago, is experiencing shortness of breath and
a productive cough with purulent sputum. These episodes have become more frequent within the last few
years. What is the best management if the chest x-ray shows a pneumothorax with a 2.5 cm rim of air and
no mediastinal shift?
a) Aspiration
b) Discharge
c) Intercostal drain insertion
d) Repeat chest X-ray
e) Hospitalization
The answer is: Intercostal drain insertion
68) When handling items which are soiled with bodily fluids, it’s important to wear gloves. Which of these would not be
suitable?
a) Latex
b) Neoprene
c) Polythene
d) All are suitable
e) None of them is suitable
The answer is: Polythene
69) A 31-year-old HIV patient presents with a 1-week history of dysphagia and odynophagia, mainly to solids.
Barrium swallow shows multiple esophageal ulcers. What is your diagnosis?
a) Bulbar palsy
b) Esophageal carcinoma
c) GERD
d) Cardiac achalasia
e) Esophageal candidiasis
The answer is: Esophageal candidiasis
70) Which of the following is considered to be an ADVANTAGE of
the equipment shown in this photo ?
a) Pressure release (pop-off ) valve makes over inflation less
likely.
b) Requires tight face mask seal to inflate the lungs.
c) Requires oxygen reservoir to provide high concentration of
oxygen.
d) Cannot give free- flow of oxygen through the mask.
e) Cannot be used for CPAP

The answer is: pressure release (pop-off ) valve makes over inflation less likely
71) A 24-year-old woman gravida 2, para 1 at 24 weeks' gestation comes to the physician for a prenatal visit. She
feels well. Her earlier pregnancy was uncomplicated. This is her 4th prenatal visit. She had an ultrasound scan
2 weeks ago that showed a live intrauterine pregnancy consistent with a 22-week gestation with no anomalies.
She had a normal Pap smear 2 years ago. Vital signs are within normal limits. Pelvic examination shows a
uterus consistent in size with a 24-week gestation. Her blood group and type is B positive. Which of the
following is the most appropriate next step in management?
a) Serum PAPP-A and HCG level

74
EMLE Trial Exams October 2020 (1000 Questions) Version (1.0) 9/12/2020

b) Oral glucose challenge test


c) Cardiotocography
d) Rh antibody testing
e) Swab for GBS culture
The answer is: Oral glucose challenge test
72) Duchenne muscular dystrophy is characterized by all except:
a) X-linked recessive disease
b) Proximal muscle weakness
c) Affects boys
d) Pseudohypertrophy of the calf muscles
e) Exaggerated knee jerk
The answer is: Exaggerated knee jerk
73) Which ONE of the following disorders is inherited as sex- linked recessive disorder?
a) Thalassemia
b) Idiopathic thrombocytopenia.
c) Glucose 6 phosphate dehydrogenase deficiency.
d) Von Willebrand’s disease.
e) Hereditary spherocytosis
The answer is: Glucose 6 phosphate dehydrogenase deficiency.
74) A 24-year-old patient came to the outpatient with a 6-week history of fever, bloody
diarrhea, and weight loss. On examination, he is clinically anemic and has aphthous
ulcer of the mouth and mild tenderness of the abdomen. Sigmoidoscopy shows
transmural inflammation and non-caseating granulomas of the colon and rectum. If
the patient also complains of that lesion on his shins: What is the diagnosis of this
skin lesion ?
a) Erythema nodosum
b) Pyoderma gangerosum
c) Dermatitis hepetitformis
d) Eczyma
e) Atopic dermatitis
The answer is: Erythema nodosum
75) Which one of the following is most likely to be found in a patient with thyrotoxicosis?
a) Raised ESR
b) Anti-TSH receptor stimulating antibodies
c) Anti-thyroid peroxidase antibodies
d) Decreased TSH
e) Co-existing type 2 DM
The answer is: Decreased TSH
76) Pervaginal bloody discharge and hemoptysis with recent complete history of abortion raises the suspicion of:
a) Hydatiform mole.
b) Choriocarcinoma.
c) Krukenberg tumor.
d) Endometrialcancer.
75
EMLE Trial Exams October 2020 (1000 Questions) Version (1.0) 9/12/2020

e) Incomplete abortion
The answer is: Choriocarcinoma.
77) Which of the following statements are true?
a) Intermittent claudication may be present at rest.
b) Intermittent claudication is commonly relieved by getting out of bed.
c) Intermittent claudication is most commonly felt in the calf.
d) Intermittent claudication distance is usually inconsistent on a day-to-day basis for a given patient.
e) Intermittent claudication is thought to be due to nerve compression
The answer is: c) Intermittent claudication is most commonly felt in the calf.
78) High fever occur in which infection
a) Rubella
b) Measles
c) Chicken box
d) Roseola Viral
e) Croup
The answer is: Measles
79) A 55-year-old male with history of Diabetes and hypertension presents to the emergency department with
chest pain that started 3 days ago after he had runny nose and sneezing for five days. ECG showed ST
elevation in all leads. Which is the best initial treatment?
a) Morphine
b) Oxygen
c) Warfarin
d) Heparin
e) Ibuprofen
The answer is: Ibuprofen
80) A 19-year-old man is brought to the emergency department by ambulance with a stab-wound to the right
upper quadrant (RUQ) of the abdomen. A FAST scan shows free fluid, and the patient is taken to the operating
room for an exploratory laparotomy. The findings are a non bleeding laceration of the right lobe of the liver
and a gallbladder laceration. Which of the following is TRUE?
a) The gallbladder injury can be treated with cholecystectomy .
b) Isolated gallbladder injuries are uncommon.
c) Bile is usually sterile.
d) The liver laceration does not require closed suction drainage.
e) A thorough exploration is not necessary if the bleeding is confined to the RUQ
The answer is: Isolated gallbladder injuries are uncommon.
81) healthy primigravida is admitted at 30 weeks' gestation with spontaneous rupture of the mem- branes.
The following statement is correct:
a) Fetal pulmonary hypoplasia may occur due to oligohydramnios.
b) Prophylactic antibiotics are contraindicated.
c) Corticosteroids are contraindicated because of the risk of infection.
d) Active management results in a better perinatal outcome than is achieved by expectant management.
e) The risk of preterm prelabor rupture of the membranes in subsequent pregnancies is 70%
The answer is: Fetal pulmonary hypoplasia may occur due to oligohydramnios
76
EMLE Trial Exams October 2020 (1000 Questions) Version (1.0) 9/12/2020

82) The following statements, about vaginal discharge, are correct EXCEPT
a) Leucorrhea means excessive amount of normal vaginal discharge
b) Columnar epithelium of the cervix contributes to the normal vaginal discharge
c) Atrophic vaginitis is relatively common in postmenopausal women.
d) Estrogen therapy increases the amount of vaginal discharge
e) The amount of vaginal discharge increases only during infection
The answer is: The amount of vaginal discharge increases only during infection
83) A 14-year-old boy presents with a 4-hour history of severe, sudden onset hemi-scrotal pain. He has vomited
several times. There is no history of trauma. On examination, the right hemi-scrotum is swollen, tender and
there is an absent cremasteric reflex. What is the diagnosis?
a) Epididymo-orchitis
b) Testicular torsion
c) Scrotal abscess
d) Scrotal haematoma
e) Testicular tumour
The answer is: Testicular torsion
84) What is the appropriate type of diuretic to help prevent reaccumulation of ascites in patients with liver cirrhosis?
a) Spironolactone
b) Frusemide
c) Thiazides
d) Osmotic diuretics
e) Carbonic anhydrase inhibitors
The answer is: Spironolactone
85) Which of the following features is most likely to indicate a life threatening asthmatic attack?
a) Cannot complete sentences
b) Oxygen saturation of 94% on room air
c) Respiratory rate of 42/min
d) Failure to improve after nebulized salbutamol
e) Normal PaCo2
The answer is: Normal PaCo2
86) A child 3 years old can localize pain only what score will be given for that in Glasgow coma scale (DEBATABLE)
a) 1
b) 3
c) 4
d) 5
e) 6
The answer is: 5
87) Which is not true regarding informed consent?
a) All risks of a procedure should be explained
b) Benefits of the procedure should be explained
c) Has to be written and cannot be oral
d) Can be revoked by the patient at any time
e) The operating doctor should take the consent himself
77
EMLE Trial Exams October 2020 (1000 Questions) Version (1.0) 9/12/2020

The answer is: Has to be written and cannot be oral


88) Five years old child presenting with lethargy, respiratory distress, wheezes, cold extremities, BP: 60/30, weak
pulse .He received an injection of an unknown drug just few minutes before developing these symptoms .
What is the MOST appropriate management?
a) ABC + IV steroids ± antihistamine
b) ABC + IM steroids ± antihistamine
c) ABC + Inhaled ß2 agonist
d) ABC + IM adrenaline
e) ABC + SC adrenaline
The answer is: ABC + IM adrenaline
89) A 29 -year-old G 1 P 00 1 0 woman with regular menses ( every 2 8 dayslasting 5 days) is currently trying to
conceive. She has not used Chapter 9: Gynecology 1 93 contraception for 6 months . Her ovulation predictor
kit revealed an luteinizing hormone (Debatable)
a) Cystic teratoma
b) Ectopic pregnancy
c) Follicular cyst
d) Hemorrhagic corpus luteum cyst
e) Serous cystadenoma
The answer is: Follicular cyst
I do not understand the question >>>it appears that something missed
90) During a regular check-up of an 8-yr-old child, you note a loud first heart sound with a fixed and widely split
second heart sound at the left upper left sternal border that does not change with respirations. The most
likely diagnosis is:
a) Atrial septal defect (ASD)
b) Ventricular septal defect
c) Isolated tricuspid regurgitation
d) Tetralogy of Fallot
e) Transposition of great arteries
The answer is: Atrial septal defect (ASD)
91) A 24-year-old handball player presents to the fracture clinic with a comminuted, displaced fracture of
the clavicle. What is the most appropriate treatment option?
a) Hanging cast
b) External fixation
c) Manipulation and cast application
d) Plating
e) Dynamic hip screw
The answer is: Hanging cast
92) Mesalazine is used as a treatment and maintenance of remission of which of the following diseases?
a) Behcet’s disease
b) Celiac disease
c) Primary dysmenorrhea
d) Ulcerative colitis
e) Whipple’s disease
78
EMLE Trial Exams October 2020 (1000 Questions) Version (1.0) 9/12/2020

The answer is: Ulcerative colitis


93) Side effects of progestins in treatment of endometriosis DO NOT
a) Depression.
b) Secondary amenorrhea.
c) Nausea.
d) Breakthrough bleeding.
e) Weight gain
The answer is: Secondary amenorrhea.
94) A 33-year-old female is diagnosed with a personality disorder by her community psychiatrist. She has struggled
to hold down a job as an assistant store manager as she often finds her colleagues to be lacking in morals or
values and is reluctant to delegate work to them. She feels that her colleagues are lazy and do not perform
their duties to a sufficiently high standard, as a result, she is often overwhelmed with outstanding tasks that
she cannot complete and ends up staying late to get things right. What personality disorder is she most likely
to have been diagnosed with? Depatable
a) Schizoid
b) Narcissistic
c) Borderline
d) Obsessive – compulsive
e) Dependant
The answer is: Obsessive – compulsive
95) the most common cause in newborns with LGA (large for gestational age) is
a) Maternal obesity
b) Beckwith-Wiedemann syndrome
c) Maternal diabetes
d) Wilms tumor
e) Simpson-Golabi-Behmel overgrowth syndrome (SGBS)
The answer is: Maternal diabetes
96) A 43-year-old woman undergoes open cholecystectomy. Intraoperative cholangiogram revealed multiple
stones in the CBD. Exploration of the CBD was performed to extract gallstones. The CBD was drained with a
#18 T-tube. After 10 days, a T-tube cholangiogram reveals a retained CBD stone. This should be treated by
which of the following?
a) Laparotomy and CBD exploration
b) Subcutaneous heparinization
c) Antibiotic therapy for 6 months and then reevaluation
d) Extraction of the stone through the pathway created by the T-tube (after 6 weeks)
e) Ultrasound crushing of the CBD stone
The answer is: Extraction of the stone through the pathway created by the T-tube (after 6 weeks)
97) The MOST COMMON congenital fetal malformation in diabetes mellitus is
a) Ventricular and atrial septal defect(CVS).
b) Anenchephaly.
c) Spina bifida.
d) Sacral agenesis.
e) None of the above

79
EMLE Trial Exams October 2020 (1000 Questions) Version (1.0) 9/12/2020

The answer is: Ventricular and atrial septal defect(CVS).


98) Which of the following is considered to be an ADVANTAGE of the equipment shown in this photo ?
a) Pressure release (pop-off ) valve makes over inflation less likely.
b) Requires tight face mask seal to inflate the lungs.
c) Requires oxygen reservoir to provide high concentration of
oxygen.
d) Cannot give free- flow of oxygen through the mask.
e) Cannot be used for CPAP

The answer is: pressure release (pop-off ) valve makes over inflation
less likely

99) An agitated and nervous 24-year-old woman has had severe wheezing and shortness of breath for 2 days.
After receiving oxygen, steroids, and salbutamol in the emergency room, her breathing improves. She is still
wheezing and now feels tremulous and anxious with a pulse of 110/min and respiratory rate 30/min. Arterial
blood gases on oxygen reveal a pH of 7.40, PO2 100 mm Hg, PCO2 40 mm Hg (35 to 45 mmHg), and
bicarbonate of 24 mEq/L (23 to 30 mEq/L). She is hospitalized for further treatment. Which of the following
treatments or medications should be avoided in her?
a) Corticosteroids
b) Intravenous (IV) fluids
c) Sedatives
d) Sympathomimetic amines
e) Theophylline
The answer is: Sedatives
100) A 30-year-old woman, gravida 2, para 1, at 28 weeks' gestation comes to the physician for a prenatal visit.
She feels well. Pregnancy and delivery of her first child were uncomplicated. She has a history of bipolar
disorder and hypothyroidism. She uses cocaine once a month and has a history of drinking alcohol excessively,
but has not consumed alcohol for the past 5 years. Medications include quetiapine, levothyroxine, folic acid,
and a multivitamin. Her temperature is 37.1°C (98.8°F), pulse is 88/min, and blood pressure is 115/75 mm Hg.
Pelvic examination shows a uterus consistent in size with a 28-week gestation. Serum studies show a
hemoglobin concentration of 11.2 g/dL and thyroid-stimulating hormone level of 3.5 µU/mL. Her fetus is at
greatest risk of developing which of the following complications?
a) Aplasia cutis congenita
b) Cretinism
c) Chromosomal abnormality
d) Neural tube defect
e) Premature placental separation
The answer is: Neural tube defect

80
EMLE Trial Exams October 2020 (1000 Questions) Version (1.0) 9/12/2020

Exam 5
1) A 42-year-old obese woman presents with a 9-month history of burning epigastric pain which is worse at night or
after oily food. Over the last 2 weeks she has developed occasional dysphagia to solids. Endoscopy shows hiatus
hernia, inflammation and stricture at the lower third of esophagus. what is your diagnosis? (Debatable)
a) Bulbar palsy
b) Esophageal carcinoma
c) GERD
d) Cardiac achalasia
e) Esophageal candidiasis
The answer is E
Debatable: Mostly Accepted is Achalasia but Answer is Candidiasis on site
2) A 16-year-old girl has an enlarging mass on the right side of her neck for the last six weeks. She has no other symptoms.
She has a two x two cm lymph node in the right anterior triangle of the neck and several smaller cervical lymph nodes.
Her tonsils are enlarged and covered by a grey membrane. What is the most likely diagnosis?
a) Infectious mononucleosis
b) Leukaemia
c) Lymphoma
d) Sarcoidosis
e) Tuberculosis
The answer is Infectious mononucleosis
3) Schistosomiasis causes which type of splenomegaly?
Immune hyperplasia
RES hyperplasia
Congestion
Infiltration
Neoplastic
The answer is: Congestion
4) A town with 1000 citizens has a 10% prevalence of disease X. A screening test for disease X was just developed,
with a sensitivity of 80% and a specificity of 70%. How many people without disease X will be falsely diagnosed
positive by this screening test?
a) 20
b) 80
c) 100
d) 270
e) 630
The answer is 270
5) A child 3 years old can localize pain only what score will be given for that in Glasgow coma scale
a) 3
b) 4
c) 5
d) 6
e) 1
The answer is 5

81
EMLE Trial Exams October 2020 (1000 Questions) Version (1.0) 9/12/2020

6) A 16-year-old girl presents to the Emergency Department with an exacerbation of her asthma. On examination,
her observations are respiratory rate 32 breaths/min, oxygen saturation is 91%, blood pessure 123/80 mmHg,
heart rate 110 bpm and temperature 36.5 c. There is widespread wheeze on auscultation of the chest. She is able
to talk, but is taking two or three breaths to complete a sentence. What is the most important therapy to relieve
her bronchoconstriction?
a) Propranolol
b) Salbutamol
c) Oxygen
d) Glucocorticosteroids
e) Cromoglicate
The answer is Salbutamol
7) A 13-year-old boy presents to the Emergency Department at 01:00 in the morning with a 2-hour history of acute
left-sided testicular pain and vomiting. He has had no recent history of a viral illness and has had a past medical
history of delayed testicular descent. On clinical examination, abdominal examination revealed no abnormality;
the left testicle is high riding, and is in a transverse lie. It is exquisitely tender on palpation. A urine dipstick
analysis is performed which shows no abnormality and no evidence of infection. From the following list, select the
next appropriate course of action:
a) Immediate out of hours ultrasound scan of scrotum
b) Wait until 08:00 and proceed to scrotal exploration
c) Immediate scrotal exploration
d) Urgent ultrasound scan of scrotum at 08:00
e) Discharge with outpatient ultrasound scan and urgent outpatient follow-up
The Answer is Immediate scrotal exploration
8) A 61-year-old nulliparous woman comes to the physician for a follow-up examination. Her last Pap smear 3 years
ago showed atypical squamous cells of undetermined significance. HPV testing was negative at that time. On
questioning, she has had fatigue and an increase in abdominal girth despite a 5-kg (11.0-lb) weight loss over the
past 6 months. She has gastroesophageal reflux disease and Hashimoto's thyroiditis. Menarche was at the age of
10 years and her last menstrual period was 2 years ago. Current medications include omeprazole and
levothyroxine. Abdominal examination shows shifting dullness. There is tenderness to palpation of the right lower
quadrant but no guarding or rebound. Bimanual palpation shows a small uterus and a right adnexal mass. Further
evaluation of this patient is most likely to show which of the following findings?
a) Proliferation of endometrial glands
b) Elevated serum CA-125 level
c) Elevated serum beta-hCG level
d) Prolonged prothrombin time
e) Chocolate cyst of the right ovary
The Answer is : Elevated serum CA-125 level
9) A 10-month-old female presents to the emergency department with a 2-day history of runny nose and fever . On
examination, her temperature is 103°F(39⁰C), HR 140 beats /minute, and a RR 30 / minute. She is alert and playful
with copious rhinorrhea. After the examination is complete, she becomes stiff and displays tonic-clonic
movements of all four extremities.Which of the following should be your first task?
a) Obtain whole blood glucose
b) Administer intravenous lorazepam (Ativan)
c) Perform lumbar puncture
d) Establish airway patency
e) Administer intramuscular fosphenytoin (Cerebyx)
The answer is Establish airway patency

82
EMLE Trial Exams October 2020 (1000 Questions) Version (1.0) 9/12/2020

10) A healthy primigravida is admitted at 30 weeks' gestation with spontaneous rupture of the mem- branes. The
following statement is correct:
a) Fetal pulmonary hypoplasia may occur due to oligohydramnios.
b) Prophylactic antibiotics are contraindicated.
c) Corticosteroids are contraindicated because of the risk of infection.
d) Active management results in a better perinatal outcome than is achieved by expectant management.
e) The risk of preterm prelabor rupture of the membranes in subsequent pregnancies is 70%
The answer is Fetal pulmonary hypoplasia may occur due to oligohydramnios
11) You are reviewing a 65-year-old in the renal clinic. He has been on hemodialysis for chronic kidney disease for
the past 6 years. What is he most likely to die from?
a) Hyperkalemia
b) Malignancy
c) Dilated cardiomyopathy
d) Dialysis related sepsis
e) Ischemic heart disease
The answer is: Ischemic heart disease
12) A patient refuses to undergo a necessary surgery. this is a demonstration of which of the following?
a) Autonomy
b) Confidentiality
c) Justice
d) Equity
e) beneficence
The answer is Autonomy
13) Which color bag should you use when disposing of clinical waste?
a) Red
b) Yellow
c) Green
d) Black
e) None of the above
The answer is Yellow
14) As regards the anterior pituitary the following are correct EXCEPT:
a) Develops from the alimentary tract
b) Dopamine reduces prolactin release from the pituitary
c) Is controlled by releasing factors produced in the hypothalamus
d) In short feedback loop: gonadotrophins reduce GnRH release
e) Lies above the optic chiasma
The answer is : Lies above the optic chiasma
15) All of the following conditions are causes of anemia in the neonatal period except:
a) Hereditary spherocytosis
b) Twin to twin transfusion
c) Β-thalassemia
d) Vitamin E deficiency
e) Septicemia
The answer is Β-thalassemia
16) A 32-year-old newly-married lady is complaining of watery vaginal discharge which has fishy odor especially post
coital. She denies any fevers. The likely diagnosis is:
a) Bacterial vaginosis
b) Monilial vaginitis

83
EMLE Trial Exams October 2020 (1000 Questions) Version (1.0) 9/12/2020

c) Trichomonas vaginitis
d) PID
e) Syphilis
The answer is Bacterial vaginosis
17) Which of the following statements about ischemia-reperfusion syndrome is correct?
a) This refers to the cellular injury because of the direct effects of tissue hypoxia.
b) It is seen after the normal circulation is restored to the tissues following an episode of hypoperfusion.
c) The increased -sodium load can lead to myocardial depression.
d) This is not influenced by the duration and extent of tissue hypoperfusion.
e) It usually does not cause death.
The answer is : It is seen after the normal circulation is restored to the tissues following an episode of
hypoperfusion.
18) A 51-year-old woman is diagnosed with invasive cervical carcinoma by cone biopsy Pelvic examination and
rectalvaginal examination reveal the parametrium to be free of disease, but the upper portion of the vagina is
involved with tumor. Intravenous pyelography (IVP) and sigmoidoscopy are negative, but a computed
tomography (CT) scan of the abdomen and pelvis shows grossly enlarged pelvic and periaortic nodes. This patient
is classified at
which of the following stages?
a) IIa
b) Ilb
c) IlIa
d) IlIb
e) IIIc
The answer is IIIc
19) A Culdocentesis is performed in a 19-year-old G1 P0 woman with lower abdominal pain and vaginal spotting. A
total of 3 cc of clotted blood is aspirated. Which one of the following is the best interpretation?
a) A hemoperitoneum is present.
b) No hemoperitoneum is present.
c) The blood probably came from a blood vessel.
d) The patient probably has an ectopic pregnancy.
e) None og the above
The answer is The patient probably has an ectopic pregnancy.
20) The correct statement for vasa previa:
a) It is a common safe anomaly of the placental circulation
b) It occurs with circumvallate placenta
c) Bleeding is from the fetal circulation
d) The fetal mortality with vaginal delivery is lower than with cesarean section
e) Complications are mainly maternal rather than fetal
The answer is Bleeding is from the fetal circulation
21) Which one of the following features is most indicative of a continuing high risk of suicide?
a) Severe depression
b) Psychosis
c) Female gender
d) Severe grief
e) Previous self-injury
The answer is Previous self-injury
22) A 36-year-old man presents to the emergency department with a severe left-sided headache with pain around the
left eye. He has had several similar episodes over the last 2 weeks, lasting 40-60 minutes each. The headaches are
associated with a runny nose. On examination, there is redness and tearing of his left eye.What is the first line
long-term preventative management of cluster headaches?
84
EMLE Trial Exams October 2020 (1000 Questions) Version (1.0) 9/12/2020

a) Verapamil
b) Carbamazepine
c) Lamotrigine
d) Phenytoin
e) sumatriptan
The answer is Verapamil
23) A patient with poorly controlled asthma is started on montelukast. What is the mechanism of action of this drug?
a) B2-receptor antagonist
b) B2-receptor agoinst
c) Leukotriene receptor agonist
d) Leukotriene receptor antagonist
e) Muscarinic receptor blocker
The answer is Leukotriene receptor antagonist
24) Which one of the following is the main benefit of erythropoietin in patients with end stage renal disease?
a) Reduces proteinuria
b) Corrects anemia and improve exercise intolerance
c) Reduces blood pressure
d) Improves renal functions
e) Reduces long-term all case mortality
The answer is Corrects anemia and improve exercise intolerance
25) What of the following is an absolute contraindication to thrombolytic in stroke? (Debatable)
a) Active pancreatitis
b) Esophageal varices
c) Hemorrhagic diathesis
d) Major surgery in preceding 2 weeks
e) Pregnancy
The answer is Esophageal varices
MOHP site Answer: Esophageal varices
26) Common symptom of VSD is:
a) Brain abcess
b) Repeated chest infections
c) Hemolysis
d) Squatting position
e) Cyanosis
The answer is Repeated chest infections
27) When is the age of weaning?
a) 3 months
b) 6 months
c) 9 months
d) 12 months
e) 15 months
The answer is 6 months
28) A 67-year-old woman is noted to have a gradual increase in the size of the abdomen. A CT scan reveals a large
pancreatic mass. The lesion was excised; on pathology examination, it is shown to be a TRUE cyst. Which
statement is correct regarding true cysts?
a) They are commonly seen in alcoholic pancreatitis.
b) They commonly occur after trauma.
c) They are frequently malignant

85
EMLE Trial Exams October 2020 (1000 Questions) Version (1.0) 9/12/2020

d) They are associated commonly with choledochocele.


e) They have an epithelial lining.
The answer is They have an epithelial lining.
29) A 25-year-old lady presents with a 2-week history of “growths” in the vulvar region. On examination, you find
multiple “cauliflower” verrucous lesions on the labia majora and minora.What is the most likely diagnosis in this
patient?
a) Condyloma lata
b) Condyloma acuminatum
c) Herpes simplex type 1
d) Herpes simplex type 2
e) Genital acrochordon (skin tags)
The answer is Condyloma acuminatum
30) A family came after car accident to the ER the child is 5 years old what is your recommendation for him if he
witnessed loss of conscious for 10 min?
a) Urgent CT
b) US
c) Neurological assessment
d) Reassure
e) Admit and observe
The answer is Urgent CT
31) You are called to review a 55-year-old man on the wards who started fitting around 5 minutes ago. He was
admitted three days ago following an acute coronary syndrome. His past medical history includes tonic-clonic
epilepsy which is generally well controlled on sodium valproate. On your arrival he is still fitting. Oxygen
saturations are 99% on 100% oxygen and his pulse rate is 96/min. Intravenous access is already in-situ. What is
the most appropriate next step?
a) Observation
b) IV lorazepam
c) IV phenytoin
d) Buccal midazolam
e) Call anaesthetic doctor
The answer is IV lorazepam
32) Which one of the following is most likely to be a contraindication for tocoloysis at 28 weeks gestation?
(Debatable)
a) Suspected abruption
b) Asymptomatic bacteriuria
c) Group B streptococcal bacteriuria
d) Uterine fibroid
e) Contracted pelvis
The answer is: Suspected abruption
33) A 33-year-old woman with a breast lump is asked if it is tender. When she says that it is tender, the clinician
concludes that the diagnosis is a cyst. No further history is obtained and the clinician fails to realize there has
been an increase in size, associated adenopathy and fixation to the chest wall (hence the tenderness), all
suggesting breast cancer.Which bias caused this adverse event?
a) Violation bias
b) Anchoring bias
c) Confirmation bias
d) Premature closure
e) Availability bias
The answer is Confirmation bias
86
EMLE Trial Exams October 2020 (1000 Questions) Version (1.0) 9/12/2020

34) A 47-year-old alcoholic presents with a 24-hour history of severe epi¬ gastric pain radiating to the back, nausea
and vomiting. O/E: epigastric tenderness and rigidity. Investigations show macrocytic anaemia, leukocytosis,
hyperglycaemia and prolonged clotting .what is the cause of his abdominal pain?
a) Acute appendicitis
b) Acute pancreatitis
c) Ascending cholangitis
d) Acute MI
e) Ruptured abdominal aortic aneurysm
The answer is Acute pancreatitis
35) A 37-year-old woman with right lower extremity edema is evaluated because of the sudden onset of shortness of
breath and pleuritic chest pain. A diagnosis of pulmonary embolism is made. Which of the following signs, if
present on physical examination, would be the most specific indicator of pulmonary arterial hypertension in this
patient?
a) Increased jugular venous pressure
b) P2 louder than A2
c) Peripheral edema
d) Presence of an S3
e) Pulmonary crackles
The answer is P2 louder than A2
36) What of the following is an absolute contraindication to thrombolytic in stroke? (DEBATABLE)
a) Active pancreatitis
b) Previous intracranial hemorrhage
c) Hemorrhagic diathesis
d) Major surgery in preceding 2 weeks
e) Pregnancy
The answer is Previous intracranial hemorrhage
37) Prioritizing young healthy patients over old frail patients in COVID19 epidemics regarding mechanical ventilation
spaces is a demonstration of which of the following?
a) Autonomy
b) Confidentiality
c) Justice
d) Equity
e) Beneficence
The answer is Equity
38) An 11-year-old white girl attended clinic. Three weeks ago. She presented to the emergency department with an
acute abdomen and was found to have numerous gallstones and splenomegaly on ultrasound of her abdomen.
She has no other past medical history of note, though her mother recalls required phototherapy for jaundice
when she was born what is the most likely diagnosis? Select one answer only
a) Sickle cell anaemia
b) Glucose-6-phosphate dehydrogenase (G6PD)deficency
c) Pyruate kinase deficiency
d) Herediatry spherocytosis
e) Haemoglobin C (HbC) disease
The answer is Herediatry spherocytosis
39) An 8- year- old girl presented with low grade fever and diffuse maculopapular rash. On examination her physician
noted mild tenderness & swelling of her cervical & occipital lymph nodes. Three days after the onset of illness, the
rash has vanished. What is the most likely diagnosis?
a) Measles.
b) German measles
c) Scarlet fever.
d) Infectious mononucleosis
87
EMLE Trial Exams October 2020 (1000 Questions) Version (1.0) 9/12/2020

e) Mumps
The answer is German measles
40) All of the following are characteristics of β-thalassemia major except:
a) Pallor since birth
b) Hepatosplenomegaly
c) Reticulocytosis
d) Presence of target cells in peripheral blood
e) Hair on end appearance on skull x-ray
The answer is Pallor since birth
41) The COMMONEST white lesion of the vulva
a) Squamous cell hyperplasia
b) Lichen sclerosis.
c) Carcinoma of the vulva
d) Paget’s disease of the vulva
e) Lichen planus
The answer is Lichen sclerosis.
42) A 76-year-old man with a haemoglobin of 7.9g/dL, mean corpuscular volume of 72fL, and mean corpuscular
haemoglobin of 32pg found on routine bloods tests during a GP medical check. choose the single most
appropriate first investigation:
a) Barium enema (double contrast)
b) Colonoscopy
c) CT scan of the abdomen and pelvis
d) Endoanal ultrasound scan
e) Flexible sigmoidoscopy
The answer is Colonoscopy
43) Which of the following is not a problem associated with surgery in obese patients?
a) Myocardial infarction.
b) Aspiration.
c) Deep venous thrombosis (DVT)/embolism.
d) Pressure sore.
e) Pain control.
The answer is Pain control.
44) Breast feeding has high content in which vitamin or mineral
a) Vitamin D
b) Iron
c) Vitamin K
d) Vitamin A
e) Vitamin C
The answer is Vitamin A
45) A 50‐year old cachectic Korean woman with known stage 4 gastric cancer presents with progressive oral
intolerance and weight loss. An upper gastrointestinal series demonstrates high‐grade gastric outlet obstruction.
Which of the following statements about management of malignant gastric outlet obstruction is true?
a) Gastrojejunostomy is the current standard of care for patients with malignant gastric outlet obstruction
b) Endoscopic stenting is contraindicated in malignant gastric outlet obstruction due to the risk of biliary sepsis
from ampullary obstruction
c) Compared to endoscopic stenting, gastrojejunostomy is associated with a higher proportion of late
complications including recurrent obstruction and cholangitis
d) Compared to gastrojejunostomy, endoscopic stenting is associated with better short‐term outcomes as well as
88
EMLE Trial Exams October 2020 (1000 Questions) Version (1.0) 9/12/2020

shorter hospital length of stay


e) Nasogastric decompression and nil per os will resolve malignant gastric outlet obstruction in 60% of patients
The answer is Compared to gastrojejunostomy, endoscopic stenting is associated with better short‐term outcomes as
well as shorter hospital length of stay
46) Management of Bartholin abscess DOES NOT include:
a) Incision and drainage.
b) Marsupialization.
c) Excision of chronic abscess
d) Saucerization operation.
e) Wood catheter
The answer is Saucerization operation.
47) A 64-year-old woman presents with a 2-day history of increasing left-sided abdominal pain with fever. On
examination, she has localized peritonism in the left iliac fossa. Her blood tests reveal a raised white cell count
and C-reactive protein. Which of the following is the most likely diagnosis?
a) Constipation
b) Diverticular disease
c) Diverticulitis
d) Diverticulosis
e) Irritable bowel syndrome
The answer is Diverticulitis
48) Male patient 60-years-old presented with progressive dysphagia more for food than to fluids for 4 months. On
examination patient is cachectic pulse rate was 100 B/min., B.P. 100/65 mmHg. What is the best investigation?
a) Upper endoscopy
b) PH monitoring study
c) Barium swallow
d) CT scan abdomen
e) Manometry
The answer is Upper endoscopy
49) Child presented to you with severe abdominal pain and red currant jelly stool and you diagnosed intussusception
what is your recommendation
a) Go to ER
b) Reassure
c) Anti emetic
d) Analgesic
e) Antibiotic
The answer is Go to ER
50) A 24-year-old lady presents to the emergency department by acute abdomen. She is married 8 months ago. Her
history is suggestive for recurrent vaginal infections. On examination she is stable. The costovertebral angle is free
but has tender lower abdomen with positive rebound. Ultrasound revealed free fluid in the pelvis. The best next
step to reach diagnosis will be:
a) Urine analysis with culture and sensitivity
b) Laparoscopy
c) Cordocentesis
d) B-HCG
e) Urine pregnant test
89
EMLE Trial Exams October 2020 (1000 Questions) Version (1.0) 9/12/2020

The answer is B-HCG


51) A 2 years old male presented by high grade fever and refusal of feeding for one day followed by appearance of
purpuric rash and extreme irritability. You should suspect
a) Measles
b) Scarlet fever
c) Meningococcemia
d) Typhoid fever
e) Mumps
The answer is Meningococcemia
52) Child 7 years old presents with purpuric eruptions on his legs, he complains of abdominal pain and some joint
discomfort, what is the condition he might have?
a) ITP
b) Henochen line purpura
c) Hemophilia
d) WVD
e) SLE
The answer Henochen line purpura
53) A 26-year-old para 1, pregnant at 8 weeks gestation, presented to the Reception Room with heavy vaginal
bleeding and opened cervical Os. The MOST COMMON cause of her condition is
a) Anatomic defects of the uterus or cervix.
b) Multiple pregnancy.
c) Endocrine factors.
d) Chromosomal anomalies.
e) Toxoplasmosis.
The answer Chromosomal anomalies.
54) Mother came complaining that her child Mona who is 13 years old has low ability to concentrate at school
recently she had her period, she looks slightly pale and usually feels tired what the first investigation you will do
is?
a) Chest X ray
b) ESR
c) CBC
d) Urine analysis
e) Nothing is needed
The answer is CBC
55) What is the level of BP at which thrombolytic can be given to stroke patients?
a) <200/100
b) <240/120
c) <185/110
d) <140/90
e) <120/90
The answer is <185/110
56) Anemia in pregnancy IS NOT related to
a) Iron deficiency
b) Chronic pyelonephritis
c) Folic acid deficiency
d) Multiple pregnancy
e) Amniotic fluid volume
The answer is Amniotic fluid volume

90
EMLE Trial Exams October 2020 (1000 Questions) Version (1.0) 9/12/2020

57) The following are true about urticaria/angioedema except:


a) It is a common allergic disorder in children
b) Not caused by local contact with allergens
c) Food allergy is a common cause
d) It is usually a self-limiting condition
e) Oral antihistamines are the main stay of therapy
The answer is Not caused by local contact with allergens
58) Diagnostic Criteria of SIRS Requires two or more of the following: DEBATABLE
T > 38 (or < 36)
HR > 90
RR > 20 (or pCO2 < 32)
WBC > 12,000 (or < 4000) or > 10% bands
There’s no relation between SIRS and blood pressure
58) Which is not a component of systemic inflammatory response syndrome (SIRS)?
a) Temperature.
b) White blood cell (WBC) count.
c) Blood pressure.
d) Heart rate.
e) Pa CO2.
The answer is Blood pressure.
59) A 34‐year‐old man was involved in a motorcycle accident. On arrival, his initial systolic blood pressure (SBP) was
80/50 mm Hg but subsequently improved to 120/65 mm Hg after 2 L of normal saline bolus. His Glasgow coma
scale (GCS) was 13. He was taken to the computerized scan (CT). His head CT showed depressed skull fracture with
a subdural hematoma. While being transported back to the trauma bay, he again became hypotensive with SBP
85/50 mm Hg. What would be the most appropriate treatment for this patient?
a) Administer 2 L of normal saline and continue to observe
b) Administer 2 units of O‐ and continue to observe
c) Arterial embolization by interventional radiologist
d) Administer 2 units of O‐ and take the patient directly to surgery
e) Type and cross and wait for radiologist report
The answer is Administer 2 units of O‐ and take the patient directly to surgery
60) All of the following will occur in renal failure except:
a) Metabolic acidosis
b) Hyperkalemia
c) ↑ serum creatinine
d) Hypertension
e) Jaundice
The answer is Jaundice
61) A 37-year-old woman, gravida 3, para 3, comes to the physician for very painful menses that have caused her to
miss at least 3 days of work during each menstrual cycle for the past 6 months. Menses occur with heavy bleeding
at regular 28-day intervals. She also has constant dull pain in the pelvic region between cycles. She is otherwise
healthy. She weighs 53 kg (117 lb) and is 160 cm tall; BMI is 20.7 kg/m2. Pelvic examination shows no
abnormalities. Pelvic ultrasonography shows a uniformly enlarged uterus and asymmetric thickening of the
myometrial wall with a poorly defined endomyometrial border. Which of the following is the most likely cause of
these findings?
91
EMLE Trial Exams October 2020 (1000 Questions) Version (1.0) 9/12/2020

a) Endometrial tissue within the uterine wall .


b) Endometrial tissue in the fallopian tubes
c) Neoplastic change of the endometrium
d) Cystic enlargement of the ovaries
e) Varicose veins in the lower abdomen
The answer is Endometrial tissue within the uterine wall .
62) A 26-year-old man comes to the physician because of sore throat, fever, and malaise for 1 week and a diffuse skin
rash for 1 day. The skin rash developed after the patient took ampicillin. Examination reveals pharyngitis and
tonsillitis, cervical lymphadenopathy, and splenomegaly. Laboratory studies showed: Hematocrit 40%, Leukocyte
count 4800/mm3, Segmented neutrophils 45%, Lymphocytes 40%, Platelet count 76,000/mm3, Alanine
aminotransferase 80 U/L (10 to 40 U/L), Aspartate aminotransferase 70 U/L (10 to 40 U/L). Total bilirubin 1.2
mg/dL. Peripheral blood smear showed numerous atypical large lymphocytes with vacuolated cytoplasm. A
heterophil antibody test is positive. Which of the following is the most likely diagnosis?
a) Acute cytomegalovirus (CMV) disease
b) Acute lymphocytic leukemia
c) Drug-induced thrombocytopenia
d) Infectious mononucleosis
e) Streptococcal pharyngitis with Leukemoid reaction
The answer is Infectious mononucleosis
63) A 19-year-old female with moon face and a hump on the upper back. On examination: pulse 90 bpm regular, BP
was150/90 mmHg and abdominal striae. What is the first lab test change found in this patient?
a) Disturbed FSH and LH levels
b) Low 24-hour urine cortisol
c) High morning cortisol level
d) Hyperkalemia
e) Loss of cortisol diurnal variation
The answer is Loss of cortisol diurnal variation
64) You see a patient on the Intensive Care Unit who has been admitted with severe pancreatitis. He is having a
central venous catheter inserted for intravenous fluid monitoring. Other than using a central venous cannula to
measure central venous pressure during fluid resuscitation, from the list below choose the answer which correctly
describes a long-term use of a central venous cannula.
a) Haemodialysis
b) Total parenteral nutrition
c) Pulmonary artery catheterisation
d) Drug administration
e) Transvenous cardiac pacing
The answer is Total parenteral nutrition
65) A 4 years old boy and has recently started school. He presents with fever and this widespread rash that is
itchy. Select the organism that is most likely to be the causative agent.
a) Chickenpox
b) Herpes simplex
c) Measles
d) Erythema infectiosum
e) Roseola infantum
The answer is Chickenpox

92
EMLE Trial Exams October 2020 (1000 Questions) Version (1.0) 9/12/2020

66) Acute pancreatitis associated with all EXCEPT:


a) ARDS
b) Pleural effusions
c) Ascites
d) Hypocalcaemia
e) Hepatocellular failure
The answer is Hepatocellular failure
67) A 37-year-old man presented to the Urology Outpatient department having been referred from the fertility
services. He has had multiple failed attempts to conceive a child with his partner. Of note, he experiences a
dragging sensation within his right testicle which often causes him discomfort. On examination of the scrotum,
the testicle and epididymis appeared normal. However, the cord structures were boggy and felt like a ‘bag of
worms.’ This did not transilluminate nor is it tender. There is an associated cough impulse.From the following list,
select the next most appropriate diagnosis associated with these findings.
a) Varicoceles
b) Epididymal cyst
c) Hydrocoele
d) Direct inguinal hernia
e) Indirect inguinal hernia
The answer is Varicoceles
68) A 46-year-old woman presents with a 12-hour history of severe epigastric pain radiating to her back. It is
associated with profuse vomiting. She is currently awaiting a cholecystectomy for gallstones, and her abdominal
X-ray reveals a sentinel loop of small bowel. What is the most likely diagnosis?
a) Pancreatitis
b) Ruptured AAA
c) Mesenteric ischaemia
d) Ascending cholangitis
e) Perforated duodenal ulcer
The answer is Pancreatitis
69) A 30-year-old woman (gravida 4, para 2, abortus 1) at 29 weeks’ gestation because of the sudden onset of
painless vaginal bleeding that soaked four perineal pads. The mother’s vital signs are normal, and the FHTs are
normal. At this time, what should you do?
a) Perform a double setup examination
b) Order an ultrasound examination
c) Perform a cesarean section
d) Send the patient home on bed rest
e) Hospital observation with tocolysis as necessary
The answer is Order an ultrasound examination
70) A 75-year-old man presents with severe central abdominal pain, radiating to the back and collapse. His past
medical history includes hypertension and angina. On examination, he is peripherally shut down, pale,
hypotensive and tachycardic with a distended abdomen and a tender epigastrium. What is the most likely
diagnosis?
a) Pancreatitis
b) Ruptured AAA
c) Mesenteric ischaemia
d) Ascending cholangitis
e) Perforated duodenal ulcer
The answer is Ruptured AAA

93
EMLE Trial Exams October 2020 (1000 Questions) Version (1.0) 9/12/2020

71) A 35-year-old man has walked into the A&E department after his involvement in a RTA. On arrival he is anxious,
later, he suddenly becomes faint. He is taken into the resuscitation bay where he is found to have a HR of 40/min
and a blood pressure of 75/46 mmHg and the patient has warm peripheries.Which is the most likely cause of his
symptoms?
a) Cardiogenic shock
b) Haemorrhagic shock
c) Hypovolaemic shock
d) Spinal shock
e) Neurogenic shock
The answer is Neurogenic shock
72) A 34-years old woman is complaining of recurrent itching and vaginal discharge. Wet mount preparation
reveals
flagellated organism. The next step will be:
a) Treat with Ketoconazole.
b) Screen for STDs.
c) Screen for diabetes.
d) Prescribe COCs
e) Assure the women
The answer is Screen for STDs.
73) The COMMONEST site of occurrence ectopic pregnancy is:
a) Isthmic or interstitial portion of the
b) Fallopian tube Corneal end of the tube
c) Ampullary portion of the fallopian tube
d) Fimbrial end of the
e) Fallopian tube Ovary
The answer is Ampullary portion of the fallopian tube
74) Infection transmitted through blood transfusion?
a) Scarlet fever
b) Hepatitis
c) Measles
d) Rubella
e) Chicken box
The answer is Hepatitis
75) Which of the following statements about hypertension in childhood is most factually correct?
a) Ambulatory BP measurements are acceptable in the vast majority of children due to discomfort
b) Hypertension in childhood is defined as a systolic blood pressure (BP)> 120 mmHg
c) The use of a small blood pressure cuff will produce a falsely low reading of blood pressure
d) The vast majority of cases of hypertension in children are secondary to endocrine disease.
e) The vast majority of children are asymptomatic but hypertensive encephalopathy presenting as severe
hypertension with headache, vomiting, hyperreflexia & seizures is medical emergency requiring admission to
hospital for controlled gradual decrease in BP.
The answer is The vast majority of children are asymptomatic but hypertensive encephalopathy presenting as
severe hypertension
with headache, vomiting, hyperreflexia & seizures is medical emergency requiring admission to hospital for
controlled gradual decrease in BP.

94
EMLE Trial Exams October 2020 (1000 Questions) Version (1.0) 9/12/2020

76) What is the thrombotic disorder that carries the highest risk of DVT?
a) Anti-thrombin III deficiency
b) Factor V Leiden mutation
c) Protein c deficiency
d) Prothrombin gene mutation
e) Protein S deficiency
The answer is Anti-thrombin III deficiency
77) An 85-year-old male patient with a history of chronic constipation presents with acute severe colicky abdominal
pain and absolute constipation. Plain abdominal film shows a grossly dilated oval of large bowel arising from the
left lower quadrant. A diagnosis of sigmoid volvulus is made. The next step in management is:
a) Laparotomy
b) Sigmoidoscopy with flatus tube insertion
c) Sigmoid colectomy with colostomy
d) Barium swallow
e) Computed tomography
The answer is Sigmoidoscopy with flatus tube insertion
78) A 66-year-old woman with known gallstones is admitted with acute pancreatitis. Her Glasgow pancreatitis score is
4 and she is admitted to ITU. She develops large purpuric bruises, epistaxis and oozing from her central line
insertion site. What is the most likely complication of acute pancreatitis?
a) ARDS
b) Paralytic ileus
c) Pancreatic abscess
d) Pancreatic pseudocyst
e) Disseminated intravascular coagulation (DIC)
The answer is Disseminated intravascular coagulation (DIC)
79) You are assisting with a total thyroidectomy for a patient who has a goitre and is experiencing compressive
symptoms. The Surgical Registrar asks you to name the artery supplying the thyroid gland, which originates from
the thyrocervical trunk. From the list below, choose the most likely answer.
a) Inferior thyroid artery
b) Thyroid ima artery
c) Superior thyroid artery
d) Ascending pharyngeal artery
e) Lingual artery
The answer is Inferior thyroid artery
80) Malaria causes which type of splenomegaly?
a) Immune hyperplasia
b) RES hyperplasia
c) Congestion
d) Infiltration
e) Neoplastic
The answer is Immune hyperplasia
81) Which of the following physical findings would be least likely on examination of a child with moderate to severe
asthma?
a) Tachypnea
b) Wheezing
c) Clubbing
d) Decreased air exchange over the right middle lobe

95
EMLE Trial Exams October 2020 (1000 Questions) Version (1.0) 9/12/2020

e) An increased anterior-posterior diameter of the chest


The answer is Clubbing
82) The most common cause in newborns with LGA (large for gestational age) is
a) Maternal obesity
b) Beckwith-Wiedemann syndrome
c) Maternal diabetes
d) Wilms tumor
e) Simpson-Golabi-Behmel overgrowth syndrome (SGBS)
The answer is Maternal diabetes
83) The most common small bowel malignancy is:
a) Adenoma
b) Adenocarcinoma
c) Carcinoid
d) Lymphoma
e) Leiomyosarcoma
The answer is Adenocarcinoma
84) A 36-year-old woman comes to the physician to discuss contraceptive options. She is currently sexually active
with her husband, and they have not been using any contraception. She has no significant past medical history
and takes no medications. She has smoked one pack of cigarettes daily for 15 years. She is allergic to latex and
copper. A urine pregnancy test is negative. Which of the following contraceptive methods is absolutely
contraindicated in this patient?
a) Diaphragm with spermicide
b) Progestin-only pill
c) Intrauterine device
d) Condoms
e) Combined oral contraceptive pill
The answer is Combined oral contraceptive pill
85) A 30-year-old woman in her last trimester of pregnancy suddenly develops massive swelling of the left lower
extremity. Which of the following would be the most appropriate workup and treatment at this time:
a) Venography and heparin
b) Duplex ultrasonography and heparin
c) Duplex ultrasonography, heparin, and vena caval filter
d) Duplex ultrasonography, heparin, warfarin (Coumadin)
e) Impedance plethymography, warfarin
The answer is Duplex ultrasonography and heparin
86) Child 3 years in the ER talks normally and interact you judgment for speech in Glasgow coma scale, what score
you will give?
a) 1
b) 2
c) 5
d) 4
e) 3
The answer is 5
87) Pattern of CIN lesions DO NOT include:
a) Regular surface contour
b) Stain brown with Lugol’s iodine
c) A marked acetowhite appearance
d) Coarseepithelialpunctations
96
EMLE Trial Exams October 2020 (1000 Questions) Version (1.0) 9/12/2020

e) Multisector involvement of the transformation zone


The answer is Stain brown with Lugol’s iodine
88) which feature could be consistent with a diagnosis of Bell’s palsy?
a) Hypersalivation
b) Hyperacusis
c) Mastication problems
d) Anesthesia of forehead
e) Anesthesia at jaw
The answer is Hyperacusis
89) Folic acid deficiency during pregnancy is associated with the following EXCEPT:
a) Hypersegmented neutrophils
b) Megaloblastic erythropoiesis
c) Increased rate of systemic infections
d) Megaloblastic anemia
e) Abruptioplacentae
The answer is Increased rate of systemic infections
90) Right sided sensory loss affecting arms more than the legs together with right sided homonymous hemianopia.
What area is the stroke most likely to be affected?
a) ACA
b) MCA
c) PCA
d) ASA
e) AICA
The answer is MCA
91) After having identified a fetus at risk in prolonged pregnancy, management should consist of:
a) Amniocentesisformaturitystudies.
b) Delivery regardless of the status of the cervix.
c) Fetal sampling of scalp pH.
d) Measurement of human chorionic somato- mammotropin.
e) Repeat antepartum studies in 1 week.
The answer is Delivery regardless of the status of the cervix.
92) Macrosomic infants of diabetic mothers are more prone to:
a) Birth trauma
b) Hypertrophic cardiomyopathy
c) Neonatalhypoglycaemia
d) Vascular thrombosis
e) All of the above
The answer is All of the above
93) Below the following percentile of birth weight, small-for-gestational age neonate is determined:
a) 5 th
b) 10 th
c) 15 th
d) 20 th
e) None of the above
The answer is 10 th
94) A 5-year-old girl, presents to the Emergency Department with abdominal pain and painful micturition. A
midstream urine sample is obtained and is positive for leucocytes. She was started on oral antibiotics and
discharged;

97
EMLE Trial Exams October 2020 (1000 Questions) Version (1.0) 9/12/2020

48 hours later you receive the following result from the microscopy and culture: white blood cells >200/mm3; red
blood cells, many seen; organisms, none seen; red cell casts, none seen; culture, >100.000 coliforms. What is the
most probable diagnosis?
a) Glomerulonephritis
b) Normal result
c) Perineal contamination
d) Urinary tract infection
e) Vulvovaginitis
The answer is Urinary tract infection
95) A 70-year-old woman presents to accident and emergency with sudden onset pain and swelling in the right knee.
Her past medical history includes hypertension and hypercholesterolaemia. She is currently taking aspirin,
ramipril and simvastatin. On examination, she is apyrexial and the right knee is swollen. There is reduced range of
movement in the knee due to swelling and pain. X-ray of the right knee shows chondrocalcinosis. What is the
most likely diagnosis?
a) Gout
b) Pseudo-gout
c) Septic arthritis
d) Reactive arthritis
e) Osteoarthritis
The answer is Pseudo-gout
96) A 68-year-old woman presents to the colorectal outpatient clinic with pain and streaks of fresh blood on the
surface of stool and on the paper, following defaecation. On examination, she is found to have a hard irregular
area at the 12 o’clock position, below the dentate line. What is the most likely diagnosis here from the list of
options below?
a) Haemorrhoids
b) Fissure in ano
c) Anal carcinoma
d) Perianal skin tags
e) Rectal prolapse
The answer is Anal carcinoma
97) In trauma imaging, which of the following statement is false?
a) In a multiply injured patient, CT of head and spine should be the first line of imaging.
b) Focused assessment with sonography for trauma helps in detecting intraperitoneal fluid and cardiac tamponade.
c) CT should not be used when a patient is unstable.
d) U/S is useful for diagnosing occult pneumothorax.
e) CT is the main imaging method for intracranial, intra-abdominal and vertebral injuries.
The answer is In a multiply injured patient, CT of head and spine should be the first line of imaging.
98) A 33-year-old man is admitted following an assault in a street fight. During the fight, he was hit by a blunt
object
across the side of the head. On admission his Glasgow Coma Scale score is initially 13/15 but falls to 7/15 during
his evaluation. The decision is taken to perform a computed tomography head scan, which identifies a lensshaped
space-occupying lesion within the cranial vault. The diagnosis is
a) Extradural haematoma
b) Subdural haematoma
c) Subarachnoid haemorrhage
d) Cerebral contusion
e) Intracerebral haemorrhage
The answer is Extradural haematoma

98
EMLE Trial Exams October 2020 (1000 Questions) Version (1.0) 9/12/2020

99) An 17-year-old boy is brought into the emergency department having fallen down a flight of stairs outside his flat.
His cervical spine is immobilized by the attending paramedics. The patient is saying he does not have neck pain
and wants the collar removed. In which of the following circumstances is it appropriate to remove the collar?
a) He has been drinking alcohol
b) He has lost consciousness for only 5 minutes
c) He has no cervical spine tenderness
d) He is in shock with a splenic injury which requires a laparotomy
e) He has no peripheral neurological signs
The answer is He has no cervical spine tenderness

100) A 2 years old girl develops a rash on her legs. The next day she is brought to surgery, by which time the rash
has spread to the rest of the body . What is the most likely diagnosis?
a) erythema multiforme
b) erthyma chronica migrans
c) erythema nodosum
d) urticarial
e) dermatitis artefacta
The answer is erythema multiforme

99
EMLE Trial Exams October 2020 (1000 Questions) Version (1.0) 9/12/2020

Exam 6
1) A 23-year-old man with a history of depression is admitted to the inpatient psychiatry ward after his third attempt
at suicide with an intentional drug overdose. The patient is stabilized medically; however, he is put under 24-hour
monitoring by the nursing staff due to repeated attempts at self-harm. During a change of shift, there is a mistake
in communication and no one is assigned to the patient. The mistake is noticed 15 minutes into the new shift, and
a member of the nursing team is assigned to watch the patient. Fortunately, during that 15-minute period, the
patient made no attempt to harm him- self. Which of the following statements is correct about this event?
a) This is a sentinel event and should be reported to the medical board.
b) This is a sentinel event and should be reported to the hospital and family.
c) This is a near-miss and should be reported to the hospital.
d) This is a near-miss and should be reported to the patient and family.
e) This is a near-miss and no reporting is required since the patient was not harmed.
Answer is: This is a near-miss and should be reported to the hospital.
2) The INCORRECT statement regarding endometriosis:
a) Chocolate fluid represents inspissated old blood.
b) Clinical picture is not diagnostic.
c) Classification system of the endometriosis is not related to the type of adenxal adhesions.
d) The tumor marker CA 125 might be elevated.
e) Laparoscopic picture is helpful and biopsy is essential.
Answer is: Classification system of the endometriosis is not related to the type of adenxal adhesions.
3) Child 7 years old presents with purpuric eruptions on his legs, he complains of abdominal pain and some joint
discomfort, what is the condition he might have?
a) ITP
b) Henochen line purpura
c) Hemophilia
d) WVD
e) SLE
Answer is: Henochen line purpura
4) A 27-year-old man presents to the emergency department complaining of severe nasal pain and a blocked nose. On
examination, he has a bluish discoloured swelling over the bridge of the nose. His friend tells you he was assaulted
four days ago. What is the most likely cause of his symptoms?
a) Fracture of the orbit
b) Fracture of the cribriform plate
c) Intranasal foreign body
d) Maxillary sinusitis
e) Septal haematoma
Answer is: Septal haematoma
5) Normal infant can sit unsupported at age of
a) 4 - 5 months
b) 6 - 8 months
c) 9 - 10 months
d) 3 - 5 months
e) 3-7 months
Answer is: 6 - 8 months

100
EMLE Trial Exams October 2020 (1000 Questions) Version (1.0) 9/12/2020

6) Definite final diagnosis of cervical intraepithelial neoplasia Class III (CIN III) is made by:
a) Cervical smear
b) Colposcopy
c) Histopathology
d) Tumormarkers
e) Schiller iodine stain
Answer is: Histopathology
7) The treatment for a testicular torsion of 10 days duration is:
a) Orchidectomy
b) Exploration only
c) Fixing up of other testis
d) Belat fixation of testes
e) Hormonal of treatment
Answer is: Fixing up of other testis
8) Which of the following statements concerning hematuria in children is TRUE:
a) Urine in glomerular hematuria is usually red
b) Glomerular hematuria occurs at the beginning of urinary stream
c) UTI is the most common cause of hematuria
d) Lower urinary tract hematuria is usually accompanied by proteinuria
e) Renal biopsy should be done to all cases
Answer is: UTI is the most common cause of hematuria
9) A 12-year-old presents with sneezing, clear rhinorrhea and nasal itching. Physical examination reveals with boggy,
pale nasal edema with clear discharge. The most likely diagnosis is
a) Foreign body
b) Vasomotor rhinitis
c) Neutropenic rhinitis
d) Nasal mastocytosis
e) Allergic rhinitis
Answer is: Allergic rhinitis
10) Delayed bone age may occur in all of the following except:
a) Hypothyroidism
b) Genetic short stature
c) Isolated growth hormone deficiency
d) Psychological deprivation
e) Severe malnutrition
Answer is: Genetic short stature
11) When handling items which are soiled with bodily fluids, it’s important to wear gloves. Which of these would not
be suitable?
a) Latex
b) Neoprene
c) Polythene
d) All are suitable
e) None of them is suitable
Answer is: Polythene

101
EMLE Trial Exams October 2020 (1000 Questions) Version (1.0) 9/12/2020

12) 42-year-old obese woman presents with a 9-month history of burning epigastric pain which is worse at night or
after oily food. Over the last 2 weeks she has developed occasional dysphagia to solids. Endoscopy shows hiatus
hernia, inflammation and stricture at the lower third of esophagus. What is your diagnosis? (Debatable)
a) Bulbar palsy
b) Esophageal carcinoma
c) GERD
d) cardiac achalasia
e) esophageal candidiasis
Answer is: esophageal candidiasis
13) A 26-year-old woman, gravida 2, para 1, at 28 weeks' gestation comes to the physician for a prenatal visit. She feels
well. Pregnancy and delivery of her first child were uncomplicated. Her temperature is 37.2°C (99°F) and blood
pressure is 163/105 mm Hg. Her blood pressure 10 weeks ago was 128/84 mm Hg. At her last visit two weeks ago,
her blood pressure was 142/92 mm Hg. Pelvic examination shows a uterus consistent in size with a 28-week
gestation. A complete blood count and serum concentrations of electrolytes, creatinine, and hepatic transaminases
are within the reference range. A urinalysis is within normal limits. Which of the following is the most appropriate
next step in management?
a) Magnesium sulfate therapy
b) Lisinopril therapy
c) Complete bed rest
d) Low-dose aspirin therapy
e) Alpha methyl dopa therapy
Answer is: Alpha methyl dopa therapy
14) A 24-year-old patient came to the outpatient with a 6-week history of fever, bloody diarrhea and weight loss. On
examination, he is clinically anemic and has aphthous ulcer of the mouth and mild tenderness of the abdomen.
Sigmoidoscopy shows transmural inflammation and non-caseating granulomas of the colon and rectum. What is
the most likely diagnosis?
a) Ulcerative colitis
b) Chron’s disease
c) IBS
d) Diverticulosis
e) Infectious diarrhea
Answer is: Chron’s disease
15) An unsensitized Rh-negative woman in her second pregnancy is seen in her thirty-sixth week. She complains of
edema in her legs and some tingling in her left hand. The most appropriate action at this time is:
a) Analysis of the husband’s blood type
b) Intramuscular Rh (anti-D) immune globulin
c) Assessment for possible preeclampsia
d) Rh antibody titer
e) Amniocentesis
Answer is: Assessment for possible preeclampsia
16) Which of the following statements about malignant transformation in cells is false?
a) Cells become immortal.
b) Cells acquire angiogenic competence.
c) Cells increase apoptosis.
d) Cells resist signals that inhibit growth.
e) Cells evade detection/elimination.
Answer is: Cells increase apoptosis

102
EMLE Trial Exams October 2020 (1000 Questions) Version (1.0) 9/12/2020

17) Which of these patients should undergo prophylactic cholecystectomy for asymptomatic gallstones?
a) 55‐year‐old man with end‐stage renal disease on the kidney transplant waiting list
b) 25‐year‐old African American man with sickle cell anemia
c) 35‐year‐old woman with a complete cervical spinal cord injury
d) 50‐year‐old man with short bowel syndrome who is dependent on total parenteral nutrition
e) 85‐year‐old woman with moderately well‐controlled diabetes
Answer is: 25‐year‐old African American man with sickle cell anemia
18) In the metabolism of iron, the following statements are correct EXCEPT:
a) Absorption occurs mainly in the duodenum and upper jejunum.
b) Absorption is increased when stores are depleted.
c) Traces of copper are essential for hemopoiesis.
d) Transferrin is increased in pregnancy.
e) Ferric salts are more readily absorbed than ferrous salts.
Answer is: Ferric salts are more readily absorbed than ferrous salts.
19) A 45-year-old man is brought to the emergency department having fallen off a ladder. He complains of pain in the
right hip. He is normally fit and well. Hip X-rays show a displaced intracapsular fracture. Which of the following is
the best treatment option?
a) Conservative management
b) Dynamic hip screw
c) Hemiarthroplasty
d) Reduction and internal fixation
e) Total hip replacement
Answer is: Reduction and internal fixation
20) Carcinoid tumors most likely to produce carcinoid before syndrome metastasis originate from:
a) Appendix
b) Stomach
c) Bronchus and terminal ileum
d) Rectum
e) Liver
Answer is: Bronchus and terminal ileum
21) Five years old child presenting with lethargy, respiratory distress, wheezes, cold extremities, BP: 60/30, weak pulse
.He received an injection of an unknown drug just few minutes before developing these symptoms. What is the
MOST appropriate management?
a) ABC + IV steroids ± antihistamine
b) ABC + IM steroids ± antihistamine
c) ABC + Inhaled ß2 agonist
d) ABC + IM adrenaline
e) ABC + SC adrenaline
Answer is: ABC + IM adrenaline
22) A 55-year-old man presents with a 1-day history of severe perianal pain and a palpable anal lump. He has noticed
spots of bright red blood on the toilet paper in recent weeks. On examination you find the following: What is the
diagnosis? incomplete question from the source(Debatable)
a) Perianal hematoma
b) Hemorrhoids
c) Skin tags
d) Rectal prolapse
e) Anal carcinoma

103
EMLE Trial Exams October 2020 (1000 Questions) Version (1.0) 9/12/2020

Answer is: Haemorrhoids


23) 55-year-old male with history of Diabetes and hypertension presents to the emergency department with chest pain
that occurs with exertion and is relieved with rest and nitrates. However, chest pain is progressing in intensity and
frequency over the last month. Which is the most likely diagnosis?
a) Stable angina
b) Unstable angina
c) STEMI
d) NSTEMI
e) Pericarditis
Answer is: Unstable angina
24) A 43-year-old woman undergoes open cholecystectomy. Intraoperative cholangiogram revealed multiple stones in
the CBD. Exploration of the CBD was performed to extract gallstones. The CBD was drained with a #18 T-tube. After
10 days, a T-tube cholangiogram reveals a retained CBD stone. This should be treated by which of the following?
a) Laparotomy and CBD exploration
b) Subcutaneous heparinization
c) Antibiotic therapy for 6 months and then reevaluation
d) Extraction of the stone through the pathway created by the T-tube (after 6 weeks)
e) Ultrasound crushing of the CBD stone
Answer is: Extraction of the stone through the pathway created by the T-tube (after 6 weeks)
25) Which one of the following interventions is most likely to increase survival in patients with COPD?
a) Home nebulizer
b) Tiotropium inhaler
c) Long term steroid
d) Smoking cessation
e) Long-term O2 therapy
Answer is: Smoking cessation
26) A 23-year-old African man who underwent total thyroidectomy 3 years ago presents with an unsightly swelling over
the scar of the surgery. He complains of needle-like pains and itching over the swelling. What is the most likely
diagnosis?
a) Wound dehiscence
b) Abscess
c) Tracheomalacia
d) Injury to external branch of the superior laryngeal nerve.
e) Keloid scar
Answer is: Keloid scar
27) An 18-month-old girl in your practice has been evaluated for recurrent episodes of hemolytic anemia. Her
hematologist has diagnosed hereditary spherocytosis and recommended an elective splenectomy. Of the following,
the MOST appropriate treatment before splenectomy
a) Hepatitis A vaccine
b) Measles, mumps, rubella vaccine
c) Meningococcal conjugate vaccine
d) Penicillin prophylaxis for 2 months
e) Pneumococcal vaccine
Answer is: Pneumococcal vaccine

104
EMLE Trial Exams October 2020 (1000 Questions) Version (1.0) 9/12/2020

28) Husband have been trying to conceive for more than a year with no success. Conception by in vitro fertilization was
attempted once 3 months ago but was unsuccessful. Analysis of her husband's semen has shown normal sperm
counts and morphology. She has a 6-year-old daughter who was born at term after an uncomplicated pregnancy.
She has no history of severe illness and tries to stay healthy by going to the gym an hour per day. Her menses occur
at regular 28-day intervals and last 5 to 6 days; her last menstrual period started 2 days ago. Physical examination
shows no abnormalities. Which of the following is the most likely cause of this patient's inability to conceive?
a) Polycystic ovarian syndrome
b) Pelvic inflammatory disease
c) Asherman's syndrome
d) Diminished ovarian reserve
e) Hypogonadotropic hypogonadism
Answer is: Diminished ovarian reserve
29) The risk for development of ectopic pregnancy increases with
a) Clamydial salpingitis.
b) Combined oral contraceptive pills.
c) Familial mediranean fever.
d) LNG-intrauterine system.
e) Cu IUD 380-A.
Answer is: Clamydial salpingitis.
30) The lactational amenorrhoea method (LAM) has the followings advantages EXCEPT
a) Universally available to all breastfeeding women
b) Effectiveness is 98% in well selected women
c) Protection begins immediately after delivery
d) Provide health benefits to the infant
e) Duration of method is unlimited.
Answer is: Duration of method is unlimited.
31) The MOST COMMON congenital fetal malformation in diabetes mellitus is
a) Ventricular and atrial septal defect(CVS).
b) Anenchephaly.
c) Spina bifida.
d) Sacral agenesis.
e) None of the above
Answer is: Ventricular and atrial septal defect (CVS).
32) Which feature could be consistent with a diagnosis of Bell’s palsy?
a) Hypersalivation
b) Hyperacusis
c) Mastication problems
d) Anesthesia of forehead
e) Anesthesia at jaw
Answer is: Hyperacusis
33) A case of 18-years-old male with lower abdominal pain, vomiting, low grade fever. The following investigations may
be indicated for diagnosis except: (Debatable)
a) CBC.
b) Urine analysis
c) U/S
d) CT scan with contrast
e) Upper GIT endoscopy

105
EMLE Trial Exams October 2020 (1000 Questions) Version (1.0) 9/12/2020

Answer is: Upper GIT endoscopy


DEBATABLE
34) For a child 15 kg with severe dehydration how many cc fluids you will give in maintenance phase?
a) 1000 cc
b) 1250 cc
c) 1500 cc
d) 2000 cc
e) non of the above
Answer is: 1250 cc
35) A 21-year-old married woman comes to the physician because she had unprotected sexual intercourse the previous
day. Menses have occurred at regular 28-day intervals since menarche at the age of 13 years. Her last menstrual
period was 12 days ago. Physical examination shows no abnormalities. A urine pregnancy test is negative. She does
not wish to become pregnant until after college. Which of the following is the most appropriate step in
management? incomplete question from the source(Debatable)
a) Woman misses 3 or more pills
b) At any time missed one pill
c) The women are lactating
d) The dose of progestin's in the pills is low
e) The dose of estrogens is high
Answer is: Woman misses 3 or more pills
36) The INCORRECT statement regarding missed abortion is:
a) It means retained dead embryo/ fetus.
b) Ultrasound is helpful in its diagnosis.
c) The uterus is usually felt smaller than the duration of pregnancy.
d) There may be brownish vaginal discharge.
e) There are exaggerated pregnancy symptoms.
Answer is: There are exaggerated pregnancy symptoms.
37) The MOST SPECIFIC fetal malformation to diabetes mellitus is: (Debatable)
a) Ventricular septal defect.
b) Coarctation of the aorta.
c) Spina bifida
d) Sacral agenesis
e) non of the above
Answer is: Sacral agenesis
38) A 29-year-old man is brought into the resuscitation room having been hit by a car. The patient is stable and his only
injury is a fractured right tibia with overlying tissue loss of around 5 cm. His leg is currently in a splint. His distal
pulses are palpable. What would be the most appropriate next step in the general and local management?
a) Dress wound
b) External fixation of fracture
c) Internal fixation of fracture
d) Intravenous antibiotics and debridement of tissue
e) Plaster immobilization of fracture
Answer is: Intravenous antibiotics and debridement of tissue
The debate about the model answer (D) and dress wound (A)
39) The most likely finding in the initial stages of septic shock is:
a) Abnormally low blood pressure.
b) Absent femoral pulses.
106
EMLE Trial Exams October 2020 (1000 Questions) Version (1.0) 9/12/2020

c) Delayed capillary refill.


d) Hyperpnea.
e) Tachypnea.
Answer is: Delayed capillary refill.
DEBATABLE
40) A 41-year-old woman teacher with angular stomatitis has a recent history of dysphagia to solids and fingers with
tight skin and trophic nail changes, which go pale then blue on exposure to cold. choose the single most likely
diagnosis
a) Cellulitis
b) Deep vein thrombosis
c) Lipodermatosclerosis
d) Lymphoedema
e) Scleroderma
Answer is: Scleroderma
41) A 61-year-old nulliparous woman comes to the physician for a follow-up examination. Her last Pap smear 3 years
ago showed atypical squamous cells of undetermined significance. HPV testing was negative at that time. On
questioning, she has had fatigue and an increase in abdominal girth despite a 5-kg (11.0-lb) weight loss over the
past 6 months. She has gastro esophageal reflux disease and Hashimoto's thyroiditis. Menarche was at the age of
10 years and her last menstrual period was 2 years ago. Current medications include omeprazole and levothyroxine.
Abdominal examination shows shifting dullness. There is tenderness to palpation of the right lower quadrant but
no guarding or rebound. Bimanual palpation shows a small uterus and a right adnexal mass. Further evaluation of
this patient is most likely to show which of the following findings?
a) Proliferation of endometrial glands
b) Elevated serum CA-125 level
c) Elevated serum beta-hCG level
d) Prolonged prothrombin time
e) Chocolate cyst of the right ovary
Answer is: Elevated serum CA-125 level
42) Pervaginal bloody discharge and hemoptysis with recent complete history of abortion raises the suspicion of:
a) Hydatiform mole.
b) Choriocarcinoma.
c) Krukenberg tumor.
d) Endometrialcancer.
e) Incomplete abortion.
Answer is: Choriocarcinoma.
43) Which of the following histological changes is not usually seen in Crohn's disease?
a) Granulomas
b) Crypt abscesses
c) Goblet cells
d) Continuous inflammation
e) Trans-mural inflammation
Answer is: Continuous inflammation
44) A 75-year-old woman with 48h of vomiting, abdominal distension, and a new swelling in the left groin which
appears to arise below the inguinal ligament. choose the single most likely abdominal mass choose the single most
likely abdominal mass
a) Femoral hernia
b) Inguinal hernia
107
EMLE Trial Exams October 2020 (1000 Questions) Version (1.0) 9/12/2020

c) Lymphoma
d) Palpable bladder
e) Pelvic abscess
Answer is: Femoral hernia
45) A child 3 years old open his eye to pain only what Glasgow coma scale score you will give for that?
a) 2
b) 3
c) 4
d) 5
e) 6
Answer is: 2
46) What is the most specific and sensitive investigation of TIA?
a) CT brain
b) MRI brain
c) Cranial US
d) Duplex US on neck
e) Diagnostic angiography
Answer is: MRI brain
47) A 3-day-old infant has his routine newborn check. You notice blue and black macules on his back and buttocks. He
was born by normal vaginal delivery and has been feeding well since birth. From the following list pick the diagnosis
that fits with the clinical description and picture.
a) bruising
b) Capillary hemangioma
c) Erythema toxicum
d) Mongolian blue spots
e) Port wine stain
Answer is: Mongolian blue spots
48) 24-year- old G 1 P 1 is being discharged from the hospital after an
uncomplicated vaginal delivery. She is breastfeeding and has no comorbidities. She would like a progesterone-only
birth control pill for contraception. Upon hospital discharge, which of the following statements should be included
in your counseling?
a) Do not start taking the pill until 3 months postpartum
b) Use condoms during the first 6 weeks of taking the pill until hormone levels reach full efficacy
c) Take the pill at the same time every day
d) Be aware that the pill can decrease the milk supply she produces
e) Send the patient home on bed rest
Answer is: Take the pill at the same time every day
49) An 18 month-old boy who was previously well, developed fever and irritability for 3 days. He was not coryzal.
Mother tells that she kept giving him antipyretics and the fever subsided spontaneously on the fourth day followed
by the appearance of this widespread maculopapular rash. What is the most probable diagnosis?
a) Erythema infectiosum
b) Roseola infantum
c) Rubella virus
d) Measles virus
e) Infectious mononucleosis
Answer is: Roseola infantum

108
EMLE Trial Exams October 2020 (1000 Questions) Version (1.0) 9/12/2020

50) Which statement regarding esophageal anatomy is false?


a) There are three predictable areas of narrowing: the cricopharyngeus muscle, the aortic arch, and the diaphragm
b) The cervical and most distal esophagus lie to the left of midline
c) The left gastric vein provides the principle venous drainage when esophageal varices develop
d) The segmental blood supply to the esophagus arises from the superior thyroid, the intercostals and the left gastric
arteries
e) The lower esophageal sphincter is a physiologic rather than an anatomic entity
Answer is: The segmental blood supply to the esophagus arises from the superior thyroid, the intercostal and the left
gastric arteries
51) A 25-year-old female presents with episodes of bizarre behaviour, memory lapse, and unconsciousness. She also
demonstrated previously episodes of extreme hunger, sweating, and tachycardia. During one of these episodes,
her blood sugar was tested and was found to be 40 mg/dL. Which of the following would most appropriately
indicate a diagnosis of insulinoma?
a) Demonstration of insulin antibodies in blood
b) Abnormal glucagon level
c) CT of the pancreas showing a mass
d) Hypoglycemia during a symptomatic episode with relief of symptoms by intravenous glucose
e) Decreased circulating C peptide in the blood
Answer is: Hypoglycemia during a symptomatic episode with relief of symptoms by intravenous glucose
52) A 2-year-old girl appears in clinic for a routine check-up. She does not like to eat at all but she loves potato chips.
The best measure to check her nutritional status:
a) Head circumferance
b) Length
c) Weight
d) Weight for height
e) Skin fold thickness
Answer is: Weight for height
53) The INCORRECT statement regarding endometriosis
a) It may be asymptomatic
b) It may be responsible for chronic acyclic pelvic pain
c) The only presenting symptom might be infertility
d) It might be associated with luteal insufficiency
e) The process of ovulation is not disturbed in patients with endometriosis
Answer is: The process of ovulation is not disturbed in patients with endometriosis
54) A 33-year-old female presents 6 weeks after the birth of her first child with a two-week history of polyarthralgia,
fever and a skin rash. First-line investigations show: A 33-year-old female presents 6 weeks after the birth of her
first child with a two-week history of polyarthralgia, fever and a skin rash. First-line investigations show: ESR
High.What is the most specific antibody of limited scleroderma?
a) Anticentromere antibody
b) Anti-topoisomerase antibody
c) antiJO1 antibody
d) ANA
e) Anti CCP antibody
Answer is: Anticentromere antibody

109
EMLE Trial Exams October 2020 (1000 Questions) Version (1.0) 9/12/2020

55) A child 6 years old found only responsive to pain you should?
a) Urgent CT
b) Urgent MRI
c) Intubate
d) Intubate and urgent CT
e) Neurological assessment
Answer is: Intubate and urgent CT
56) risk factors for atherosclerotic disease. What is the best to do to differentiate between acute and chronic ischemia
clinically:
a) History of 40 pack-years of smoking
b) History of intermittent claudication
c) History of thrombotic stroke
d) Presence of femoro-popliteal bypass scar in the left leg
e) Presence of foot pulses in the right leg
Answer is: Presence of foot pulses in the right leg
57) A 34-year-old man presents to his GP with a lump in his right testicle. This has been present for 6 months but has
started to increase in size causing him some discomfort. He has no lower urinary tract symptoms and is otherwise
well with no past medical history of note. On clinical examination of his external genitalia, there is a lump present
in his right hemiscrotum. It is 6 × 6 cm in size and is fluctuant but not tender. The testis is separate from the lump
and feels normal. You are able to get above the lump and it is not reducible nor does it have a cough impulse. On
attempting to shine a light behind the lump, it is brilliantly transilluminable. From the following list, select the most
likely diagnosis suggested by the history and clinical findings in this vignette.
a) Testicular tumour
b) Hydrocoele
c) Epididymal cyst
d) Inguinoscrotal hernia
e) Hydatid of Morgagni
Answer is: Epididymal cyst
58) A 35–year-old banker presents with intermittent lower abdominal pain, gaseous distension with a tendency toward
diarrhea whenever he is “stressed out”. He is a heavy cigarette smoker. Physical examination shows only diffuse
abdominal distension. The most important criteria of abdominal pain of this disease is:
a) Associated with a diarrhea
b) Association with rectal bleeding
c) Relieved with defecation
d) Relieved with urination
e) Relieved with vomiting
Answer is: Relieved with defecation
59) A 2 3-year-old.woman who is 10 weeks pregnant with a 3-week history of hyperemesis gravidarum is brought to
the emergency room after she collapsed and started fiting.What is the best drug to treat absence epilepsy?
a) Lamotrigine
b) Haloperidol
c) Midazolam
d) Ethusiximide
e) Phenytoin
Answer is: Ethusiximide

110
EMLE Trial Exams October 2020 (1000 Questions) Version (1.0) 9/12/2020

60) When a patient is admitted with acute stroke, which out of the following assessments should be completed first?
a) Abdominal ultrasound
b) Carotid Doppler
c) Cognitive function
d) Echocardiography
e) Swallow safety
Answer is: Swallow safety
61) A 34‐year‐old man was involved in a motorcycle accident. On arrival, his initial systolic blood pressure (SBP) was
80/50 mm Hg but subsequently improved to 120/65 mm Hg after 2 L of normal saline bolus. His Glasgow coma scale
(GCS) was 13. He was taken to the computerized scan (CT). His head CT showed depressed skull fracture with a
subdural hematoma. While being transported back to the trauma bay, he again became hypotensive with SBP 85/50
mm Hg. What would be the most appropriate treatment for this patient?
a) Administer 2 L of normal saline and continue to observe
b) Administer 2 units of O‐ and continue to observe
c) Arterial embolization by interventional radiologist
d) Administer 2 units of O‐ and take the patient directly to surgery
e) Type and cross and wait for radiologist report
Answer is: Administer 2 units of O‐ and take the patient directly to surgery
62) Prioritizing young healthy patients over old frail patients in COVID19 epidemics regarding mechanical ventilation
spaces is a demonstration of which of the following?
a) Autonomy
b) Confidentiality
c) Justice
d) Equity
e) Beneficience
Answer is: Equity
63) What of the following is an absolute contraindication to thrombolytic in stroke?
a) Active pancreatitis
b) BP >200/120 mmHg
c) Hemorrhagic diathesis
d) Major surgery in preceding 2 weeks
e) Pregnancy
Answer is: BP >200/120 mmHg
64) A 36-year-old woman comes to the physician to discuss contraceptive options. She is currently sexually active with
her husband, and they have not been using any contraception. She has no significant past medical history and takes
no medications. She has smoked one pack of cigarettes daily for 15 years. She is allergic to latex and copper. A urine
pregnancy test is negative. Which of the following contraceptive methods is absolutely contraindicated in this
patient?
a) Diaphragm with spermicide
b) Progestin-only pill
c) Intrauterine device
d) Condoms
e) Combined oral contraceptive pill
Answer is: Combined oral contraceptive pill
65) What is the current evidence base approach to the management of Bell’s palsy?
a) Carbamazepine
b) Prednisolone

111
EMLE Trial Exams October 2020 (1000 Questions) Version (1.0) 9/12/2020

c) Acyclovir
d) Propranolol
e) Aspirin
Answer is: Prednisolone
66) What is the best drug to treat absence epilepsy?
a) Lamotrigine
b) Haloperidol
c) Midazolam
d) Ethusiximide
e) Phenytoin
Answer is: Ethusiximide
67) Reed-stenberg cells are characteristic of(Debatable)
a) Hodgkin's lymphoma
b) Non – Hodgkin's lymphoma
c) Burkitt's lymphoma
d) B-cell lymphoma
e) T-cell lymphoma
Answer is: Hodgkin's lymphoma
68) The following pregnant women are considered potentially diabetic EXCEPT
a) Past history of macrosomia
b) Past history of eclampsia
c) Past history of congenital malformations
d) Past history of unexplained stillbirths.
e) Positive family history of overt diabetes mellitus.
Answer is: Past history of macrosomia
I think the correct answer is b but the model answer is a
69) A 30-year-old woman, gravida 2, para 1, at 40 weeks' gestation is admitted to the hospital in active labor. Pregnancy
has been complicated by iron deficiency anemia, which was treated with iron supplements. Her first pregnancy and
vaginal delivery were uncomplicated. There is no personal or family history of serious illness. Her pulse is 90/min,
respirations are 15/min, and blood pressure is 130/80 mm Hg. The abdomen is nontender and contractions are felt.
Ultrasonography shows that the fetal long axis is at a right angle compared to the long axis of the maternal uterus.
The fetal heart rate is 140/min and is reactive with no decelerations. Which of the following is the most appropriate
next step in the management of this patient?
a) Administration of oxytocin and normal vaginal birth
b) Vacuum-assisted delivery
c) Lateral positioning of the mother
d) Assisted vaginal breech delivery
e) Cesarean section
Answer is: Cesarean section
70) A 30-year-old type 1 diabetic man is brought to the emergency department by his boyfriend. The boyfriend says
that the patient has been acting confused and drowsy after recently 'coming down with the flu'. A simple blood test
reveals marked hyperglycemia. A urinalysis shows the presence of ketones ++ .What is the next step in
management?
a) IV insulin
b) SC insulin
c) Buccal glucose gel
d) Potassium supplement

112
EMLE Trial Exams October 2020 (1000 Questions) Version (1.0) 9/12/2020

e) IV fluids
Answer is: IV fluids
71) A Culdocentesis is performed in a 19-year-old G1 P0 woman with lower abdominal pain and vaginal spotting. A total
of 3 cc of clotted blood is aspirated. Which one of the following is the best interpretation?
a) A hemoperitoneum is present.
b) No hemoperitoneum is present.
c) The blood probably came from a blood vessel.
d) The patient probably has an ectopic pregnancy.
e) None og the above
Answer is: The patient probably has an ectopic pregnancy.
72) A 19-year-old woman presents with a 6-hour history of sudden onset right iliac fossa pain with low back pain. She
is obese and hirsute, and admits to irregular periods. She is currently taking metformin. What is the most likely
diagnosis? (Debatable)
a) Ruptured ovarian cyst
b) Crohn’s disease
c) Acute appendicitis
d) Ureteric colic
e) Caecal adenocarcinoma
Answer is: Ruptured ovarian cyst
73) How many patients undergoing cholecystectomy are found to have a gallbladder carcinoma?
a) 0.001
b) 0.01
c) 0.1
d) 0.25
e) 0.35
Answer is: 0.01
74) In bronchial asthma , symptoms are all of the following except:
a) Wheezing
b) Increased eosinophils in blood
c) Paroxysmal nocturnal attacks
d) Gross hemoptysis
e) Dry cough
Answer is: Gross hemoptysis
75) A 28‐year‐old man presents to the emergency department with a complaint of severe anal pain of 2 days duration.
Surgery is consulted with a referring diagnosis of “hemorrhoids.” The patient is very uncomfortable and cannot sit.
He relates no prior episodes or prior history of anorectal problems. His past medical history reveals a childhood
appendectomy and a history of asthma that is infrequent. On examination there is a small round bluish mass, about
8 mm in diameter in a left‐lateral position at the anal verge. This is very tender on palpation. The remainder of the
examination is unremarkable. What is the best management of this condition?
a) Excision under local anesthesia
b) Observation, Sitz baths, oral analgesics
c) Hemorrhoid banding
d) Examination under general anesthesia, formal high ligation and hemorrhoid excision.
e) Incision and expression of hematoma.
Answer is: Excision under local anesthesia

113
EMLE Trial Exams October 2020 (1000 Questions) Version (1.0) 9/12/2020

76) A 48-year-old man presents to the Emergency Department with full spinal immobilization following a fall from a
ladder. He denies any loss of conscious and maintains has a Glasgow Coma Score of 15/15 but complains of midline
neck tenderness. Chest and pelvic X-rays are normal. The C6 vertebrae is the last vertebrae visible on lateral C-spine
X-ray. What is the most appropriate management option?
a) C-spine X-ray
b) Chest drain
c) Laparotomy
d) CT head
e) CT neck
Answer is: CT neck
77) Side effects of corticosteroids include:
a) Hypotension
b) Diabetes insipidus
c) Osteopetrosis
d) Stunted growth
e) Mental retardation
Answer is: Stunted growth
78) A 24-year-old woman complains of bothersome hirsutism and skipping periods, she does not have evidence of voice
changes, hair loss, or clitoromegaly. The pelvic examination does not reveal adnexal masses. The serum DHEA-S,
testosterone, and 17-hydroxyprogesterone levels are normal. The LH to FSH ratio is 2:1. Which of the following is
the MOST LIKELY diagnosis? (Debatable)
a) Polycystic ovarian syndrome
b) Familialhirsutism
c) Ovarian tumor
d) Adrenaltumor
e) Cushing’s syndrome
Answer is: Polycystic ovarian syndrome
79) Which factor is most associated with an increased risk of going on to have a stroke in a patient presented with TIA?
a) Old age
b) Blood pressure > 140/90
c) Unilateral weakness
d) >60 minutes duration
e) Diabetes
Answer is: >60 minutes duration
80) A 50-year-old man presents with a right-sided thyroid swelling and episodes of diarrhoea and flushing. His past
medical history includes a bilateral adrenalectomy for phaeochromocytoma. You suspect an underlying thyroid
cancer. What is the most likely diagnosis?
a) Papillary
b) Anaplastic
c) Follicular
d) Medullary
e) Thyroid lymphoma
Answer is: Medullary

114
EMLE Trial Exams October 2020 (1000 Questions) Version (1.0) 9/12/2020

81) A 3-day-old infant, has his routine newborn check. you notice blue and black macules on his back and buttocks. He
was born by normal vaginal delivery and has been feeding well since birth. From the following list pick the diagnosis
that fits with the clinical description and picture.
a) Bruising
b) Capillary hemangioma
c) Erythema toxicum
d) Mongolian blue spots
e) Port wine stain
Answer is: Mongolian blue spots
82) The following about Candidal vaginal infection is INCORRECT:
a) The infection rate rises with pregnancy
b) Vaginal pH is usually alkaline
c) Vulval itching may occur
d) Vaginal isoconazole or miconazole are effective
e) Diagnosis is usually made without need for culture
Answer is: Vaginal pH is usually alkaline
83) Management of asymptomatic bacteruria includes:
a) Expectant management.
b) Induction of labor
c) Antibiotics.
d) Diuretics
e) Intravenous hydration
Answer is: Antibiotics.
84) A 37-year-old woman with right lower extremity edema is evaluated because of the sudden onset of shortness of
breath and pleuritic chest pain. A diagnosis of pulmonary embolism is made. Which of the following signs, if present
on physical examination, would be the most specific indicator of pulmonary arterial hypertension in this patient?
a) Increased jugular venous pressure
b) P2 louder than A2
c) Peripheral edema
d) Presence of an S3
e) Pulmonary crackles
Answer is: P2 louder than A2
85) You find in congenital hypothyroidism all except:
a) Constipation
b) Rough skin
c) Hypotonia
d) Tachycardia
e) Bradypna
Answer is: Tachycardia
86) A 10-month-old boy presents to the Emergency Department with a 2-day history of fever and runny nose. During
the night he gradually developed a barking cough in association with a loud noise on inspiration. His temperature
is 38° C and has marked sternal recession. Select the most likely diagnosis from the list below.
a) Acute epiglottitis
b) Anaphylaxis
c) Bronchiolitis
d) Laryngeal foreign body
e) Laryngotracheobronchitis (croup)

115
EMLE Trial Exams October 2020 (1000 Questions) Version (1.0) 9/12/2020

Answer is: Laryngotracheobronchitis (croup)


87) A 70-year-old woman presents to accident and emergency with sudden onset pain and swelling in the right knee.
Her past medical history includes hypertension and hypercholesterolaemia. She is currently taking aspirin, ramipril
and simvastatin. On examination, she is apyrexial and the right knee is swollen. There is reduced range of
movement in the knee due to swelling and pain. X-ray of the right knee shows chondrocalcinosis. What is the most
likely diagnosis?
a) Gout
b) Pseudo-gout
c) Septic arthritis
d) Reactive arthritis
e) Osteoarthritis
Answer is: P seudo-gout
88) Diagnosis of ectopic pregnancy can be ruled out in almost all clinically significant cases if
a) Transvaginal sonography showed no abnormalities in the adnexal region.
b) Patient is using the IUCD for contraception.
c) Serumb-hCG was negative.
d) There is no pelvic fluid by transvaginal uterine ultrasound.
e) Patient did not miss her period.
Answer is: Serumb-hCG was negative.
89) In post-streptococcal glomerulonephritis there is:
a) History of pancreatitis
b) Low ASO titer
c) High C3 level
d) HTN
e) Hypotension
Answer is: HTN
90) You review a 24-year-old woman with a history of asthma in the Emergency Department. She has been admitted
with acute shortness of breath associated with tongue tingling and an urticarial rash after eating a meal containing
shellfish. Her symptoms settle with nebulized salbutamol and intravenous hydrocortisone . What is the diagnostic
test of asthma severity?
a) Arterial blood gases
b) Spirometry
c) Serum IgE
d) Serum tryptase
e) Eosinophil count
Answer is: Spirometry
91) Right sided sensory loss affecting arms more than the legs together with right sided homonymous hemianopia.
What area is the stroke most likely to be affected?
a) ACA
b) MCA
c) PCA
d) ASA
e) AICA
Answer is: MCA

116
EMLE Trial Exams October 2020 (1000 Questions) Version (1.0) 9/12/2020

92) As part of a research project you are trying to ascertain whether the use of dummies in infants is linked to sudden
infant death syndrome. What is the most appropriate form of study design?
a) Cohort
b) RCT
c) Meta-analysis
d) Cross-sectional
e) Case-control
Answer is: Case-control

93) Which of the following is the MOST LIKELY mechanism of septic abortion following an induced abortion?
a) Instrumental contamination
b) Ascending infection
c) Skin organisms
d) Urinary tract penetration
e) Hematogenous infection
Answer is: Ascending infection
94) Which of the following is not a reason for breaking patient’s confidentiality?
a) Suicide
b) Homicide
c) TB
d) HIV
e) Influenza
Answer is: Influenza
95) Child 3 years with wet purpura and platelets 3000 what is the best action?
a) Packed RBC transfusion
b) Platelets transfusion
c) Plasma transfusion
d) Reassure
e) Observe
Answer is: Platelets transfusion
96) What of the following combination of diuretics is used in ascites secondary to liver cirrhosis?
a) Chlorthalidone and frusemide
b) Metolazone and frusemide
c) Bumetanide and eplerenone
d) Metolazone and spironolactone
e) Frusemide and metolazone
Answer is: Bumetanide and eplerenone
97) All of the following conditions are causes of anemia in the neonatal period except:
a) Hereditary spherocytosis
b) Twin to twin transfusion
c) Β-thalassemia
d) Vitamin E deficiency
e) Septicemia
Answer is: Β-thalassemia

117
EMLE Trial Exams October 2020 (1000 Questions) Version (1.0) 9/12/2020

98) Pattern of CIN lesions DO NOT include:


a) Regular surface contour
b) Stain brown with Lugol’s iodine
c) A marked acetowhite appearance
d) Coarseepithelialpunctations
e) Multisector involvement of the transformation zone
Answer is: Stain brown with Lugol’s iodine
99) A 38-year-old man is brought into the resuscitation room having been involved in a house fire. On arrival you
estimate him to have 20% burns. The man is around 80 kg in weight. How much fluid should the patient have over
the next 12 hours?
a) 300 mL
b) 600 mL
c) 1200 mL
d) 2400 mL
e) 3000 mL
Answer is: 2400 mL
100) A 41-year-old lawyer presents with a 3-month history of recurrent headaches which feel like a tight band around
the head. Which of the following is considered a dangerous sign of headache?
a) Morning headache and vomiting
b) Being unilateral
c) Being bilateral
d) Being in the morning
e) Associated with aura
Answer is: Morning headache and vomiting

118
EMLE Trial Exams October 2020 (1000 Questions) Version (1.0) 9/12/2020

Exam 7
1) A 38-year-old woman presented in the outpatient clinic by a lump in the left side of the neck associated with
weight loss, anorexia, malaise and profuse sweeting mainly at night. Examination showed cystic mildly tender
swelling along the anterior border of left stern mastoid muscle. Collar stud abscess was suspected, aspiration was
done & biopsy was sent to cytology that confirmed diagnosis. Which of the following chemotherapy regimens is
more suitable for this case?
a) Amoxicillin+ clavulanic acid + Ciprofloxacin
b) Gentamicin +Levofloxacin +Metronidazole
c) Rifampicin + Isoniazid +Pyrazinamide
d) Erythromycin +Tindazole
e) Ceftriaxone +Ampicillin
The answer is C
2) The following statements about Trichomonas vaginalis are correct EXCEPT:
a) Trichomonas vaginalis (TV) may be a sexually transmitted disease
b) Most patients, having TV, have symptoms
c) The disease may cause vulval itching
d) Unlike bacterial vaginosis, metronidazole is ineffective therapy
e) It may cause a strawberry appearance of the cervix
The answer is: Unlike bacterial vaginosis, metronidazole is ineffective therapy
3) Which of the following about blood transfusion are false?
a) A hemoglobin level of 6 g/dL or less is now considered a typical indication.
b) Fresh frozen plasma (FFP) is considered as the first-line therapy in coagulopathic hemorrhage.
c) Cryoprecipitate is useful in low fibrinogen states and in factor VIII deficiency.
d) Platelets have a shelf life of 3 weeks.
e) Patients can pre-donate blood up to 3 weeks before surgery for autologous transfusion
The answer is: Platelets have a shelf life of 3 weeks.
4) A 38-year-old man presents with a 3 × 3cm swelling in the right groin. It has been present for several months and
has slowly enlarged in size. On examination there is a soft non-tender swelling in the right groin crease, which has
a positive cough impulse. It does not reduce in the standing position, but disappears on lying supine. What is the
most likely diagnosis? (Debatable)
a) Inguinal lymphadenopathy
b) Femoral hernia
c) Inguinal hernia
d) Saphena varix
e) Femoral artery aneurysm
The answer is D
5) The incidence of PIH in diabetic pregnant women is: (Debatable)
a) 0.05
b) 0.25
c) 0.5
d) 0.75

119
EMLE Trial Exams October 2020 (1000 Questions) Version (1.0) 9/12/2020

e) None of the above


The answer is B
6) A primigravida, 36 weeks’ gestation is in labor with 3 cm dilated cervix and minimal uterine contraction. On rupture
of membranes, fresh bleeding is noted with late fetal deceleration reaching 50 beats/min. The patient was taken for
LSCS but fetus could not be saved. No abruptio or placenta previa was seen. The MOST LIKELY diagnosis is:
a) Placenta previa
b) Revealed abruptio placentae
c) Mixed type of abruptio placentae
d) Circumvallate placenta
e) Vasa previ
The answer is: vasa previa
7) A 23-year-old man is admitted following the sudden onset of an occipital headache. On examination GCS is 15/15,
neurological examination is unremarkable but neck stiffness is noted. A subarachnoid hemorrhage is suspected
but the CT scan is normal. What is the best drug to give in subarachnoid hemorrhage?
a) Propranolol
b) Nimodipine
c) Atenolol
d) Valproate
e) phenytoin
The answer is B
8) A 22-year-old pedestrian is brought to the Emergency Department following a road traffic collision with a car
travelling at 60 kmph. She was witnessed to have ‘bullseyed’ the windscreen of the car. On presentation she has a
primary survey which revealed a Glasgow Coma Score of 9. What is the most appropriate management option?
Trauma CT
Chest drain
Laparotomy
CT head
CT neck
The answer is: CT head
9) Regarding Meckel’s diverticulum all are true except:
a) Arises from the midgut .
b) May cause massive gastrointestinal bleeding.
c) Is a congenital anomaly.
d) Is associated with carcinoma of the small bowel.
e) May cause intussusceptions.
The answer is D

10) Failure of the heart to pump effectively causes the following type of shock?
a) Anaphylactic
b) Cardiogenic
c) Hypovolemic
d) Neurogenic
e) Septic
The answer is B
11) A 6 years old girl with offensive vaginal bleeding is brought by her mother, the best next step is:
a) Local Estrogen.
b) Examination under anesthesia to exclude retained foreign body.
120
EMLE Trial Exams October 2020 (1000 Questions) Version (1.0) 9/12/2020

c) Parenteral ampicillin for infection.


d) Metastatic work-up to exclude malignancy.
e) MRI
The answer is: Examination under anesthesia to exclude retained foreign body.
12) Which one of the following is most likely to be found in a patient with Hashimoto's thyroiditis? (Debatable)
a) Raised ESR
b) Anti-TSH receptor stimulating antibodies
c) Anti-thyroid peroxidase antibodies
d) Decreased TSH
e) Co-existing type 2 DM
The answer is B
13) Which one of the following statements is correct regarding the definition of neonatal deaths?
a) Neonatal deaths occur only up to the first 24 hours of life
b) Neonatal deaths occur only up to the first 72 hours of life
c) Neonatal deaths occur only up to the first 7 days of life
d) Neonatal deaths occur only up to the first 21 days of life
e) Neonatal deaths occur only up to the first 28 days of life
The answer is: Neonatal deaths occur only up to the first 28 days of life
14) A 25-year-old man is brought into the emergency department following an assault. On arrival he has multiple
bruises over the left side of his chest and upper abdomen. On examination, he has abdominal tenderness with
guarding throughout. His observations include a heart rate of 132/min and blood pressure 86/42 mmHg. A chest X-
ray shows a lower rib fracture and elevated diaphragm on the left. Which of the following is the most likely cause
of his symptoms?
a) Hepatic injury
b) Left-sided haemothorax
c) Left-sided pneumothorax
d) Ruptured diaphragm
e) Splenic injury
The answer is: Splenic injury
15) What of the following is an absolute contraindication to thrombolytic in stroke?
a) Active pancreatitis
b) Esophageal varices
c) Hemorrhagic diathesis
d) Major surgery in preceding 2 weeks
e) Pregnancy
The answer is Esophageal varices
MOHP site Answer: Esophageal varices
16) What is the staging of Hodgkin’s lymphoma presenting with lymphadenopathy and splenomegaly ?
a) Stage I
b) Stage IIA
c) Stage IIB
d) Stage III
e) Stage IV
The answer is D

121
EMLE Trial Exams October 2020 (1000 Questions) Version (1.0) 9/12/2020

17) A full-term baby boy, born 16 hours ago. His mother is blood group O rhesus positive. The baby is breastfeeding
well but he looks jaundiced. On examination the baby is clinically well. His bilirubin was 9 mg/dl at 10 hours, and six
hours later his bilirubin is 15.5 mg/dl. His blood group was identified as group A rhesus positive. Choose the most
likely cause of the baby’s jaundice:
a) ABO incompatibility
b) Physiologic jaundice
c) Congenital hypothyroidism
d) Congenital infection
e) Rhesus hemolytic disease
The answer is ABO incompatibility
18) A 33-year-old female is diagnosed with a personality disorder by her community psychiatrist. She has struggled
to hold down a job as an assistant store manager as she often finds her colleagues to be lacking in morals or values
and is reluctant to delegate work to them. She feels that her colleagues are lazy and do not perform their duties to a
sufficiently high standard, as a result, she is often overwhelmed with outstanding tasks that she cannot complete
and ends up staying late to get things right. What personality disorder is she most likely to have been diagnosed
with?
a) Schizoid
b) Narcissistic
c) Borderline
d) Obsessive – compulsive
e) Dependant
The answer is: Obsessive – compulsive
19) A 65-year-old female requires emergency surgery for a strangulated inguinal hernia. Which of the following is
correct?
The sac is formed by an unobliterated processus vaginalis.
The hernia is direct rather than indirect.
Such herniae never contain small intestine
Strangulation never results in bowel ischemia and gangrene requiring resection.
Indirect inguinal herniae are never found in female patients.
The answer is: The sac is formed by an unobliterated processus vaginalis.
20) An ultrasound examination of a 30-year-old woman reveals the following findings , all of which are suggestive of
erythroblastosis fetalis EXCEPT:
a) Bowing of fetal femur
b) Fetal ascites
c) Fetal cardiomegaly
d) Thickening of the placenta
e) Fetal head measurement that is large for gestational age
The answer is A
21) A 21-year-old married woman comes to the physician because she had unprotected sexual intercourse the
previous day. Menses have occurred at regular 28-day intervals since menarche at the age of 13 years. Her last
menstrual period was 12 days ago. Physical examination shows no abnormalities. A urine pregnancy test is
negative. She does not wish to become pregnant until after college. Which of the following is the most
appropriate step in management? (Debatable)
a) Woman misses 3 or more pills
b) At any time missed one pill
c) The women are lactating
d) The dose of progestins in the pills is low
e) The dose of estrogens is high

122
EMLE Trial Exams October 2020 (1000 Questions) Version (1.0) 9/12/2020

The answer is a
I think question asks about what choice increases chance of pregnancy not management.
22) Patients with long standing cough and expectoration seeks medical consultation and was diagnosed as a case of
bronchiectasis. In which circumstances are inhaled corticosteroids recommended?
a) In all bronchiectasis patients
b) In those who also have asthma
c) In those with a connective tissue disease
d) In those with co-existent emphysema
e) In those with rhino sinusitis
Answer is b
23) In acute glomerulonephritis, all are true except:
a) May be related to prior group A β-hemolytic streptococcal infection
b) Results in significant oliguria in the majority of cases
c) Is a recognized cause of headache and convulsions
d) Is an indication for long-term penicillin prophylaxis
e) Results in hypocomplementemia
Answer is d
24) A 29 -year-old G 1P001 0 woman with regular menses ( Every 28 days lasting 5 days) is currently trying to
conceive. She has not used Chapter 9: Gynecology 1 93 contraception for 6 months. Her ovulation predictor kit
revealed an luteinizing hormone (Debatable)
a) Cystic teratoma
b) Ectopic pregnancy
c) Follicular cyst
d) Hemorrhagic corpus luteum cyst
e) Serous cystadenoma
Answer is c
25) A 30-year-old primigravida presents at 34 weeks gestational age with blood pressure of 170/100 mmHg,
headache, epigastric pain, visual abnormalities and +3 proteinuria. Biophysical profile of the fetus is 8/8. Which
one of the following is the immediate response?
a) Start magnesium sulfate intravenously
b) Perform an emergency C-section
c) Give betaclomethasone to induce fetal lung maturity
d) Perform an amniocentesis to assess fetal lung maturity
e) Repeat the biophysical profile daily
Answer is a
26) A 24-year-old patient came to the outpatient with a 6-week history of fever, bloody diarrhea, and weight loss. On
examination, he is clinically anemic and has aphthous ulcer of the mouth and mild tenderness of the abdomen.
Sigmoidoscopy shows transmural inflammation and non-caseating granulomas of the colon and rectum. What is
the most likely diagnosis?

a) Ulcerative colitis
b) Chron’s disease
c) IBS
d) Diverticulosis
e) Infectious diarrhea
Answer is b

123
EMLE Trial Exams October 2020 (1000 Questions) Version (1.0) 9/12/2020

27) A 28-year-old P1 CS was diagnosed gestational hypertension at 28 weeks of gestation. She presents at 32 weeks
with pain in abdomen. O/E: P 115 bpm, BP 100/60 mmHg, and Hb 6 g/dl. Uterus is 32–34 weeks tonically
contracted with absent fetal heart sounds. P/V examination revealed no bleeding seen. The diagnosis is:
a) Ruptured uterus.
b) Internal bleeding from placenta previa
C) Concealed abruptio placentae
d) Revealed abruptio placentae
e) Mixed type of abruptio placentae
Answer is c
28) All of the following will occur in renal failure except:
a) Metabolic acidosis
b) Hyperkalemia
c) ↑ serum creatinine
d) Hypertension
e) Jaundice
Answer is e
29) 40-year-old G2P2002 presents to the emergency department (ED) with a 2 -hour history of severe left lower
quadrant (LLQ) pain. The pain began suddenly and has stayed constant since. On examination, her pulse is 85
beats/min, her BP is 1 45/80 mmHg, and her abdomen is diffusely tender without rebound or guarding. On
ultrasound, the left ovary is larger than the right, and blood flow is diminished. What is the next best step in the
management of this patient?
a) Emergent surgery
b) Left ovary biopsy.
c ) Obtain CA- 1 25 , a-fetoprotein, and [3 -hCG levels] .
d) CT of the abdomen and pelvis
e) Meconium aspiration
Answer is a
30) The patient’s son is asking you about his father’s diagnosis. Which is the most appropriate response?
a) Discuss with the son full details of the diagnosis
b) Tell him that you have to take consent firstly from the patient
c) Tell him about the diagnosis but not the full details
d) Tell him that you are not allowed to tell because he has a bad prognosis.
e) None of the above
Answer is b
31) Which one of the following is most likely to be found in a patient with thyrotoxicosis?
a) Raised ESR
b) Anti-TSH receptor stimulating antibodies
c) Anti-thyroid peroxidase antibodies
d) Decreased TSH
e) Co-existing type 2 DM
Answer is d
32) Detection of maternal antibodies in the fetal blood that had been attached to fetal cells is accomplished by:
a) Indirect Coomb’s test
b) Direct Coomb’s test
c) Radioimmunoassay
d) Rosette test
e) Enzyme linked antiglobulin test
Answer is test Direct Coomb’s test

124
EMLE Trial Exams October 2020 (1000 Questions) Version (1.0) 9/12/2020

33) A 13-year-old boy is referred to you, as his General Practitioner has incidentally noticed protein in his urine on
two occasions when the child came to him after school. The boy is healthy & has no signs of edema or recent
infection. He is the first child you see in your morning clinic & the urine dip you obtain is negative for protein.
a) Alport syndrome
b) Cystitis Nephritic syndrome
c) Nephrotic syndrome
d) Orthostatic proteinuria
Answer is e
34) The following are risk factors for development of placental abruption EXCEPT
a) Short umbilical cord
b) Folic acid deficiency
c) Pre-eclampsia
d) History of threatened abortion
e) Previous placental abruption
Answer is d
35) All of the following are risk factors for vaginal candidiasis, EXCEPT:
a) Diabetes mellitus
b) Pregnancy
c) HIV
d) Hypertension
e) None of the above
Answer is d
36) A 40-year-old man underwent laparoscopic cholecystectomy 2 years earlier. He remains asymptomatic until 1
week before admission, when he complains of RUQ pain and jaundice. He develops a fever and has several rigor
attacks on the day of admission. An ultrasound confirms the presence of gallstones in the distal CBD. The patient
is given antibiotics. Which of the following should be undertaken as the next step in therapy?
a) Should be discharged home under observation
b) Should be observed in the hospital
c) Undergo surgical exploration of the CBD
d) ERCP with sphincterotomy and stone removal
e) Anticoagulants
Answer is d
37) Investigators of a study are examining the reliability of a patient questionnaire. Both inter-rater reliability and
test-retest reliability were examined by the investigators. If the study is determined to have poor test-retest
reliability, which of the following can be concluded?
a) This study suffers from low accuracy
b) This study suffers from low precision
c) This study suffers from selection bias
d) This study's poor reliability is due to random error
e) This study's poor reliability is due to systematic error
Answer is b
38) Which of the following is not a problem associated with surgery in the jaundiced patient?
a) Clotting disorders.
b) Hepatorenal syndrome.
c) Infection.
d) Poor wound healing
e) Myocardial infarction.
Answer is e

125
EMLE Trial Exams October 2020 (1000 Questions) Version (1.0) 9/12/2020

39) A 40-year-old P4 with hypertrophic, clinically suspicious cervix & a negative smear, the BEST NEXT STEP is:
a) Previous electrodiathermy is of little relevance to the follow up
b) Post-coital bleeding is not a relevant symptom
c) The negative smear is reassuring
d) Outpatient punch biopsy is not a satisfactory technique
e) Cone biopsy is needed under general anesthesia
Answer is e
40) A child 8 years had head trauma and has an attack of seizure after that what is your recommendation?
a) Urgent CT
b) Intubate
c) Urgent MRI
d) Reassure U/S
Answer is a
41) An otherwise well 2-year-old presents with extreme pallor. Growth & development are normal. The child is pale
but there is no lymphadenopathy or splenomegaly. Full blood count shows hemoglobin 4g\dl (40 g\l). MCV 59.
Other indicies are normal. What is the most appropriate management option?
a) Bone marrow aspirate
b) Chelation therapy for lead
c) Dietary review
d) Small bowel technetium scan
e) Start gluten-free diet
The answer is C
42) The treatment of choice for an asymptomatic acquired duodenal diverticulum is:
a) Observation.
b) Endoscopic resection.
c) Surgical resection.
d) Surgical bypass.
e) pancreaticoduodenectomy
The answer is A
43) A 70 years old man is suspected to have developed hepatoceullar carcinoma. Elevated serum levels of which of
the following substances would be most likely to help confirm the diagnosis:
a) AFP
b) Alkaline phosphates
c) CEA
d) Beta human chorionic gonadotropin (HCG)
e) AST
The answer is A
44) What is the antibiotic regimen of choice for a patient on long‐standing proton pump inhibitor therapy with
suspected esophageal perforation from balloon dilation of a benign esophageal stricture? (DEBATABLE )
a) Cefazolin, piperacillin/tazobactam
b) Cefazolin, fluconazole
c) Vancomycin, piperacillin/tazobactam
d) Piperacillin/tazobactam, fluconazole
e) Vancomycin, piperacillin/tazobactam, fluconazole
The answer is Piperacillin/tazobactam, fluconazole

126
EMLE Trial Exams October 2020 (1000 Questions) Version (1.0) 9/12/2020

45) A 70-year-old man presents with a 6-hour history of inability to pass urine. He complained of bloodstained urine
‘rose wine’ appearance the night before. On examination there is a large, tender mass arising from the pelvis to
the umbilicus, and a blood clot at the external urethral meatus. What is the most appropriate next management
step?
a) 2-way Foley urethral catheter insertion
b) Urgent cystourethroscopy
c) 3-way Foley urethral catheter insertion and bladder irrigation
d) CT KUB (kidney, ureter, bladder)
e) Suprapubic catheter insertion
The answer is C
46) Which of the following statements is false for optimal perioperative care?
a) Volume loss should be promptly treated by large rapid intravenous (IV) infusions of fluid.
b) Hypothermia and pain are to be avoided.
c) Starvation needs to be combated.
d) Correct any comorbidities.
e) Helpful measures can be taken
The answer is A
47) A 36-year-old man presents to the emergency department with a severe left-sided headache with pain around the
left eye. He has had several similar episodes over the last 2 weeks, lasting 40-60 minutes each. The headaches are
associated with a runny nose. On examination, there is redness and tearing of his left eye. What is the first line
long-term preventative management of cluster headaches?
a) Verapamil
b) Carbamazepine
c) Lamotrigine
d) Phenytoin
e) Sumatriptan
The answer is A
48) Test not useful in case of tubal pregnancy is:
a) Pelvic examination
b) Ultrasonography
c) HCG levels
d) Hysterosalpingography (HSG)
e) Non of the above
The answer is D
49) A 55-year-old male with history of Diabetes and hypertension presents to the emergency department with chest
pain that started 1 hour ago and didn’t resolve with nitrates. ECG showed ST depression. Troponin is elevated.
Which is the most likely diagnosis?
a) Stable angina
b) Unstable angina
c) STEMI
d) NSTEMI
e) Pericarditis
The answer is D
50) Which of the following disease is characterized by abnormal movements?
a) Huntigton’s disease
b) B.Rheumatic chorea
c) C.Parkinson’s disease
d) B & C
e) All of the above
127
EMLE Trial Exams October 2020 (1000 Questions) Version (1.0) 9/12/2020

The answer is E
51) To measure obesity best indicator is:
a) Measure the body weight
b) Measure the weight for height
c) Calculate body mass index (BMI)
d) Determine growth velocity
e) Assists lipid profile
The answer is C
52) A 49-year-old man presents with a 4-week history of recurrent head¬ aches occurring at the same time every day,
shortly after going to sleep. It is usually associated with pain around the right eye and lacrimation. There is no
aura and each episode lasts less than an hour. The headache is relieved by movement .What is the most common
cause of the worst headache in patient’s life?
a) Giant cell arteritis
b) Subarachinoid hemorrhage
c) Tension headache
d) Cluster headache
e) Migraine
The answer is B
53) A49-year-old woman develops a tonic-clonic seizure while on a surgical ward 1 day after a
thyroidectomy. What is your diagnosis?
a) Tonic-clonic seizures
b) Hyponatremia
c) Absence seizures
d) Hypomagnesemia
e) Hypocalcemia
The answer is E
54) A 78-year-old woman presents with a 12-hour history of a unilateral acutely painful, pale, cold lower leg. On
examination, no popliteal or foot pulses are palpable, and she is unable to move the foot or toes. She has no
history of limb 55**ischaemia. An ECG shows atrial fibrillation. You diagnose an acute ischaemic limb. What is the
most appropriate management?
a) Angioplasty
b) Below knee amputation
c) Systemic thrombolysis
d) Embolectomy
e) Femoral-popliteal bypass
The answer is D
55) A 55-year-old male with history of Diabetes and hypertension presents to the emergency department with chest
pain that occurs with exertion and is relieved with rest and nitrates. However, chest pain is progressing in
intensity and frequency over the last month. Which is the most likely diagnosis?
a) Stable angina
b) Unstable angina
c) STEMI
d) NSTEMI
e) Pericarditis
The answer is b) Unstable angina

128
EMLE Trial Exams October 2020 (1000 Questions) Version (1.0) 9/12/2020

56) A 6-month-old previously healthy girl is brought to your office because she has not been eating well today. The
mother reports that the baby is interested in taking the bottle, but stops feeding within 1 minute and seems to
have trouble breathing. She is irritable but consolable in her mother’s arms. She is pale, afebrile, and has a
respiratory rate of 70 breaths/min. Her heart rate is too fast to count, she has palpable pulses in all extremities,
and her perfusion is fair, with a capillary refill time of 2 to 3 seconds. Of the following, the MOST likely additional
finding expected in this child is :
a) crackles over the lungs
b) conjunctivitis
c) hepatomegaly
d) nuchal rigidity
e) rash on the extremities hepatomegaly
The answer is C
57) A 16-year-old girl has been referred on to you by her gynaecologist. She has been complaining of amenorrhea for
5 months, although no gynaecological abnormality has been found. She feels well and is very active but her
weight has decreased from 61 kg to 43 kg in the last 6 months. her height is 168 cm. on examination her BP is
90/60 mmHg, heart rate 64 bpm. What is the most likely diagnosis?
a) Conn's syndrome
b) Crohn's disease
c) Anorexia nervosa
d) Hyperthyroidism
e) Diabetes mellitus
The answer is C
58) A 33-year-old lady develops a thunderclap headache and collapses. A CT scan shows that she has developed a
subarachnoid haemorrhage. She currently has no evidence of raised intracranial pressure. Which of the following
drugs should be administered?
a) Atenolol
b) Labetalol
c) Nimodipine
d) Amlodipine
e) Mannitol
The answer is C
59) A 27-year-old man presents to the emergency department complaining of severe nasal pain and a blocked nose.
On examination, he has a bluish discoloured swelling over the bridge of the nose. His friend tells you he was
assaulted four days ago. What is the most likely cause of his symptoms?
a) Fracture of the orbit
b) Fracture of the cribriform plate
c) Intranasal foreign body
d) Maxillary sinusitis
e) Septal hematoma
The answer is E
60) A 32-year-old man is stabbed in the left side of the chest. Chest x-ray shows a whiteout of the left lung field.
a) Haemothorax
b) diaphragmatic rupture
c) diaphragmatic contusions
d) myocardial contusion
e) pleural effusion
The answer is A

129
EMLE Trial Exams October 2020 (1000 Questions) Version (1.0) 9/12/2020

61) An 8-years-old boy brought to the ER after accidentally touching a hot iron with forearm. On examination, the
burned area has weeping blisters containing clear fluid and severely tender. What is the burn depth?
a) First degree.
b) Second degree.
c) Third degree.
d) Fourth degree.
e) Scald.
Answer is Second degree burns
62) Pregnancy DOES NOT have which of the following effects on diabetic women:
a) Tendency toward ketosis in the first trimester
b) Tendency toward hypoglycemia during early pregnancy
c) Increase in insulin requirement during early pregnancy
d) Increase in insulin requirement during late pregnancy
e) Women with preexisting but undetected overt DM, may be initially diagnosed during pregnancy.
Answer is increase insulin requirement during early pregnancy
63) Indications of therapeutic abortion DO NOT include:
a) Advanced hypertensive vascular disease.
b) Missed abortion.
c) Active pulmonary T.B.
d) Advanced chronic renal disease.
e) All cases of uncontrolled diabetes mellitus
The answer is all cases of uncontrolled diabetes mellitus
64) A 61-year-old nulliparous woman comes to the physician for a follow-up examination. Her last Pap smear 3 years
ago showed atypical squamous cells of undetermined significance. HPV testing was negative at that time. On
questioning, she has had fatigue and an increase in abdominal girth despite a 5-kg (11.0-lb) weight loss over the
past 6 months. She has gastroesophageal reflux disease and Hashimoto's thyroiditis. Menarche was at the age of
10 years and her last menstrual period was 2 years ago. Current medications include omeprazole and
levothyroxine. Abdominal examination shows shifting dullness. There is tenderness to palpation of the right lower
quadrant but no guarding or rebound. Bimanual palpation shows a small uterus and a right adnexal mass. Further
evaluation of this patient is most likely to show which of the following findings?
a) Proliferation of endometrial glands
b) Elevated serum CA-125 level
c) Elevated serum beta-hCG level
d) Prolonged prothrombin time
e) Chocolate cyst of the right ovary
The answeris elevated serum CA125 level
65) Child 2 years old his weight is 12 kg what is the maintenance fluid requirement for him if he is severely
dehydrated?
a) 550 ml
b) 750 ml
c) 1100 ml
d) 1500 ml
e) 2000 ml
The answer is 1100 ml
66) A 10-year-old boy with 1 week of vague abdominal pains, 24h of a swinging pyrexia, and a tender, firm mass in the
right iliac fossa. choose the single most likely abdominal mass
a) Appendix mass
b) Femoral hernia
c) Inguinal hernia
130
EMLE Trial Exams October 2020 (1000 Questions) Version (1.0) 9/12/2020

d) Lymphoma
e) Palpable bladder
The answer is appendix mass
67) The patient’s nurse places a peripheral intravenous catheter in a 15-kg, 4-year-old male and requests orders for
fluid type and rate. Which of the following is the most appropriate initial choice (both fluid and amount) for
volume resuscitation in this patient?
a) Packed red blood cells 150 mL
b) Lactated Ringers 30 ml
c) Normal saline (0.9% sodium chloride solution) 300 mL
d) ½ Normal saline (0.045% sodium chloride solution) 300 mL
e) 3% Normal saline 45 mL
The answer is normal saline (0.9% sodium chloride solution) 300 ml
68) A 19-year-old man is brought to the emergency department by ambulance with a stab-wound to the right upper
quadrant (RUQ) of the abdomen. A FAST scan shows free fluid, and the patient is taken to the operating room for
an exploratory laparotomy. The findings are a non bleeding laceration of the right lobe of the liver and a
gallbladder laceration. Which of the following is TRUE?
a) The gallbladder injury can be treated with cholecystectomy.
b) Isolated gallbladder injuries are uncommon.
c) Bile is usually sterile.
d) The liver laceration does not require closed suction drainage.
e) A thorough exploration is not necessary if the bleeding is confined to the RUQ
The answer: isolated gallbladder injuries are uncommon
69) A 37-year-old man presented to the Urology Outpatient department having been referred from the fertility
services. He has had multiple failed attempts to conceive a child with his partner. Of note, he experiences a
dragging sensation within his right testicle which often causes him discomfort. On examination of the scrotum,
the testicle and epididymis appeared normal. However, the cord structures were boggy and felt like a ‘bag of
worms.’ This did not transilluminate nor is it tender. There is an associated cough impulse. From the following list,
select the next most appropriate diagnosis associated with these findings.
a) Varicoceles
b) Epididymal cyst
c) Hydrocoele
d) Direct inguinal hernia
e) Indirect inguinal hernia
The answer is varicoceles
70) Which of the following is not included in Standard Infection Control Precautions?
a) Management of blood and bodily fluid spillage
b) Patient’s personal hygiene
c) Safe handling of linen
d) Cleanliness of care equipment
e) Sharps safety
The answer is patient’s personal hygeine
71) A 70-year-old woman presents with a 2-month history of anal pain and itching. More recently she had been
having some fresh bleeding and mucous discharge per rectum. On examination, there is an irregular tender
ulceration at the anal margin which appears to be extending into the anal canal. What is the most likely
diagnosis?
a) Anal carcinoma
b) Anal fissure
c) Anal warts
d) Fistula-in-ano
131
EMLE Trial Exams October 2020 (1000 Questions) Version (1.0) 9/12/2020

e) Primary syphilis
The answer is anal carcinoma
72) What is the prophylactic treatment of Meniere’s disease?
a) intramuscular prochlorperazine
b) buccal prochlorperazine
c) betahistidine
d) oral morphine
e) oral tramadol
The answer is betahistidine
73) Which of the following statements concerning hematuria in children is TRUE:
a) Urine in glomerular hematuria is usually red Glomerular hematuria
b) Occurs at the beginning of urinary stream
c) UTI is the most common cause of hematuria
d) Lower urinary tract hematuria is usually accompanied by proteinuria
e) Renal biopsy should be done to all cases
The answer is UTI is the most common cause of hematuria
74) Management of asymptomatic bacteruria includes:
a) Expectant management.
b) Induction of labor
c) Antibiotics.
d) Diuretics
e) Intravenous hydration
The answer is antibiotics
75) A 4-year-old boy presents with new-onset type 1 diabetes mellitus and diabetic ketoacidosis. He appears tired
but, on physical examination, shows only mild signs of dehydration. Initial laboratory tests reveal the following
results: • Serum glucose, 884 mg/dL • Serum potassium, 4.5 mEq/L • pH, 6.92 on arterial blood gas Of the
following, the MOST important first step in managing this patient is to administer:
a) An insulin drip, beginning at 0.1 units/kg/h
b) An intravenous insulin bolus of 0.1 units/kg
c) Lactated Ringer solution, 20 mL/kg over 15 minutes
d) Normal saline, 10 to 20 mL/kg over 1 to 2 hours
e) Sodium bicarbonate, 1 to 2 mmol/kg over 60 minutes
The answer is normal saline 10-20 ml/kg over 1-2 hours
76) A male patient is admitted following a fall from height. On arrival his Glasgow Coma Scale score is 4/15 and he is
therefore intubated. During primary resuscitation a chest x ray is taken which shows a widened mediastinum and
right-sided deviation of the trachea. The diagnosis is
a)Tension pneumothorax
b) Ruptured oesophagus
c) Cardiac tamponade
d) Right lobe collapse
e) Aortic rupture
The answer is aortic rupture
77) A 25-year old with recently diagnosed ulcerative colitis is started on mesalazine after a recent tapering of high
dose steroids. Two weeks later, he develops severe pain in his epigastrium which radiates through to his back.
What is the most likely diagnosis?
a) Hepatitis
b) Acute pancreatitis
c) Primary sclerosing cholangitis
d) Duodenal ulceration
132
EMLE Trial Exams October 2020 (1000 Questions) Version (1.0) 9/12/2020

e) Acute coronary syndrome


The answer is acute pancreatitis
78) During treatment of anaphylaxis include:
a) Bronchodilator
b) IM epinephrine
c) IV antihistamines
d) Systemic corticosteroids
e) Non of the above
The answer is IM epinephrine
79) A 30-year-old woman, gravida 2, para 1, at 40 weeks' gestation is admitted to the hospital in active labor.
Pregnancy has been complicated by iron deficiency anemia, which was treated with iron supplements. Her first
pregnancy and vaginal delivery were uncomplicated. There is no personal or family history of serious illness. Her
pulse is 90/min, respirations are 15/min, and blood pressure is 130/80 mm Hg. The abdomen is non tender and
contractions are felt. Ultrasonography shows that the fetal long axis is at a right angle compared to the long axis
of the maternal uterus. The fetal heart rate is 140/min and is reactive with no decelerations. Which of the
following is the most appropriate next step in the management of this patient?
a) Administration of oxytocin and normal vaginal birth
b) Vacuum-assisted delivery
c) Lateral positioning of the mother
d) Assisted vaginal breech delivery
e) Cesarean section
The answer is cesarean section
80) A 16-year-old girl has an enlarging mass on the right side of her neck for the last six weeks. She has no other
symptoms. She has a two x two cm lymph node in the right anterior triangle of the neck and several smaller
cervical lymph nodes. Her tonsils are enlarged and covered by a grey membrane. What is the most likely
diagnosis?
a) Infectious mononucleosis
b) Leukaemia
c) Lymphoma
d) Sarcoidosis
e) Tuberculosis
The answer: infectious mononucleosis
81) A 42-year-old woman presents with an upper midline mass that has been present for over a year. She has a
history of a partial gastrectomy for a perforated ulcer. On examination, the mass is 5 cm in size, soft, non-tender
and reducible. Which of the following is the most likely diagnosis?
a) Direct inguinal hernia
b) Epigastric hernia
c) Hiatus hernia
d) Incisional hernia
e) Paraumbilical hernia
Answer: Incisional hernia
82) A 24-year-old woman gravida 2, para 1 at 24 weeks' gestation comes to the physician for a prenatal visit. She
feels well. Her earlier pregnancy was uncomplicated. This is her 4th prenatal visit. She had an ultrasound scan 2
weeks ago that showed a live intrauterine pregnancy consistent with a 22-week gestation with no anomalies. She
had a normal Pap smear 2 years ago. Vital signs are within normal limits. Pelvic examination shows a uterus
consistent in size with a 24-week gestation. Her blood group and type is B positive. Which of the following is the
most appropriate next step in management?
a) Serum PAPP-A and HCG level
b) Oral glucose challenge test
133
EMLE Trial Exams October 2020 (1000 Questions) Version (1.0) 9/12/2020

c) Cardiotocography
d) Rh antibody testing
e) Swab for GBS culture
Answer is Oral glucose challenge test
83) The sings of acute respiratory distress include all except:
a) Tachypnea
b) Working ala nasi
c) Intercostal retraction
d) Suprasternal and subcostal recessions
e) Clubbing
Answer is Clubbing
84) What is the preferred treatment for an anaphylactic shock?
a) Epinephrine
b) Placing the patient in sitting position and administer oxygen
c) Preventing the reaction from occurring through patient teaching
d) Placing a bag of ice on the area, administer antihistaminic and corticosteroids
e) Antibiotic therapy
Answer is Epinephrine
85) An 8-year-old is accidentally hit in the abdomen by a football. After several minutes of discomfort, he seems to be
fine. Over the ensuing 24 hours, however, he develops a fever, abdominal pain radiating to the back, and
persistent vomiting. On examination, the child appears quite uncomfortable. The abdomen is tender, with
decreased bowel sounds throughout, but especially painful in the mid epigastric region with guarding. Which of
the following tests is most likely to confirm the diagnosis?
a) Serum amylase levels
b) CBC with differential and platelets
c) Serum total and direct bilirubin levels
d) Abdominal radiograph
e) Electrolyte panel
Answer is Serum amylase levels
86) A 22‐year‐old male presents to the trauma bay with a 6 cm laceration to his right forearm after punching through
a window. His neurovascular exam is within normal limits. The incident happened approximately 45 minutes ago.
You decide to give him prophylactic antibiotics. You want to examine the wound and close it in the trauma bay.
Your intern asks you the mechanism of action of local anesthetics. You tell him that:
a) The exact mechanism is unknown
b) Anesthetics work by reversibly binding calcium channels along the nerve
c) Anesthetics work by blocking acetylcholine release from the presynaptic cleft
d) Anesthetics work by reversibly binding sodium channels within the nerve fibers
e) Anesthetics work by competitively inhibiting potassium channels along the nerve
Answer is Anesthetics work by reversibly binding sodium channels within the nerve fibers
87) A 25-year-old woman at 33 weeks, gestation is noted to have a placenta previa. Upon cesarean section, bluish
tissue densely adherent between the uterus and maternal bladder is noted. The most likely added placental
abnormality is (Debatable)
a) Placenta accreta
b) Placenta increta
c) Placenta percreta
d) Placental polyp
e) Placental hemangioma
Answer is Placenta increta
Placenta increta corrected to Placenta percreta
134
EMLE Trial Exams October 2020 (1000 Questions) Version (1.0) 9/12/2020

88) A 23-year-old man with a history of depression is admitted to the inpatient psychiatry ward after his third
attempt at suicide with an intentional drug overdose. The patient is stabilized medically; however, he is put under
24-hour monitoring by the nursing staff due to repeated attempts at self-harm. During a change of shift, there is a
mistake in communication and no one is assigned to the patient. The mistake is noticed 15 minutes into the new
shift, and a member of the nursing team is assigned to watch the patient. Fortunately, during that 15-minute
period, the patient made no attempt to harm him- self. Which of the following statements is correct about this
event?
a) This is a sentinel event and should be reported to the medical board.
b) This is a sentinel event and should be reported to the hospital and family.
c) This is a near-miss and should be reported to the hospital
d) This is a near-miss and should be reported to the patient and family
e) This is a near-miss and no reporting is required since the patient was not harmed.
Answer is This is a near-miss and should be reported to the hospital
89) Serum Na level in hypotonic dehydration:
a) 130-140 mEq/L
b) <130 mEq/L
c) 170-180 mEq/L
d) >180 mEq/L
e) 150-160 mEq/L
Answer <130 mEq/L
90) A 48-year-old smoker, who was diagnosed with COPD 10 years ago, is experiencing shortness of breath and a
productive cough with purulent sputum. These episodes have become more frequent within the last few years.
What is the best management if the chest x-ray shows a pneumothorax with a 2.5 cm rim of air and no
mediastinal shift?
a) Aspiration
b) Discharge
c) Intercostal drain insertion
d) Repeat chest X-ray
e) Hospitalization
Answer is Intercostal drain insertion
91) A 55-year-old male with history of Diabetes and hypertension presents to the emergency department with chest
pain that started 3 days ago after he had runny nose and sneezing for five days. ECG showed ST elevation in all
leads. Which is the best initial treatment?
a) Morphine
b) Oxygen
c) Warfarin
d) Heparin
e) Ibuprofen
Answer is Ibuprofen
92) A patient has just delivered her first child after an uncomplicated pregnancy and term vaginal delivery. She is
anxious to breast-feed. As part of her postpartum discharge counseling, she should be told that few things
interfere with lactation, but she should avoid which of the following?
a) Depo-Provera
b) Frequent suckling
c) High dose (=50 µ gestradiol) oral contraceptive pills
d) Levonorgestrel intrauterine device (IUD)
e) Progestin-only oral contraceptive pill (minipill)
Answer is High dose (=50 µ gestradiol) oral contraceptive pills

135
EMLE Trial Exams October 2020 (1000 Questions) Version (1.0) 9/12/2020

93) A 48-year-old builder presents following a fall from a second floor. He is assessted with a primary survey, and
found to have a respiratory rate of 32 breaths/min, a deviated trachea, distended neck veins and right-sided
hyper resonance on percussion. What is the most appropriate management option?
a) Chest X-ray
b) Chest drain
c) Laparotomy
d) CT head
e) CT neck
Answer is Chest drain
94) Right sided sensory loss affecting arms more than the legs together with right sided homonymous hemianopia.
What area is the stroke most likely to be affected?
a) ACA
b) MCA
c) PCA
d) ASA
e) AICA
Answer is MCA
95) As regards Chlamydial infection, the following statements are correct EXCEPT
a) The organisms are obligatory intracellular organisms
b) It may cause sterile pyuria
c) Chlamydia and gonorrhea are the main causes of PID.
d) The husband should be treated
e) Azithromycin 2 g in single dose is an effective treatment
Answer is Azithromycin 2 g in single dose is an effective treatment
96) You review a 24-year-old woman with a history of asthma in the Emergency Department. She has been admitted
with acute shortness of breath associated with tongue tingling and an urticarial rash after eating a meal
containing shellfish. Her symptoms settle with nebulised salbutamol and intravenous hydrocortisone what is the
diagnostic test of asthma severity?
a) Arterial blood gases
b) Spirometry
c) Serum IgE
d) Serum tryptase
e) Eosinophil count
Answer is Spirometry
97) An 85-year-old male patient with a history of chronic constipation presents with acute severe colicky abdominal
pain and absolute constipation. Plain abdominal film shows a grossly dilated oval of large bowel arising from the
left lower quadrant. A diagnosis of sigmoid volvulus is made. The next step in management is:
a) Laparotomy
b) Sigmoidoscopy with flatus tube insertion
c) Sigmoid colectomy with colostomy
d) Barium swallow
e) Computed tomography
Answer is Sigmoidoscopy with flatus tube insertion
98) An 8-year-old girl presents with fever, neck stiffness and photophobia. She is confused, with spontaneous eye-
opening and can obey motor commands. Calculate the Glasgow Coma Score (GCS) of this patient
a) 10
b) 11
c) 12
d) 13
136
EMLE Trial Exams October 2020 (1000 Questions) Version (1.0) 9/12/2020

e) 14
Answer is 14
99) A patient refuses to undergo a necessary surgery. This is a demonstration of which of the following?
a) Autonomy
b) Confidentiality
c) Justice
d) Equity
e) Beneficence
Answer is Autonomy
100) You are called to the maternity ward to see a 20-year-old primiparous patient who delivered a healthy baby 2
hours ago after prolonged labour. Now she complains of vaginal bleeding, and the vagina is distended with blood
clots. Pelvic examination reveals lax abdomen and a uterus with 28 weeks fundal height. Her vitals are:
Temperature of 37°C, BP of 100/50 mm Hg, RR of 20/min and Pulse of 116/min. The likely diagnosis of her
condition is:
a) Atonic Postpartum hemorrhage
b) Rupture uterus.
c) Broad ligamentary hematoma.
d) Normal postpartum bleeding
e) Cervical tear
Answer is Atonic Postpartum hemorrhage

137
EMLE Trial Exams October 2020 (1000 Questions) Version (1.0) 9/12/2020

Exam 8
1) Which of the following are not potentially life-threatening injuries?
Aortic injuries
Tracheobronchial injuries
Myocardial contusion
Rupture of the diaphragm
Hollow viscus injury.
The answer is: Hollow viscus injury
2) Child 1 year old his weight is 10 kg what is the maintenance fluid requirement for him if he is severely dehydrated?
500 ml
700ml
1000 ml
1500 ml
2000 ml
The answer is: 1000ml
3) A 41-year-old lawyer presents with a 3-month history of recurrent headaches which feel like a tight band around
the head. Which of the following is considered a dangerous sign of headache?
Morning headache and vomiting
Being unilateral
Being bilateral
Being in the morning
Associated with aura
The answer is: Morning headache and vomiting
4) Clinical evidences of cervical cancer may include all of the following EXCEPT:
Perimenopausal bleeding
Uremia
Postmenopausal bleeding
Gastric upset
Severe pelvic pain
The answer is: Gastric upset
5) A false statement about rheumatic chorea is:
More in females
Emotional liability is common
There is hypotonia
ESR is always high
Long term prophylaxis against streptococcal infection is indicated
The answer is: ESR is always high
6) A painful ulcer, commonly found at the anal margin. What is the most likely surgical pathology?
Fissure
Hernia
Sinus
Seroma
Abscess
The answer is: Fissure
7) Which of the following is not a reason for breaking patient’s confidentiality?
Suicide
Homicide
138
EMLE Trial Exams October 2020 (1000 Questions) Version (1.0) 9/12/2020

TB
HIV
Influenza
The answer is: Influenza
8) What of the following is an absolute contraindication to thrombolytic in stroke? (Debatable)
Active pancreatitis
Resolving neurological signs
Hemorrhagic diathesis
Major surgery in preceding 2 weeks
pregnancy
The site answer is: Resolving neurological signs
The correct answer is: Hemorrhagic diathesis
9) A 56-year-old man presents with painless jaundice, weight loss and anorexia. On examination there is a palpable
gallbladder. An ultrasound shows a dilated gallbladder with intrahepatic and extrahepatic duct dilatation. There is a
suspicious mixed echogenic lesion in the head of the pancreas. It is decided that he requires an endoscopic retrograde
cholangiopancreatography (ERCP) to obtain brushings from the pancreatic duct. His clotting studies are prothrombin
time 21 sec, activated partial prothrombin time 36 sec. You are informed that his prothrombin time needs to be
reduced to 13 sec for his ERCP. What is the most appropriate pharmacotherapy to correct the prothrombin time?
Activated factor VII
Platelet transfusion
Tranexamic acid
Warfarin
Vitamin K
The answer is: Vitamin K
10) A 65-year-old female requires emergency surgery for a strangulated inguinal hernia. Which of the following is
correct?
The sac is formed by an unobliterated processus vaginalis.
The hernia is direct rather than indirect.
Such herniae never contain small intestine
Strangulation never results in bowel ischemia and gangrene requiring resection.
Indirect inguinal herniae are never found in female patients.
The answer is: The sac is formed by an unobliterated processus vaginalis.
11) All of the following may be used in pregnancy-associated hypertension, EXCEPT:
Nifedipine
Captopril
Methyldopa
Hydralazine
Labetalol
The answer is: Captopril
12) Which one of the following is the most appropriate diagnostic investigation in a young adult complaining of
episodic shortness of breath and wheezes?
patch testing
CT chest
Serial peak flow measurement at home and at work
IgE measurement
Skin prick test
The answer is: Serial peak flow measurement at home and at work

139
EMLE Trial Exams October 2020 (1000 Questions) Version (1.0) 9/12/2020

13) Toddler age is charactarized by:


Is around 10 years of age
Physical growth spurt
Changes of gonadal hormonal profile
Personal changes
Can run unsteadily
The answer is: Can run unsteadily
14) Which of the following statements concerning hematuria in children is TRUE:
Urine in glomerular hematuria is usually red
Glomerular hematuria occurs at the beginning of urinary stream
UTI is the most common cause of hematuria
Lower urinary tract hematuria is usually accompanied by proteinuria
Renal biopsy should be done to all cases
The answer is: UTI is the most common cause of hematuria
15) A primigravida at 26 weeks gestation and her CBC showing HB of 9.4 g/dL; MCV of 77fL and TIBC saturation < 15%.
What is the INCORRECT statement regarding her management?
The recommended oral dose of elemental iron is 150 – 300 mg/day.
Oral iron therapy may cause constipation
Parentral iron therapy might induce teratogenic effects
Adding folic acid to the iron formula is better than iron formula alone.
Cyanocobalamin is essential for normal erythropoiesis
The answer is: Parentral iron therapy might induce teratogenic effects
16) A 35-year-old woman comes to the physician for sleeping problems and the inability to concentrate for 3 months.
She says she is worried because she and her husband have been trying to conceive for more than a year with no
success. Conception by in vitro fertilization was attempted once 3 months ago but was unsuccessful. Analysis of her
husband's semen has shown normal sperm counts and morphology. She has a 6-year- old daughter who was born at
term after an uncomplicated pregnancy. She has no history of severe illness and tries to stay healthy by going to the
gym an hour per day. Her menses occur at regular 28-day intervals and last 5 to 6 days; her last menstrual period
started 2 days ago. Physical examination shows no abnormalities. Which of the following is the most likely cause of
this patient's inability to conceive?
Polycystic ovarian syndrome
Pelvic inflammatory disease
Asherman's syndrome
Diminished ovarian reserve
Hypogonadotropic hypogonadism
The answer is: Diminished ovarian reserve
17) A 6 years old girl with offensive vaginal bleeding is brought by her mother, the best next step is:
Local Estrogen.
Examination under anesthesia to exclude retained foreign body
Parenteral ampicillin for infection
Metastatic work-up to exclude malignancy.
MRI
The answer is: Examination under anesthesia to exclude retained foreign body
18) What of the following medications should be given after 14 days of ischemic stroke?
Aspirin alone
Aspirin and statin
Clopidogrel alone
Clopidogrel and statin
Aspirin and dipyridamole and statin

140
EMLE Trial Exams October 2020 (1000 Questions) Version (1.0) 9/12/2020

The answer is: Clopidogrel and statin


19) Which of these factors does not influence healing of a wound?
Vascular insufficiency
Diabetes mellitus.
Malnutrition.
Site of wound.
Thyrotoxicosis.
The answer is: Thyrotoxicosis
20) A 50-year-old patient presents with jaundice 5 days after receiving a red cell transfusion.
air embolus
delayed haemolytic transfusion reaction
hypocalcaemia
viral infection
circulatory overload
The answer is: delayed haemolytic transfusion reaction
21) A 24-year-old patient came to the outpatient with a 6-week history of fever, bloody diarrhea, and weight loss. On
examination, he is clinically anemic and has aphthous ulcer of the mouth and mild tenderness of the abdomen.
Sigmoidoscopy shows transmural inflammation and non-caseating granulomas of the colon and rectum. What is the
most likely diagnosis?
Ulcerative colitis
Chron’s disease
IBS
Diverticulosis
Infectious diarrhea
The answer is: Chron’s disease
22) The mother of a 4 years old son is concerned that he only seems to like to play with his toy train. In his nursery he
plays with the toys but not with other children. His behavior can be very difficult to manage at times. He does not say
any words, whereas his classmate children are speaking in sentences. On examination you notice he does not make
eye contact with you and pushes his toy train back and forth on the floor. The rest of his examination is normal. What
is the most likely diagnosis?
A tic syndrome
Attention deficit hyperactivity disorder
Autism spectrum disorder
Developmental delay
A hearing problem
The answer is: Autism spectrum disorder
23) A 48-year-old smoker, who was diagnosed with COPD 10 years ago, is experiencing
shortness of breath and a productive cough with purulent sputum. These episodes have
become more frequent within the last few years. What is the best management if the
chest x-ray shows a pneumothorax with a 2.5 cm rim of air and no mediastinal shift?
Aspiration
Discharge
Intercostal drain insertion
Repeat chest X-ray
Hospitalization
The answer is: Intercostal drain insertion

141
EMLE Trial Exams October 2020 (1000 Questions) Version (1.0) 9/12/2020

24) 46 years old female pt. presented with incidental finding of liver mass on CT suspected to be haemangioma, the
best management is:
Lobectomy
Segmentectomy
Injection sclerotherapy
Observation
Radiofrequency
The answer is: Observation
25) Which of the following is the MOST LIKELY mechanism of septic abortion following an induced abortion?
Instrumental contamination
Ascending infection
Skin organisms
Urinary tract penetration
Hematogenous infection
The answer is: Ascending infection
26) At birth the normal heart rate is:
60-80 /min
80-110/min
110-150/min
70-120/min
180-200/min
The answer is: 110-150/min
27) A 42-year-old woman presents with an upper midline mass that has been present for over a year. She has a
history of a partial gastrectomy for a perforated ulcer. On examination, the mass is 5 cm in size, soft, non-tender and
reducible. Which of the following is the most likely diagnosis?
Direct inguinal hernia
Epigastric hernia
Hiatus hernia
Incisional hernia
Paraumbilical hernia
The answer is: Incisional hernia
28) A 61-year-old nulliparous woman comes to the physician for a follow-up examination. Her last Pap smear 3 years
ago showed atypical squamous cells of undetermined significance. HPV testing was negative at that time. On
questioning, she has had fatigue and an increase in abdominal girth despite a 5-kg (11.0-lb) weight loss over the past 6
months. She has gastroesophageal reflux disease and Hashimoto's thyroiditis. Menarche was at the age of 10 years
and her last menstrual period was 2 years ago. Current medications include omeprazole and levothyroxine.
Abdominal examination shows shifting dullness. There is tenderness to palpation of the right lower quadrant but no
guarding or rebound. Bimanual palpation shows a small uterus and a right adnexal mass. Further evaluation of this
patient is most likely to show which of the following findings?
Proliferation of endometrial glands
Elevated serum CA-125 level
Elevated serum beta-hCG level
Prolonged prothrombin time
Chocolate cyst of the right ovar
The answer is: Elevated serum CA-125 level
29) You are assisting with a total thyroidectomy for a patient who has a goitre and is experiencing compressive
symptoms. The Surgical Registrar asks you to name the artery supplying the thyroid gland, which originates from the
thyrocervical trunk. From the list below, choose the most likely answer.
Inferior thyroid artery

142
EMLE Trial Exams October 2020 (1000 Questions) Version (1.0) 9/12/2020

Thyroid ima artery


Superior thyroid artery
Ascending pharyngeal artery
Lingual artery
The answer is: Inferior thyroid artery
30) You review a 24-year-old woman with a history of asthma in the Emergency Department. She has been admitted
with acute shortness of breath associated with tongue tingling and an urticarial rash after eating a meal containing
shellfish. Her symptoms settle with nebulized salbutamol and intravenous hydrocortisone . What is the diagnostic test
of asthma severity?
Arterial blood gases
Spirometry
Serum IgE
Serum tryptase
Eosinophil count
The answer is: Spirometry
31) The correct statement for hormonal contraception:
Ethinyl Estradiol (EE) is the estrogen present in the combined oral contraceptive pills (COCS).
Progestins are synthetic compounds that mimic the structure of natural progesterone
Failure rate is about 1/HWY with COCS and 5/HWY with POPs
Combined oral contraceptive pills increases the risk of anemia
History of hepatic adenoma is not a contraindication of COC pills
The answer is: Ethinyl Estradiol (EE) is the estrogen present in the combined oral contraceptive pills (COCS).
32) A 2 years old male presented by high grade fever and refusal of feeding for one day followed by appearance of
purpuric rash and extreme irritability. You should suspect
Measles
Scarlet fever
Meningococcemia
Typhoid fever
Mumps
The answer is: Meningococcemia
33) All of the following are characteristics of β-thalassemia major except:
Pallor since birth
Hepatosplenomegaly
Reticulocytosis
Presence of target cells in peripheral blood
Hair on end appearance on skull x-ray
The answer is: Pallor since birth
34) In trauma imaging, which of the following statement is false?
In a multiply injured patient, CT of head and spine should be the first line of imaging.
Focused assessment with sonography for trauma helps in detecting intraperitoneal fluid and cardiac tamponade.
CT should not be used when a patient is unstable
U/S is useful for diagnosing occult pneumothorax
CT is the main imaging method for intracranial, intra-abdominal and vertebral injuries.
The answer is: In a multiply injured patient, CT of head and spine should be the first line of imaging.
35) A healthy primigravida is admitted at 30 weeks' gestation with spontaneous rupture of the membranes. The
following statement is correct:
Fetal pulmonary hypoplasia may occur due to oligohydramnios
Prophylactic antibiotics are contraindicated
Corticosteroids are contraindicated because of the risk of infection
143
EMLE Trial Exams October 2020 (1000 Questions) Version (1.0) 9/12/2020

Active management results in a better perinatal outcome than is achieved by expectant management
The risk of preterm prelabor rupture of the membranes in subsequent pregnancies is
The answer is: Fetal pulmonary hypoplasia may occur due to oligohydramnios
36) What type of pancreatic tumour is responsible for Zollinger–Ellison syndrome?
Insulinoma
Gastrinoma
VIPoma
Adenocarcinoma
Glucagonoma
The answer is: Gastrinoma
37) Mesalazine is used as a treatment and maintenance of remission of which of the following diseases?
Behcet’s disease
Celiac disease
Primary dysmenorrheal
Ulcerative colitis
Whipple’s disease
The answer is: Ulcerative colitis
38) What is the level of BP at which thrombolytic can be given to stroke patients?
<200/100
<240/120
<185/110
<140/90
<120/90
The answer is: <185/110
39) Child 5 years old presents with purpuric eruptions of one-week duration and nose bleeds he is otherwise good and
active, what is the first investigation you will request?
CBC
PT
PTT
INR
CT
The answer is: CBC
40) An 8- year- old girl presented with low grade fever and diffuse maculopapular rash. On examination her physician
noted mild tenderness & swelling of her cervical & occipital lymph nodes. Three days after the onset of illness, the
rash has vanished. What is the most likely diagnosis?
Measles
German measles
Scarlet fever
Infectious mononucleosis
Mumps
The answer is: German measles
41) A 4-year-old boy presents with new-onset type 1 diabetes mellitus and diabetic ketoacidosis. He appears tired
but, on physical examination, shows only mild signs of dehydration. Initial laboratory tests reveal the following
results: • Serum glucose, 884 mg/dL • Serum potassium, 4.5 mEq/L • pH, 6.92 on arterial blood gas. Of the following,
the MOST important first step in managing this patient is to administer:
an insulin drip, beginning at 0.1 units/kg/h
an intravenous insulin bolus of 0.1 units/kg
lactated Ringer solution, 20 mL/kg over 15 minutes
normal saline, 10 to 20 mL/kg over 1 to 2 hours
144
EMLE Trial Exams October 2020 (1000 Questions) Version (1.0) 9/12/2020

sodium bicarbonate, 1 to 2 mmol/kg over 60 minutes


The answer is: normal saline, 10 to 20 mL/kg over 1 to 2 hours
42) Clinical clues for the diagnosis of portal hypertension include all except:
Splenomegaly
Hematemesis
Melena
Ascites
Pleural effusion
The answer is: Pleural effusion
43) Child 3 years with wet purpura and platelets 3000 what is the best action?
Packed RBC transfusion
Platelets transfusion
Plasma transfusion
Reassure
Observe
The answer is: Platelets transfusion
44) A 25-year-old lady presents with a 2-week history of “growths” in the vulvar region. On examination, you find
multiple “cauliflower” verrucous lesions on the labia majora and minora. What is the most likely diagnosis in this
patient?
condyloma lata
condyloma acuminatum
herpes simplex type 1
herpes simplex type 2
genital acrochordon (skin tags)
The answer is: condyloma acuminatum
45) A 33-year-old female presents 6 weeks after the birth of her first child with a two-week history of polyarthralgia,
fever and a skin rash. First-line investigations show: A 33- year-old female presents 6 weeks after the birth of her first
child with a two-week history of polyarthralgia, fever and a skin rash. First-line investigations show: ESR High. What is
the most likely diagnosis?
Antiphospholipid syndrome
Systemic sclerosis
Systemic lupus
Fibromyalgia
Rheumatoid arthritis
The answer is: Systemic lupus
46) A 30-year-old primigravida presents at 34 weeks gestational age with blood pressure of 170/100 mmHg,
headache, epigastric pain, visual abnormalities and +3 proteinuria. Biophysical profile of the fetus is 8/8. Which one
of the following is the immediate response?
Start magnesium sulfate intravenously
Perform an emergency C-section.
Give betaclomethasone to induce fetal lung maturity
Perform an amniocentesis to assess fetal lung maturity
Repeat the biophysical profile daily.
The answer is: Start magnesium sulfate intravenously
47) A 22-year-old pedestrian is brought to the Emergency Department following a road traffic collision with a car
travelling at 60 kmph. She was witnessed to have ‘bullseyed’ the windscreen of the car. On presentation she has a
primary survey which revealed a Glasgow Coma Score of 9. What is the most appropriate management option?
Trauma CT
Chest drain
145
EMLE Trial Exams October 2020 (1000 Questions) Version (1.0) 9/12/2020

Laparotomy
CT head
CT neck
The answer is: CT head
48) When a woman has painless vaginal bleeding at 37 weeks’ gestation, which of the
following measures is immediately indicated?
Rupture of membranes
Urgent ultrasound
Coagulation profile
Induction of labor
Cesarean section
The answer is: Urgent ultrasound
49) In a case of recurrent spontaneous abortion, the following investigation is unwanted:
Hysteroscopy
Testing for antiphospholipid antibodies
Thyroid function tests
Testing for TORCH infections
None of the above
The answer is: Testing for TORCH infections
50) A 65-year-old man one week after an appendectomy suddenly experiences left sided (pleuritic) chest pain with
dyspnoea. The previous night he had been restless with a temperature of 37.2C. ECG was normal. No leg signs.
Immediate therapy would be:
Intranasal oxygen and serial chest x-rays
Heparinisation
Antibiotics
Observe closely for further signs
Chest physiotherapy
The answer is: Observe closely for further signs
51) A child 6 years old witnessed to have loss of conscious for 6 min after history of head trauma otherwise he is good
now so what is the recommendation?
Urgent CT
Admit and observe for 6 h
Reassure MRI
Intubate
The answer is: Admit and observe for 6 h
52) Which statement is true about urticaria/angioedema?
It is a rare allergic disorder in children
Not caused by local contact with allergen
Food allergy is common
Edema occurs independent parts of the body
Topical corticosteroids are the mainstay of therapy
The answer is: Food allergy is common
53) A 24-year-old patient came to the outpatient with a 6-week history of fever, bloody diarrhea, and weight loss. On
examination, he is clinically anemic and has aphthous ulcer of the mouth and mild tenderness of the abdomen.
Sigmoidoscopy shows transmural inflammation and non-caseating granulomas of the colon and rectum. If the patient
also complains of that lesion on his shins: What is the diagnosis of this skin lesion?
Erythema nodosum
Pyoderma gangerosum
Dermatitis hepetitformis
146
EMLE Trial Exams October 2020 (1000 Questions) Version (1.0) 9/12/2020

Eczyma
Atopic dermatitis
The answer is: Erythema nodosum
54) What is the treatment of acute attacks of Meniere’s disease?
IV ibuprofen
Betahistidine
Buccal or intramuscular prochlorperazine
IV morphine
IV tramadol
The answer is: Buccal or intramuscular prochlorperazine
55) According to Glasgow coma scale for a child 3 years old what score will you give for a child with eye opening
spontaneous (for his eye)?
2
6
4
8
10
The answer is: 4
56) In iron deficiency anemia in pregnancy, the following is true:
Mean corpuscular hemoglobin content and mean corpuscular concentration are both low.
Mean corpuscular volume is raised
Blood transfusion is indicated if HB level fall below 9 gm%
There is usually a chronic blood loss causing the anemia.
There is an increased risk of preeclampsia
The answer is: Mean corpuscular hemoglobin content and mean corpuscular concentration are both low.
57) A 36-year-old man presents to the emergency department with a severe left-sided headache with pain around the
left eye. He has had several similar episodes over the last 2 weeks, lasting 40-60 minutes each. The headaches are
associated with a runny nose. On examination, there is redness and tearing of his left eye. What is the most
appropriate acute management?
Acetazolamide
High flow O2
Ibuprofen and paracetamol
Sumatriptan
Urgent CT brain
The answer is: High flow O2
58) Child 8 years has shocked in a food in front of you cannot cough but he is conscious what will you do?
Heimlich maneuver
CPR
Encourage coughing
Send to ER
Call ambulance
The answer is: Heimlich maneuver

59) An 85-year-old male patient with a history of chronic constipation presents with acute severe colicky abdominal
pain and absolute constipation. Plain abdominal film shows a grossly dilated oval of large bowel arising from the left
lower quadrant. A diagnosis of sigmoid volvulus is made. The next step in management is:
Laparotomy
Sigmoidoscopy with flatus tube insertion
Sigmoid colectomy with colostomy
147
EMLE Trial Exams October 2020 (1000 Questions) Version (1.0) 9/12/2020

Barium swallow
Computed tomography
The answer is: Sigmoidoscopy with flatus tube insertion
60) A 79-year-old patient is admitted to the cardiology service and treated for acute CHF. He is started on a new
medication regimen including a diuretic which relieves his symptoms and improves his cardiac function. He is
discharged home, though he returns to the hospital 10 days later with another episode of CHF. During the
readmission, the team notices that the patient never filled his new prescriptions and was not taking the prescribed
diuretic while at home. What actions can be taken to prevent this from happening again?
Provide timely access to care following a hospitalization
Communication and coordination of care plan with patients is not a solution
Discharge planning and transition processes cannot help
patient education cannot help
support to optimize home care is not the proper a
The answer is: Provide timely access to care following a hospitalization
61) A healthy primigravida is admitted at 30 weeks' gestation with spontaneous rupture of the membranes. The
following statement is correct:
Fetal pulmonary hypoplasia may occur due to oligohydramnios.
Prophylactic antibiotics are contraindicated
Corticosteroids are contraindicated because of the risk of infection
Active management results in a better perinatal outcome than is achieved by expectant management
The risk of preterm prelabor rupture of the membranes in subsequent pregnancies is 7
The answer is: Fetal pulmonary hypoplasia may occur due to oligohydramnios.
62) Which is correct about vaginal discharge:
Cervical ectopy might induce vaginal discharge.
Normally the vagina is dry.
Progesterone causes a proliferation of the vaginal epithelium.
Atrophic vaginitis is uncommon in postmenopausal women.
Excessive vaginal discharge must indicate local infect.
The answer is: Cervical ectopy might induce vaginal discharge.
63) A 40-year-old woman presents with recurrent episode of vertigo associated with a feeling or 'fullness' and
'pressure' in her ears. She thinks her hearing is worse during the attacks. Clinical examination is unremarkable. What
is the most likely diagnosis?
Meniere’s
Benign paroxysmal disease
positional vertigo
Acoustic neuroma
Cholesteatoma
Somatization
The answer is: Meniere’s
64) A 45-year-old man collapses at home and is brought to accident and emergency. He has a fever at 39.5°C and
blood pressure is 90/60 mmHg, although he is in a lucid state. Bruises can be seen on his skin which he remembers
being present before he fell. Blood tests show the patient to have a normocytic anaemia with a low platelet count
and increased fibrin split products. The most likely diagnosis is:
Warm autoimmune haemolytic anaemia
Cold autoimmune haemolytic anaemia
Paroxysmal nocturnal haemoglobinuria
Disseminated intravascular coagulation
Thalassaemia minor
The answer is: Disseminated intravascular coagulation

148
EMLE Trial Exams October 2020 (1000 Questions) Version (1.0) 9/12/2020

65) A 75-year-old ex-smoker presents to the vascular outpatient clinic with a claudication distance of several yards
and rest pain. A CT angiogram reveals multi-level atherosclerotic disease, including a flush short segment occlusion of
the proximal superficial femoral artery, an occluded proximal popliteal artery. There appears to be adequate run-off
below the knee. What is the most appropriate management?
Above knee amputation
Angioplasty
Femoral distal bypass
Embolectomy
Endovascular (abdominal) aneurysm repair (EVAR)
The answer is: Femoral distal bypass
66) Commonest congenital heart in Down syndrome:
ASD
AVSD
PDA
F4
TGA
The answer is: AVSD
67) An 8-year-old is accidentally hit in the abdomen by a football. After several minutes of discomfort, he seems to be
fine. Over the ensuing 24 hours, however, he develops a fever, abdominal pain radiating to the back, and persistent
vomiting. On examination, the child appears quite uncomfortable. The abdomen is tender, with decreased bowel
sounds throughout, but especially painful in the mid-epigastric region with guarding. Which of the following tests is
most likely to confirm the diagnosis?
Serum amylase levels
CBC with differential and platelets
Serum total and direct bilirubin levels
Abdominal radiograph
Electrolyte panel
The answer is: Serum amylase levels
68) A 32-year-old man is stabbed in the left side of the chest. Chest x-ray shows a whiteout of the left lung field.
Hemothorax
diaphragmatic rupture
diaphragmatic contusions
myocardial contusion
pleural effusion
The answer is: Hemothorax
69) Which of the following is not included in Standard Infection Control Precautions?
Management of blood and bodily fluid spillage
Patient’s personal hygiene
safe handling of linen
Cleanliness of care equipment
Sharps safety
The answer is: Patient’s personal hygiene
70) Skeletal metastasis commonly arises from cancer of:
Thyroid
Bronchus
Breast
Prostate
Melanoma
The answer is: Prostate
149
EMLE Trial Exams October 2020 (1000 Questions) Version (1.0) 9/12/2020

71) A newborn appears with jaundice within the first 24 hours. The most likely cause is:
Polycythemia
Caput succedaneum
Bruises over the face
Biliary atresia
RH incommutability
The answer is: RH incommutability
72) A 40-year-old man underwent laparoscopic cholecystectomy 2 years earlier. He remains asymptomatic until 1
week before admission, when he complains of RUQ pain and jaundice. He develops a fever and has several rigor
attacks on the day of admission. An ultrasound confirms the presence of gallstones in the distal CBD. The patient is
given antibiotics. Which of the following should be undertaken as the next step in therapy?
Should be discharged home under observation
Should be observed in the hospital
Undergo surgical exploration of the CBD
ERCP with sphincterotomy and stone removal
Anticoagulants
The answer is: ERCP with sphincterotomy and stone removal
73) An 18-year virgin girl presents with on-off pelvic pain in last 4 months. In many occasions it is severe and
interfering with her activities. Her pain in most of times was not related to her menses. She has regular menses. Her
last menses started 10 days ago. Abdominal examination revealed lower abdominal tenderness with positive
rebound. She has low grade fever. Her pulse is 92 and BLP is 110/60. Her hemoglobin is 14gm/l. Ultrasound shows a
right 10 cm ovarian cyst and no fluid collection. Abdominal x-ray revealed calcifications inside the cyst. The likely
etiology for her pain will be:
Torsion dermoid cyst.
Ruptured corpus luteum cyst.
PID
Hemorrhagic corpus luteum cyst.
Monilial infection
The answer is: Torsion dermoid cyst.
74) A 65-year female complains regurgitation of undigested food, few hours after meal, she also have progressive
difficulty in swallowing, weight loss and swelling in the neck. What is your diagnosis:
Achalasia.
Cancer esophagus.
Pharyngeal pouch.
Plummer vinson`s syndrome
Diffuse esophageal spas
The answer is: Pharyngeal pouch.
75) As regards follicle stimulating hormone, all the following are correct EXCEPT:
It stimulates spermatogenesis
Its plasma concentration is high in Klinefelter syndrome
It stimulates ovarian estrogen production
It is secreted by basophilic cells of the adenohypophysis .
It prevents regression of the corpus luteum
The answer is: It prevents regression of the corpus luteum
76) In “catabolic” surgical patients, which of the following changes in body composition do not occur?
Lean body mass increases.
Total body water increases
Adipose tissue decreases.
Body weight decreases.

150
EMLE Trial Exams October 2020 (1000 Questions) Version (1.0) 9/12/2020

All of the above


The answer is: Lean body mass increases.
77) Child 3 months old presents with decrease feeding tolerance and on examination you found harsh pan systolic
murmur over the left parasternal area what is the most suspected lesion:
VSD
ASD
PDA
F4
TGA
The answer is: VSD
78) Which of the following statements regarding preoperative investigations is false?
Chest X-ray is routinely requested in all patients over 60 years old.
A ventricular ejection fraction of less than 35 per cent indicates a high risk of cardiac complications.
A body mass index (BMI) <15 is associated with significant hospital mortality.
ECG is usually required in patients above 65 years old
HIV testing requires patient’s consent.
The answer is: Chest X-ray is routinely requested in all patients over 60 years old.
79) A 54-year-old man is admitted to the Emergency Department with a left hemiplegia. His symptoms started around
5 hours ago and he has had no headache, visual disturbance or loss of consciousness. On examination a dense left
hemiplegia is noted. Blood pressure is 120/78 mmHg, GCS is 15/15 and pupils are equal and reactive to light. An
urgent CT scan is performed shortly after his arrival. This demonstrates no abnormality. What is the most appropriate
initial management?
Aspirin
Alteplase
Warfarin
Enoxaparin
Dexamethasone
The answer is: Aspirin
80) As regards luteinizing hormone the followings are true EXCEPT:
LH surge coincides with the second peak of FSH just before ovulation
LH is the stimulus for ovulation
It stimulates the synthesis of testosterone in the male
Its release is stimulated by thyrotrophin releasing hormone
It is a water-soluble glycoprotein
The answer is: Its release is stimulated by thyrotrophin releasing hormone
81) A 49-year old man presents with a4-weekhistory of recurrent head aches occurring at the same time every day,
shortly after going to sleep.it usually associated with pain around the right eye and lacrimation. there is no aura and
episode lasts less than an hour. the headache is relieved by movement. What is the cause of this headache?
Giant arteritis
subarachioid hemorrhage
tension headache
cluster headache
migraine
The answer is: cluster headache
82) Variable features of kwashiorkor include all of the following except:
anemia
ectodermal changes
GI manifestations
growth retardation
151
EMLE Trial Exams October 2020 (1000 Questions) Version (1.0) 9/12/2020

multiple vitamin and mineral deficiencies


The answer is: growth retardation
83) A patient with poorly controlled asthma is started on montelukast. What is the mechanism of action of this drug?
B2-receptor antagonist
B2-receptor agonist
leukotriene receptor agonist
leukotriene receptor antagonist
muscarinic receptor blocker
The answer is: leukotriene receptor antagonist
84) A 48-year-old man presents with 3-month history of worsening abdominal pain radiating to the back, associated
weight loss and progressive anorexia. Over the last week his wife has told him that he is looking yellow. What is the
most likely diagnosis?
Chronic pancreatitis
Acute pancreatitis
Carcinoma of the head of the pancreas
Carcinoma localized to the body or tail of the pancreas
Gastric tumour
The answer is: Carcinoma of the head of the pancreas
85) Which of the following statements about ischaemia-reperfusion syndrome is correct?
This refers to the cellular injury because of the direct effects of tissue hypoxia.
It is seen after the normal circulation is restored to the tissues following an episode of hypoperfusion.
The increased sodium load can lead to myocardial depression.
This is not influenced by the duration and extent of tissue hypoperfusion.
It usually does not cause death
The answer is: It is seen after the normal circulation is restored to the tissues following an episode of hypoperfusion.
86) Best method used to reduce nosocomial infection in NICU is:
Handwashing
Wearing gloves
Wearing gown
Wearing shoe covers
Wearing mask
The answer is: Handwashing
87) A 68-year-old woman presents to the Emergency Department complaining of early satiety and vomiting
immediately after eating. Ten weeks prior to her presentation, she was treated as an inpatient for an episode of gall
stone pancreatitis and is awaiting a laparoscopic cholecystectomy. On examination, she is well, her heart rate is 78
beats/min, her blood pressure is 140/78 mmHg and temperature is 36.5°C. Examination of her abdomen revealed a 20
x 15cm epigastric mass which was smooth, fixed, and fluctuant in nature. It is pulsatile but not expansile in nature and
is non-tender. You are able to get above and below the mass. From the following list, select the most likely cause of
this intra-abdominal mass.
Splenomegaly
Abdominal aortic aneurysm
Hypertrophy of the pylorus
Pancreatic abscess
Pancreatic pseudocyst
The answer is: Pancreatic pseudocyst
88) The MOST COMMON congenital fetal malformation in diabetes mellitus is:
Ventricular and atrial septal defect(CVS).
Anenchephaly.
Spina bifida.
152
EMLE Trial Exams October 2020 (1000 Questions) Version (1.0) 9/12/2020

Sacral agenesis.
None of the above
The answer is: Ventricular and atrial septal defect(CVS).
89) A 2-year-old girl develops a rash on her legs. The next day she is brought to surgery, by which time the rash has
spread to the rest of her body.What is the most likely diagnosis?
Erythema multiforme
Erytema chronica migrans
Erythema nodosum
Urticarial
Dermatitis artefacta
The answer is: Erythema multiforme
90) Five years old child presenting with lethargy, respiratory distress, wheezes, cold extremities, BP: 60/30, weak
pulse. He received an injection of an unknown drug just few minutes before developing these symptoms . What is the
MOST appropriate management?
ABC + IV steroids ± antihistamine
ABC + IM steroids ± antihistamine
ABC + Inhaled ß2 agonist
ABC + IM adrenaline
ABC + SC adrenaline
The answer is: ABC + IM adrenaline
91) You see a patient on the Intensive Care Unit who has been admitted with severe pancreatitis. He is having a
central venous catheter inserted for intravenous fluid monitoring. Other than using a central venous cannula to
measure central venous pressure during fluid resuscitation, from the list below choose the answer which correctly
describes a long-term use of a central venous cannula.
Haemodialysis
Total parenteral nutrition
Pulmonary artery catheterization
Drug administration
Transvenous cardiac pacing
The answer is: Total parenteral nutrition
92) A 30-year-old woman, gravida 2, para 1, at 28 weeks' gestation comes to the physician for a prenatal visit. She
feels well. Pregnancy and delivery of her first child were uncomplicated. She has a history of bipolar disorder and
hypothyroidism. She uses cocaine once a month and has a history of drinking alcohol excessively, but has not
consumed alcohol for the past 5 years. Medications include quetiapine, levothyroxine, folic acid, and a multivitamin.
Her temperature is 37.1°C (98.8°F), pulse is 88/min, and blood pressure is 115/75 mm Hg. Pelvic examination shows a
uterus consistent in size with a 28-week gestation. Serum studies show a hemoglobin concentration of 11.2 g/dL and
thyroid-stimulating hormone level of 3.5 µU/mL. Her fetus is at greatest risk of developing which of the following
complications?
Aplasia cutis congenital
Cretinism
Chromosomal abnormality
Neural tube defect
Premature placental separation
The answer is: Neural tube defect
93) The INCORRECT statement regarding patient with stage Ib carcinoma of the cervix undergoes a radical
hysterectomy and pelvic lymphadenectomy:
Prophylactic heparin is mandatory
Irradiation is an alternative comparable therapeutic modality
Ureteric fistulae are usually due to intra-operative surgical trauma

153
EMLE Trial Exams October 2020 (1000 Questions) Version (1.0) 9/12/2020

Significant long-term bladder dysfunction is common


Pelvic lymphocyst formation is characteristically a late complication
The answer is: Prophylactic heparin is mandatory
94) A 22-year-old medical student revising for finals presents to his GP complaining of a spherical, painless swelling of
the elbow. On examination there is a posterior joint swelling which is fluctuant. What is the most likely lesion?
Cubital tunnel syndrome
Epicondylitis
Olecranon bursitis
Osteochondritis dissecans
Triceps tendinitis
The answer is: Olecranon bursitis
95) What are the earliest changes observed by ophthalmoscope in background retinopathy of diabetes?
Arterio-venous shunts
Exudates
Increased capillary permeability
Microaneurysms
Venous dilatation
The answer is: Microaneurysms
96) The investigation of choice in case of post-menopausal bleeding after Trans-vaginal Ultrasound is:
Pap smear
Fractional curettage
CA-125
Breast Ultrasound
MRI
The answer is: Fractional curettage
97) A 42-year-old obese woman presents with a 9-month history of burning epigastric pain which is worse at night or
after oily food. Over the last 2 weeks she has developed occasional dysphagia to solids. Endoscopy shows hiatus
hernia, inflammation and stricture at the lower third of esophagus. what is your diagnosis? (Debatable)
Bulbar palsy
Esophageal carcinoma
GERD
cardiac achalasia
esophageal candidiasis
The site answer is: esophageal candidiasis
99) Which of the following statement(s) is/are correct concerning the cardiovascular response to shock? (Debatable)
Changes in cardiac contractile function shift the Frank Starling curve up and down
Venoconstriction from skeletal muscle is a significant contributor to the restoration of blood volume with shock
Arterial vasoconstriction affects all vascular beds equally
The total circulating blood volume is equally split between the arterial and venous system
None of the above.
The answer is: Changes in cardiac contractile function shift the Frank Starling curve up and down
100) Complications of evacuation and curettage of the uterus DO NOT include:
Uterine perforation.
Hemorrhage.
Infection.
Asherman’s syndrome
Amniotic band syndrome.
The answer is: Amniotic band syndrome.

154
EMLE Trial Exams October 2020 (1000 Questions) Version (1.0) 9/12/2020

Exam 9
1) What is the commonest cause of steatorrhea?
a) Shigella
b) Salmonella
c) Campylobacter
d) E-coli
e) Giardia
The answer is: Giardia
2) What of the following investigations are specific for autoimmune hepatitis?
a) ALT & AST
b) Alkaline phosphatase
c) GGT
d) ASMA
e) ANA
The answer is: ASMA
3) What is the treatment of acute attacks of Meniere’s disease?
a) IV ibuprofen
b) betahistidine
c) buccal or intramuscular prochlorperazine
d) IV morphine
e) IV tramadol
The answer is: buccal or intramuscular prochlorperazine
4) The most common cause in newborns with LGA (large for gestational age) is:
a) Maternal obesity
b) Beckwith-Wiedemann syndrome
c) Maternal diabetes
d) Wilms tumor
e) Simpson-Golabi-Behmel overgrowth syndrome (SGBS)
The answer is: Maternal diabetes
5) Microinvasion of carcinoma of the cervix involves a depth below the base of the epithelium of no more than:
a) 1 mm
b) 2 mm
c) 3 mm
d) 4 mm
e) 5 mm
The answer is: 3 mm
6) A 28-year-old P1 CS was diagnosed gestational hypertension at 28 weeks of gestation. She presents at 32 weeks
with pain in abdomen. O/E: P 115 bpm, BP 100/60 mmHg, and Hb 6 g/dl. Uterus is 32–34 weeks tonically contracted
with absent fetal heart sounds. P/V examination revealed no bleeding seen. The diagnosis is:
a) Ruptured uterus.
b) Internal bleeding from placenta previa
c) Concealed abruptio placentae
d) Revealed abruptio placentae
e) Mixed type of abruptio placentae
The answer is: Concealed abruptio placentae
155
EMLE Trial Exams October 2020 (1000 Questions) Version (1.0) 9/12/2020

7) A surgeon is about to close a Lanz incision made for an appendicectomy. The anesthetist asks if they will insert
lidocaine into the rectus sheath to aid with post-surgery analgesia. What is the mechanism of action of lidocaine?
a) Blockade of muscarinic acetylcholine receptors
b) Blockade of nicotinic acetylcholine receptors
c) Blockade of voltage gated calcium channels
d) Blockade of voltage gated sodium channels
e) Blockade of voltage gated potassium channels
The answer is: Blockade of voltage gated sodium channels
8) An 85-year-old male patient with a history of chronic constipation presents with acute severe colicky abdominal
pain and absolute constipation. Plain abdominal film shows a grossly dilated oval of large bowel arising from the left
lower quadrant. A diagnosis of sigmoid volvulus is made. The next step in management is:
a) Laparotomy
b) Sigmoidoscopy with flatus tube insertion
c) Sigmoid colectomy with colostomy
d) Barium swallow
e) Computed tomography
The answer is: Sigmoidoscopy with flatus tube insertion
9) A 70-year-old cigarette smoker presents with a right inguinal mass that has enlarged and has caused discomfort in
recent months. He complains of recent difficulty with micturition and nocturia. The swelling, which does not extend
to the scrotum, reduces when resting. What is the likely diagnosis?
a) Direct inguinal hernia (B) Strangulated indirect inguinal hernia
b) Hydrocele
c) Aneurysm of the femoral artery
d) Cyst of the cord
e) Non of the above
The answer is: Direct inguinal hernia (B) Strangulated indirect inguinal hernia
10) You are asked to assist the lead surgeon with a midline laparotomy in theatre. The patient has small bowel
obstruction confirmed by CT imaging. Before the start of the operation, you are asked what layers, from superficial to
deep, would be cut through during a midline laparotomy incision. Which of the following is the most likely answer?
a) Skin, subcutaneous fat, Scarpa's fascia, external oblique, internal oblique, transversalis fascia, extraperitoneal fat and
peritoneum.
b) Scarpa's fascia, skin, linea alba, transversalis fascia, extraperitoneal fat, subcutaneous fat and peritoneum.
c) Skin, Scarpa's fascia, linea alba, transversalis fascia, extraperitoneal fat, subcutaneous fat and peritoneum.
d) Linea alba, Scarpa's fascia, skin, external oblique, internal oblique, transversalis fascia, extraperitoneal fat,
subcutaneous fat and peritoneum.
e) Skin, subcutaneous fat, Scarpa's fascia, linea alba, transversalis fascia, extraperitoneal fat and Peritoneum.
The answer is: Skin, subcutaneous fat, Scarpa's fascia, linea alba, transversalis fascia, extraperitoneal fat and Peritoneum
11) A 22‐year‐old male presents to the trauma bay with a 6 cm laceration to his right forearm after punching through
a window. His neurovascular exam is within normal limits. The incident happened approximately 45 minutes ago. You
decide to give him prophylactic antibiotics. You want to examine the wound and close it in the trauma bay. Your
intern asks you the mechanism of action of local anesthetics. You tell him that:
a) The exact mechanism is unknown
b) Anesthetics work by reversibly binding calcium channels along the nerve
c) Anesthetics work by blocking acetylcholine release from the presynaptic cleft

156
EMLE Trial Exams October 2020 (1000 Questions) Version (1.0) 9/12/2020

d) Anesthetics work by reversibly binding sodium channels within the nerve fibers
e) Anesthetics work by competitively inhibiting potassium channels along the nerve
The answer is: Anesthetics work by reversibly binding sodium channels within the nerve fibers
12) A 34-year-old woman who presents with confusion, headache and fever is admitted to the Emergency
Department. Shortly after admission she has a seizure. A MRI scan is performed which shows patchy hemorrhagic
changes in the temporal lobe. Given the likely diagnosis, what is the treatment of choice?
a) Supportive treatment+ IV cefotaxime
b) Supportive treatment+ IV acyclovir
c) Supportive treatment+ amphotrecin
d) Supportive treatment alone
e) Supportive treatment+ IV immunoglobulin
The answer is: Supportive treatment+ IV acyclovir
13) A 34‐year‐old man was involved in a motorcycle accident. On arrival, his initial systolic blood pressure (SBP) was
80/50 mm Hg but subsequently improved to 120/65 mm Hg after 2 L of normal saline bolus. His Glasgow coma scale
(GCS) was 13. He was taken to the computerized scan (CT). His head CT showed depressed skull fracture with a
subdural hematoma. While being transported back to the trauma bay, he again became hypotensive with SBP 85/50
mm Hg. What would be the most appropriate treatment for this patient?
a) Administer 2 L of normal saline and continue to observe
b) Administer 2 units of O‐ and continue to observe
c) Arterial embolization by interventional radiologist
d) Administer 2 units of O‐ and take the patient directly to surgery
e) Type and cross and wait for radiologist report
The answer is: Administer 2 units of O‐ and take the patient directly to surgery
14) A 38-year-old man is brought into the resuscitation room having been involved in a house fire. On arrival you
estimate him to have 20% burns. The man is around 80 kg in weight. How much fluid should the patient have over the
next 12 hours?
a) 300 mL
b) 600 mL
c) 1200 mL
d) 2400 mL
e) 3000 mL
The answer is: 2400 mL
15) Child 5 years old is presented with cough, fever 38 and stridor and you have diagnosed croup what is the best line
of treatment:
a) penicillin
b) Dexamethasone
c) Bronchodilator
d) Erythromycin
e) Kanamycin
The answer is: Dexamethasone

157
EMLE Trial Exams October 2020 (1000 Questions) Version (1.0) 9/12/2020

16) A 30-year-old primigravida presents at 34 weeks gestational age with blood pressure of 170/100 mmHg,
headache, epigastric pain, visual abnormalities and +3 proteinuria. Biophysical profile of the fetus is 8/8. Which one
of the following is the immediate response?
a) Start magnesium sulfate intravenously
b) Perform an emergency C-section
c) Give betaclomethasone to induce fetal lung maturity
d) Perform an amniocentesis to assess fetal lung maturity
e) Repeat the biophysical profile daily
The answer is: Start magnesium sulfate intravenously
17) What of the following is an absolute contraindication to thrombolytic in stroke?
a) Active pancreatitis
b) Previous intracranial hemorrhage
c) Hemorrhagic diathesis
d) Major surgery in preceding 2 weeks
e) Pregnancy
The answer is: Previous intracranial hemorrhage
18) A 33-year-old female presents 6 weeks after the birth of her first child with a two-week history of polyarthralgia,
fever and a skin rash. First-line investigations show: A 33-year-old female presents 6 weeks after the birth of her first
child with a two-week history of polyarthralgia, fever and a skin rash. First-line investigations show: ESR High. What is
the most likely diagnosis?
a) Antiphospholipid syndrome
b) Systemic sclerosis
c) Systemic lupus
d) Fibromyalgia
e) Rheumatoid arthritis
The answer is: Systemic lupus
19) A 68-year-old woman presents to the Emergency Department complaining of early satiety and vomiting
immediately after eating. Ten weeks prior to her presentation, she was treated as an inpatient for an episode of gall
stone pancreatitis and is awaiting a laparoscopic cholecystectomy. On examination, she is well, her heart rate is 78
beats/min, her blood pressure is 140/78 mmHg and temperature is 36.5°C. Examination of her abdomen revealed a 20
x 15cm epigastric mass which was smooth, fixed, and fluctuant in nature. It is pulsatile but not expansile in nature and
is non-tender. You are able to get above and below the mass. From the following list, select the most likely cause of
this intraabdominal mass.
a) Splenomegaly
b) Abdominal aortic aneurysm
c) Hypertrophy of the pylorus
d) Pancreatic abscess
e) Pancreatic pseudocyst
The answer is: Pancreatic pseudocyst
20) at one year, the weight of the normal infant is about:
a) 5 kg
b) 14 kg
c) 10kg
d) 16 kg
e) 18 kg
The answer is: 10kg

158
EMLE Trial Exams October 2020 (1000 Questions) Version (1.0) 9/12/2020

21- A 25-year-old woman at 33 weeks, gestation is noted to have a placenta previa. Upon cesarean section, bluish
tissue densely adherent between the uterus and maternal bladder is noted. The most likely added placental
abnormality is
Placenta accreta
Placenta increta
Placenta percreta
Placental polyp
Placental hemangioma
The answer is Placenta percreta
22- A 46-year-old woman presents with a 12-hour history of severe epigastric pain radiating to her back. It is
associated with profuse vomiting. She is currently awaiting a cholecystectomy for gallstones, and her abdominal X-ray
reveals a sentinel loop of small bowel. What is the most likely diagnosis?
Pancreatitis
Ruptured AAA
Mesenteric ischaemia
Ascending cholangitis
Perforated duodenal ulcer
The answer is Pancreatitis
23- A 78-year-old man presents to his GP with three episodes of frank painless haematuria. His past medical history
includes atrial fibrillation for which he is on digoxin and warfarin. He is a smoker with a 30-pack year history. On
examination, his heart rate is 88 beats/min and irregularly irregular. His blood pressure is 148/84 mmHg. Examination
of his abdomen and external genitalia revealed no abnormality. Digital rectal examination revealed a 50 g enlarged
smooth prostate. Which of the following investigations should the patient undergo as a priority? (Debatable)
Serum prostate specific antigen measurement
Transrectal ultrasound scan of prostate
Flexible cystoscopy
INR and coagulation screen
Non-contrast CT of kidneys, ureters and bladder
The answer is Flexible cystoscopy
24- The INCORRECT statement regarding intrauterine devices:
They neither affect ovulation nor steroidogenesis
Not suitable for lactating mothers
Perforation of the uterus is rare
When intrauterine pregnancy occurs on top of IUCD the risk of spontaneous abortion increases
If a pregnancy occurs in IUCD user, the risk of it being ectopic is higher than in women using othercontraceptive
methods
The answer is Not suitable for lactating mothers
25- G6PD deficiency is inherited as:
AR
AD
X LR
XLD
None of the above
The answer is XLR
159
EMLE Trial Exams October 2020 (1000 Questions) Version (1.0) 9/12/2020

26- A full-term baby boy, born 16 hours ago. His mother is blood group O rhesus positive. The baby is breastfeeding
well but he looks jaundiced. On examination the baby is clinically well. His bilirubin was 9 mg/dl at 10 hours, and six
hours later his bilirubin is 15.5 mg/dl. His blood group was identified as group A rhesus positive. Choose the most
likely cause of the baby’s jaundice:
ABO incompatibility
Physiologic jaundice
Congenital hypothyroidism
Congenital infection
Rhesus hemolytic disease
The answer is ABO incompatibility
27- Which one of the following is most likely to be a contraindication for tocoloysis at 28 weeks gestation?
Suspected abruption
Asymptomatic bacteruria
Group B streptococcal bacteriuria
Uterine fibroid
Contracted pelvis
The answer is Suspected abruption
28- All can cause delayed puberty EXCEPT:
Gonadal dysgenesis
Polycystic ovarian disease
Constitutional factor
Neoplasms: aspituitaryadenomas
Malnutrition as in Starvation & anorexia nervosa
The answer is Polycystic ovarian disease
29- Test not useful in case of tubal pregnancy is
Pelvic examination
Ultrasonography
HCGlevels
Hysterosalpingography (HSG)
non of the above
The answer is Hysterosalpingography (HSG)
30- A 25-year-old lady presents with a 2-week history of “growths” in the vulvar region. On examination, you find
multiple “cauliflower” verrucous lesions on the labia majora and minora.What is the most likely diagnosis in this
patient?
condyloma lata
condyloma acuminatum
herpes simplex type 1
herpes simplex type 2
genital acrochordon (skin tags)
The answer is: condyloma acuminatum

160
EMLE Trial Exams October 2020 (1000 Questions) Version (1.0) 9/12/2020

31- A 6-week-old girl is referred by the General Practitioner with poor weight gain. Her mother states that she
becomes sweaty & tachypneic during feeds. On examination, she has a soft, pansystolic murmur. A chest X-ray is
performed, which shows cardiomegaly & prominent pulmonary vascular markings.
Pulmonary atresia
Pulmonary stenosis
Tetralogy of Fallot
Ventricular septal defect (VSD)
Atrial septal defect (ASD)
The answer is Ventricular septal defect (VSD)
32- A new significant groin lump is found in a 9 year old boy who had previously undergone unsuccessful laparoscopy
for undescendef testis :
maldescended testis
femoral aneurysm
hydrocoele
lipoma of the cord
inguinal hernia
The answer is : Maldescended testis
33- Based on current clinical opinion, management of labor prior to 34 weeks of gestation should include
Corticosteroid administration
Consideration of tocolytics to delay delivery briefly
Antimicrobial prophylaxis to prevent group B streptococcus infection
All of the above
None of the above
The answer is All of the above
34- Which of the following is considered a dangerous sign of headache?
Morning headache and vomiting
Being unilateral
Being bilateral
Being in the morning
Associated with aura
The answer is Morning headache and vomiting
35- A 24-year-old lady presents to the emergency department by acute abdomen. She is married 8 months ago. Her
history is suggestive for recurrent vaginal infections. On examination she is stable. The costovertebral angle is free but
has tender lower abdomen with positive rebound. Ultrasound revealed free fluid in the pelvis. The best next step to
reach diagnosis will be:
Urine analysis with culture and sensitivity
Laparoscopy
Cordocentesis
B-HCG
Urine pregnant test
The answer is B-HCG

161
EMLE Trial Exams October 2020 (1000 Questions) Version (1.0) 9/12/2020

36- A 30-year-old woman, gravida 2, para 1, at 28 weeks' gestation comes to the physician for a prenatal visit. She
feels well. Pregnancy and delivery of her first child were uncomplicated. She has a history of bipolar disorder and
hypothyroidism. She uses cocaine once a month and has a history of drinking alcohol excessively, but has not
consumed alcohol for the past 5 years. Medications include quetiapine, levothyroxine, folic acid, and a multivitamin.
Her temperature is 37.1°C (98.8°F), pulse is 88/min, and blood pressure is 115/75 mm Hg. Pelvic examination shows a
uterus consistent in size with a 28-week gestation. Serum studies show a hemoglobin concentration of 11.2 g/dL and
thyroid-stimulating hormone level of 3.5 µU/mL. Her fetus is at greatest risk of developing which of the following
complications?
Aplasia cutis congenita
Cretinism
Chromosomal abnormality
Neural tube defect
Premature placental separation
The answer is Neural tube defect
37- A 27-year-old G 1 PO at 1 5 week's gestation has a history of chronic hypertension. She presents for her first
prenatal visit. Vital signs: T 98°F, P 66 beats/min, R 1 6 breaths/min, BP 1 46/98 mmHg. She stopped taking her BP
medication 6 weeks ago when she found out that she was pregnant. Of the following medications, which is the best
choice for treating chronic hypertension in pregnancy?
Hydralazine
Hydrochlorothiazide
Labetalol
Lisinopril
Atenolol
The answer is Labetalol
38-- Diagnosis of ectopic pregnancy can be ruled out in almost all clinicall significant cases if
Transvaginal sonography showed no abnormalities in the adnexal region
Patient is using the IUCD for contraception.
Serumb-hCG was negative.
There is no pelvic fluid by transvaginal uterine ultrasound.
Patient did not miss her period.
The answer is Serumb-hCG was negative
39- Child 2 years old his mother’s says that he can stand supported with difficulty and he only say mama what is your
advice?
Nothing as he is good to age
Appraise him as he is advanced
Asses him neurologically
Ask for a CBC
Ask for CRP
The answer is Asses him neurologically
40- An 8-year-old boy goes to his GP following a diarrheal illness. On examination, he has mild jaundice ,which he has
had before & his mother says it " doesn't seem to bother him". What is the most likely diagnoasis?
Autoimmune hepatities
Gilbert syndrome
Hepato-renal syndrome
162
EMLE Trial Exams October 2020 (1000 Questions) Version (1.0) 9/12/2020

Reye syndrome
Salmonella
The answer is Gilbert syndrome
41) In a clinical round, while seeing an infant with congenital heart disease, you have been asked by a medical student
“what is the most common type of congenital heart disease”?
a) Atrial septal defect
b) Persistent arterial duct
c) Pulmonary stenosis
d) Tetralogy of Fallot
e) Ventricular septal defect
The answer is Ventricular septal defect
42) A 18-year-old man is rushed to hospital by his brother, late on Friday evening. The brother states that they were
playing football that morning and his brother was hit on the side of his head with a solid bar while playing. Further
questioning revealed that there was a period of unconsciousness but he recovered and was back to normal after
taking some painkillers. Later that evening, according to the brother, the 17-yearold was drowsier
and he had been complaining of a severe headache and weakness in his left arm. On examination you confirm these
findings and find it increasingly difficult to keep him awake. What is the most likely diagnosis?
a) Subdural hematoma
b) Subarachnoid hemorrhage
c) Drug induced
d) Dural venous sinus thrombosis
e) Extradural hemorrhage
The answer is: Extradural hemorrhage
43) Which of the following conditions is least associated with Helicobacter pylori?
a) Peptic ulcer
b) MALT lymphoma
c) GERD
d) Atrophic gastritis
e) Gastric carcinoma
The answer is: GERD
44) Which one of the following statements is correct regarding the definition of neonatal deaths?
a) Neonatal deaths occur only up to the first 24 hours of life
b) Neonatal deaths occur only up to the first 72 hours of life
c) Neonatal deaths occur only up to the first 7 days of life
d) Neonatal deaths occur only up to the first 21 days of life
e) Neonatal deaths occur only up to the first 28 days of life
The answer is: Neonatal deaths occur only up to the first 28 days of life
45) In which of the following conditions the medical treatment of ectopic pregnancy is contraindicated?
a) Sac size is 3 cm
b) 50 mL blood in pelvis
c) Presence of fetal heart activity
d) Previous ectopic pregnancy
e) Pulse 100 bpm
The answer is: Presence of fetal heart activity
46) A 28-year-old P1 CS was diagnosed gestational hypertension at 28 weeks of gestation. She presents at 32 weeks
with pain in abdomen. O/E: P 115 bpm, BP 100/60 mmHg, and Hb 6 g/dl. Uterus is 32–34 weeks tonically contracted
with absent fetal heart sounds. P/V examination revealed no bleeding seen. The diagnosis is:

163
EMLE Trial Exams October 2020 (1000 Questions) Version (1.0) 9/12/2020

a) Ruptured uterus
b) Internal bleeding from placenta previa
c) Concealed abruptio placentae
d) Revealed abruptio placentae
e) Mixed type of abruptio placentae
The answer is: Concealed abruptio placentae
47) What of the following investigations are specific for non-alcoholic steatohepatitis?
a) AST>ALT
b) ALT>AST
c) High Alkaline phosphatase
d) High GGT
e) High creatinine
The answer is: ALT>AST
48) Bartholin abscess present with variety of symptoms that DO NOT include:
a) Impairment of ability to walk
b) Dyspareunia
c) Local throbbing pain
d) Purulent discharge
e) Pruritus vulva
The answer is: Pruritus vulva
49) Regarding the treatment of anaphylaxis, What is the best site for administering intramuscular adrenaline?
a) Abdomen
b) Anterolateral thigh
c) Anteromedial thigh
d) Deltoid
e) Upper outer buttock
The answer is: Anterolateral thigh
50) Which ONE of the following disorders is inherited as sex- linked recessive disorder?
a) Thalassemia
b) Idiopathic thrombocytopenia.
c) Glucose 6 phosphate dehydrogenase deficiency.
d) Von Willebrand’s disease.
e) Hereditary spherocytosis
The answer is: Glucose 6 phosphate dehydrogenase deficiency.
51) A 24-year-old man who has a sister with adult polycystic kidney disease asks his GP if he could be screened for the
disease. What is the most appropriate screening test?
a) Anti-polycystin 1 antibodies levels
b) CT abdomen
c) PKD1 gene testing
d) Ultrasound abdomen
e) Urine microscopy
The answer is: Ultrasound abdomen
52) A 29 -year-old G1 P0010 woman with regular menses ( every 28 days lasting 5 days) is currently trying to conceive.
She has not used contraception for 6 months . Her ovulation predictor kit revealed an luteinizing hormone
(Incomplete Question) (Debatable)
a) Cystic teratoma
b) Ectopic pregnancy
c) Follicular cyst
164
EMLE Trial Exams October 2020 (1000 Questions) Version (1.0) 9/12/2020

d) Hemorrhagic corpus luteum cyst


e) Serous cystadenoma
The answer is: Follicular cyst
53) Increased circulating prolactin concentration is associated with all the followings EXCEPT:
a) Stress
b) Hypothyroidism
c) Anorexia nervosa
d) Acromegaly
e) Pregnancy
The answer is: Anorexia nervosa
54) On colonoscopy a malignant lesion is identified 5 cm proximal to the splenic flexure. There are no
contraindications to resection and the decision is made to operate with curative intent. The most appropriate
procedure would be:
a) Right hemicolectomy
b) Total colectomy
c) Sigmoid colectomy
d) Anterior resection
e) Extended right hemicolectomy
The answer is: Extended right hemicolectomy
55) A 32-year-old primigravida at 39 weeks of gestational age has a blood pressure reading of 150/100 mm Hg
obtained during a routine visit. Her baseline blood pressure during the pregnancy was 120/70 mmHg. The patient
denies any headache, visual changes, nausea, vomiting, or abdominal pain. Her repeat BP is 150/90 mmHg, and
urinalysis is negative for protein. Which of the following is the most likely diagnosis?
a) Preeclampsia
b) Chronic hypertension with superimposed preeclampsia
c) Eclampsia
d) Gestational hypertension
e) non of the above
The answer is: Gestational hypertension
56) Microcephaly can be due to:
a) Dandy-walker syndrome
b) Congenital Rubella syndrome
c) Arnold Chiari malformation
d) Hydrocephalus
e) Congenital toxoplasmosis
The answer is: Congenital Rubella syndrome
57) A 70-year-old man under palliative treatment for bronchogenic carcinoma is prescribed a diuretic for ankle
oedema. Over the next 10 days he I becomes more confused, then he develops a tonic—clonic seizure.what is your
diagnosis?
a) Tonic-clonic seizures
b) Hyponatremia
c) Absence seizures
d) Hypomagnesemia
e) Hypocalcemia
The answer is: Hyponatremia

165
EMLE Trial Exams October 2020 (1000 Questions) Version (1.0) 9/12/2020

58) A 14-year-old boy presents with a 6-hour history of a painful limp. He complains of severe pain in his left groin
and has vomited eight times. On examination, he has an absent cremasteric reflex on the left side. What is the
most likely diagnosis?
a) Slipped upper femoral epiphysis
b) Testicular torsion
c) Transient synovitis
d) Septic arthritis
e) Perthes’ disease
The answer is: Testicular torsion
59) A research group is studying sickle cell disease in a geographically isolated community of 6000 people. A genetic
analysis is performed on every community member. At the beginning of the year, it is determined that 10% are
homozygous for hemoglobin S and therefore have sickle cell disease, and 30% of the community is heterozygous for
the mutant allele. Over the course of the year, 100 infants are born, six of whom are diagnosed with sickle cell
disease. Of the 80 people who die during the year, three had sickle cell disease. Which of the following is the current
prevalence of sickle cell disease in this population?
a) 1504861
b) 6/100
c) 1504953
d) 603/6020
e) 606/6100
The answer is: 603/6020
60) What is the best drug to give in patients with hepatic encephalopathy?
a) Frusemide
b) Lactulose
c) Indapamide
d) Midodrine
e) Ciprofloxacin
The answer is: Lactulose
61) An 18-year-old man shot once in the left chest has a blood pressure of 80/50 mm. Hg, a heart rate of 130 beats per
minute, and distended neck veins. Immediate treatment might include:
a) Administration of no solutions.
b) Subxiphoid pericardiotomy.
c) Needle decompression of the left chest in the second intercostal space.
d) Emergency thoracotomy to cross-clamp the aorta.
e) All of the above
The answer is: Needle decompression of the left chest in the second intercostal space.
62) A family came after car accident to the ER the child is 5 years old what is your recommendation for him if he
witnessed loss of conscious for 10 min?
a) Urgent CT
b) US
c) Neurological assessment
d) Reassure
e) Admit and observe
The answer is: Urgent CT
63) A 5-year-old girl complains of perianal pruritus. Results of a clear adhesive tape test are positive. Of the following,
the drug of CHOICE for this infection is:
a) Iodoquinol
b) Ivermectin
c) Mebendazole
166
EMLE Trial Exams October 2020 (1000 Questions) Version (1.0) 9/12/2020

d) Praziquantel
e) Thiabendazole
The answer is: Mebendazole
64) A 42-year-old woman presents with an upper midline mass that has been present for over a year. She has a
history of a partial gastrectomy for a perforated ulcer. On examination, the mass is 5 cm in size, soft, non-tender
and reducible. Which of the following is the most likely diagnosis?
a) Direct inguinal hernia
b) Epigastric hernia
c) Hiatus hernia
d) Incisional hernia
e) Paraumbilical hernia
The answer is: Incisional hernia
65) Most common cause of microcytic hypochromic anemia is:
a) Folic acid deficiency
b) B12 deficiency
c) Iron deficiency
d) Anemia of chronic disease
e) Spherocytosis
The answer is: Iron deficiency
66) The following IS NOT a predisposing factor for premature labor:
a) Oligohydramnios
b) Heavy smoking
c) History of previous preterm birth
d) Twin gestation
e) Pregnancy induced hypertension
The answer is: Oligohydramnios
67) The investigation of choice in case of post-menopausal bleeding after Trans-vaginal Ultrasound is:
a) Pap smear
b) Fractional curettage
c) CA-125
d) Breast Ultrasound
e) MRI
The answer is: Fractional curettage
68) Which of the following describes the patient’s right to participate in decisions about the diagnostic and treatment
recommendation?
a) Autonomy
b) Beneficence
c) Non-maleficence
d) Justice
e) Maleficence
The answer is: Autonomy
69) Which of the following is considered to be an ADVANTAGE of the equipment shown in this photo?
a) pressure release (pop-off ) valve makes over inflation less likely.
b) requires tight face mask seal to inflate the lungs.
c) requires oxygen reservoir to provide high concentration of oxygen.
d) cannot give free- flow of oxygen through the mask.
e) cannot be used for CPAP
The answer is: pressure release (pop-off ) valve makes over inflation less likel
167
EMLE Trial Exams October 2020 (1000 Questions) Version (1.0) 9/12/2020

70) Which of the following is not a cause of acute shortness of breath on the first postoperative day? (Debatable)
a) Atelectasis.
b) Pulmonary embolism.
c) Myocardial infarction.
d) Chest infection.
e) Pneumothorax
The answer is: Pulmonary embolism.
71) Which one of the following is most likely to be found in a patient with Dequervain thyroiditis?
a) Raised ESR
b) Anti-TSH receptor stimulating antibodies
c) Anti-thyroid peroxidase antibodies
d) Decreased TSH
e) Co-existing type 2 DM
The answer is: Raised ESR
72) Malaria causes which type of splenomegaly?
a) Immune hyperplasia
b) RES hyperplasia
c) Congestion
d) Infiltration
e) Neoplastic
The answer is: Immune hyperplasia
73) A 65-year-old woman develops a wrist drop following a spiral distal third fracture of the humeral shaft. What is
the most likely complication that has occurred?
a) Avascular necrosis
b) Compartment syndrome
c) Osteoarthritis
d) Radial nerve palsy
e) Malunion
The answer is: Radial nerve palsy
74) Which of the following statements regarding AIDS are false?
a) After exposure, the virus binds to CD4 receptors.
b) The gut-associated lymphoid tissue (GALT) is not affected.
c) The HIV transmission risk is high during the stage of seroconversion.
d) Most antiviral drugs (HAART) act by inhibiting reverse transcriptase and protease synthesis.
e) Within 2 years, progression of HIV infection to AIDS is seen in 25–35 per cent of patients.
The answer is: The gut-associated lymphoid tissue (GALT) is not affected.
75) A 37-year-old woman, gravida 3, para 3, comes to the physician for very painful menses that have caused her to
miss at least 3 days of work during each menstrual cycle for the past 6 months. Menses occur with heavy bleeding at
regular 28-day intervals. She also has constant dull pain in the pelvic region between cycles. She is otherwise healthy.
She weighs 53 kg (117 lb) and is 160 cm tall; BMI is 20.7 kg/m2. Pelvic examination shows no abnormalities. Pelvic
ultrasonography shows a uniformly enlarged uterus and asymmetric thickening of the myometrial wall with a poorly
defined endomyometrial border. Which of the following is the most likely cause of
these findings?
a) Endometrial tissue within the uterine wall
b) Endometrial tissue in the fallopian tubes
c) Neoplastic change of the endometrium
d) Cystic enlargement of the ovaries

168
EMLE Trial Exams October 2020 (1000 Questions) Version (1.0) 9/12/2020

e) Varicose veins in the lower abdomen


The answer is: Endometrial tissue within the uterine wall
76) A 9-year-old girl presents for evaluation of regular vaginal bleeding. History reveals the at the age of 7 years
and adrenarche at the age of 8 years. The most common cause of this condition in girls is:
a) Idiopathic
b) Gonadal tumors
c) McCune-Albright syndrome
d) Hypothyroidism
e) Non of the above
The answer is: Idiopathic
77) Which one of the following is the main benefit of erythropoietin in patients with end stage renal disease? a)
Reduces proteinuria
b) Corrects anemia and improve exercise intolerance
c) Reduces blood pressure
d) Improves renal functions
e) Reduces long-term all case mortality
The answer is: Corrects anemia and improve exercise intolerance
78) Where should the surgical “time-out” for knee replacement occur?
a) Med/surg unit
b) Preoperative holding area
c) Post anesthesia care unit
d) Operating rooms
e) None of the above
The answer is: Operating rooms
79) Of the following, which is most likely to correct vitamin D deficiency in patients with end stage renal disease?
a) Potassium
b) Ca carbonate
c) Phosphate
d) Thiamine
e) Calcitriol
The answer is: Calcitriol
80) Child 8 years has shocked in a food in front of you cannot cough but he is conscious what will you do?
a) Heimlich maneuver
b) CPR
c) Encourage coughing
d) Send to ER
e) Call ambulance
The answer is: Heimlich maneuver
81) What are the earliest changes observed by ophthalmoscope in background retinopathy of diabetes?
a) Arterio-venous shunts
b) Exudates
c) Increased capillary permeability
d) Microaneurysms
e) Venous dilatation
The answer is: Microaneurysms
82) A 49-year-old man presents with a 4-week history of recurrent headaches occurring at the same time every day,
shortly after going to sleep. It is usually associated with pain around the right eye and lacrimation. There is no aura

169
EMLE Trial Exams October 2020 (1000 Questions) Version (1.0) 9/12/2020

and each episode lasts less than an hour. The headache is relieved by movement .What is the most common cause of
the worst headache in patient’s life?
a) Giant cell arteritis
b) Subarachinoid hemorrhage
c) Tension headache
d) Cluster headache
e) Migraine
The answer is: Subarachinoid hemorrhage
83) The management in rupture of membranes in 32 weeks’ gestation include all of the following EXCEPT:
a) Antibiotics for 10 days
b) Tocolysis until 36 weeks gestation
c) Steroids until 36 weeks gestation
d) Induction of labor at 36 weeks gestation
e) Expectant management
The answer is: Tocolysis until 36 weeks gestation
84) A 36-year-old man presents to the emergency department with a severe left-sided headache with pain around the
left eye. He has had several similar episodes over the last 2 weeks, lasting 40-60 minutes each. The headaches are
associated with a runny nose. On examination, there is redness and tearing of his left eye. What is the first line long-
term preventative management of cluster headaches?
a) Verapamil
b) Carbamazepine
c) Lamotrigine
d) Phenytoin
e) Sumatriptan
The answer is: Verapamil
85) During a repair of a primary inguinal hernia, you are asked to name the nerve that is located within the spermatic
cord. Which of the following is the nerve that is found outside the spermatic cord and within the inguinal canal?
a) Ilioinguinal nerve
b) Genitofemoral nerve
c) Genital branch of the genitofemoral nerve
d) Iliohypogastric nerve
e) Lateral femoral cutaneous nerve
The answer is: Ilioinguinal nerve
86) A 47-year-old alcoholic presents with a 24-hour history of severe epigastric pain radiating to the back, nausea and
vomiting. O/E: epigastric tenderness and rigidity. Investigations show macrocytic anaemia, leukocytosis,
hyperglycaemia and prolonged clotting .what is the cause of his abdominal pain?
a) Acute appendicitis
b) Acute pancreatitis
c) Ascending cholangitis
d) Acute MI
e) Ruptured abdominal aortic aneurysm
The answer is: Acute pancreatitis
87) A 75-year-old man develops bleeding per rectum and presents with blood pressure of 90/60 mm Hg and heart rate
of 120 beats/min. His vital signs improved slightly with crystalloid and packed red cells infusion. Which of the
following is considered the most appropriate next step in management?
a) CT scan with double contrast (IV&oral).
b) Bariumenema.
c) Upper GIT endoscopy and colonoscopy.
170
EMLE Trial Exams October 2020 (1000 Questions) Version (1.0) 9/12/2020

d) Radioactive labeled RBCs study.


e) Conservative medical treatment.
The answer is: Upper GIT endoscopy and colonoscopy.
88) You are reviewing a 65-year-old in the renal clinic. He has been on haemodialysis for chronic kidney disease for
the past 6 years. What is he most likely to die from?
a) Hyperkalemia
b) Malignancy
c) Dilated cardiomyopathy
d) Dialysis related sepsis
e) Ischemic heart disease
The answer is: Ischemic heart disease
89) Which one of the following is the most important cause of jaundice presenting in the first 24 hours of life?
a) Prematurity.
b) Hemolytic disease of the newborn.
c) Breast milk jaundice.
d) Early-onset sepsis.
e) Physiologic jaundice
The answer is: Hemolytic disease of the newborn.
90) A 29-year-old patient with a diagnosis of Crohn’s disease undergoes an emergency laparotomy, adhesiolysis and
multi-level small bowel resection. Prior to surgery, she continued her steroids and immunosuppression. Several days
postoperatively, a high output small bowel fistula erupts through the midline laparotomy wound. A water soluble
contrast study confirms this to be a high jejunal fistula. What is the most appropriate nutritional
regimen?
a) Nil by mouth and IV fluids
b) Fine bore naso-gastric (NG) feeding
c) Percutaneous endoscopic gastrostomy (PEG) feeding
d) Elemental diet
e) Total parenteral nutrition (TPN)
The answer is: Total parenteral nutrition (TPN)
91) In trauma imaging, which of the following statement is false?
a) In a multiply injured patient, CT of head and spine should be the first line of imaging.
b) Focused assessment with sonography for trauma helps in detecting intraperitoneal fluid and cardiac tamponade.
c) CT should not be used when a patient is unstable.
d) U/S is useful for diagnosing occult pneumothorax.
e) CT is the main imaging method for intracranial, intra-abdominal and vertebral injuries.
The answer is: In a multiply injured patient, CT of head and spine should be the first line of imaging.
92) A 25-year-old woman at 33 weeks, gestation is noted to have a placenta previa. Upon cesarean section, bluish
tissue densely adherent between the uterus and maternal bladder is noted. The most likely added placental
abnormality is
a) Placenta accreta
b) Placenta increta
c) Placenta percreta
d) Placental polyp
e) Placental hemangioma
The answer is: Placenta increta
93) Surgery for ileal and ileocaecal tuberculosis may involve?
a) Stricturoplasty.
b) Limited ileocolic resection.
171
EMLE Trial Exams October 2020 (1000 Questions) Version (1.0) 9/12/2020

c) Right hemicolectomy.
d) Ileo-transverse anatomsis.
e) All of the above.
The answer is: All of the above.
94) What type of pancreatic tumor is responsible for Zollinger–Ellison syndrome?
a) Insulinoma
b) Gastrinoma
c) VIPoma
d) Adenocarcinoma
e) Glucagonoma
The answer is: Gastrinoma
95) Treatment of carditis: (Debatable)
a) Steroids
b) IVIG
c) Haloperidol
d) Support
e) Immunomodulators
The answer is: Steroids
Website answer is Steroid
But answer is may be (Support)
96) A 12-year-old boy presents with an acutely tender & swollen scrotum. He is otherwise well. The cremasteric reflex
is absent on examination & transillumination is normal. What is the most likely diagnosis?
a) Testicular torsion
b) Torsion of the hydstid of Morgagni
c) Epididymitis
d) Orchitis
e) Inguinal hernia
The answer is: Testicular torsion
97) High risk factors of cervical intraepithelial neoplasia (CIN), include the followings EXCEPT:
a) HPV types 6 and 11
b) HIV
c) Smoking
d) First sexual intercourse before 18 years age
e) Multiple sexual partners
The answer is: HPV types 6 and 11
98) A 24-year-old sexually active woman is seen by her ObGyn physician for complaints of abdominal pain. She is
evaluated briefly and treated for a UTI without any other tests being performed. The next day, the patient presents to
the emergency department and is diagnosed with a ruptured appendicitis. Which bias caused this adverse event?
a) Violation bias
b) Anchoring bias
c) Confirmation bias
d) Premature closure
e) Availability bias
The answer is: Premature closure
99) As regards Chlamydial infection, the following statements are correct EXCEPT:
a) The organisms are obligatory intracellular organisms
b) It may cause sterile pyuria

172
EMLE Trial Exams October 2020 (1000 Questions) Version (1.0) 9/12/2020

c) Chlamydia and gonorrhea are the main causes of PID


d) The husband should be treated
e) Azithromycin 2 g in single dose is an effective treatment
The answer is: Azithromycin 2 g in single dose is an effective treatment
100) Prioritizing young healthy patients over old frail patients in COVID19 epidemics regarding mechanical
ventilation spaces is a demonstration of which of the following?
a) Autonomy
b) Confidentiality
c) Justice
d) Equity
e) Beneficence
The answer is: Equity

173
EMLE Trial Exams October 2020 (1000 Questions) Version (1.0) 9/12/2020

Exam 10
1) A 65-year-old man collapses in the street. On examination, he has an umbilical mass that is expansible and
pulsatile. What is the most likely diagnosis?
a) Hepatocellular carcinoma
b) Pseudo pancreatic cyst
c) Psoas abscess
d) Abdominal aortic aneurysm
e) Diverticulosis
The answer is: Abdominal aortic aneurysm
2) During a regular check-up of an 8-yr-old child, you note a loud first heart sound with a fixed and widely split
second heart sound at the left upper left sternal border that does not change with respirations. The most likely
diagnosis is:
a) Atrial septal defect (ASD)
b) Ventricular septal defect
c) Isolated tricuspid regurgitation
d) Tetralogy of Fallot
e) Transposition of great arteries
The answer is: ASD
3) A 61-year-old nulliparous woman comes to the physician for a follow-up examination. Her last Pap smear 3 years
ago showed atypical squamous cells of undetermined significance. HPV testing was negative at that time. On
questioning, she has had fatigue and an increase in abdominal girth despite a 5-kg (11.0-lb) weight loss over the
past 6 months. She has gastroesophageal reflux disease and Hashimoto's thyroiditis. Menarche was at the age of
10 years and her last menstrual period was 2 years ago. Current medications include omeprazole and
levothyroxine. Abdominal examination shows shifting dullness. There is tenderness to palpation of the right lower
quadrant but no guarding or rebound. Bimanual palpation shows a small uterus and a right adnexal mass. Further
evaluation of this patient is most likely to show which of the following findings?
a) Proliferation of endometrial glands
b) Elevated serum CA-125 level
c) Elevated serum beta-hCG level
d) Prolonged prothrombin time
e) Chocolate cyst of the right ovary
The answer is: Elevated serum CA-125 level
4) A primigravida at 26 weeks gestation and her CBC showing HB of 9.4 g/dL; MCV of 77fL and TIBC saturation < 15%.
What is the INCORRECT statement regarding her management?
a) The recommended oral dose of elemental iron is 150 – 300 mg/day.
b) Oral iron therapy may cause constipation
c) Parenteral iron therapy might induce teratogenic effects
d) Adding folic acid to the iron formula is better than iron formula alone.
e) Cyanocobalamin is essential for normal erythropoiesis
The answer is: Parenteral iron therapy might induce teratogenic effects

174
EMLE Trial Exams October 2020 (1000 Questions) Version (1.0) 9/12/2020

5) A 12-year-old girl is brought to the GP by her mother having noticed a painless swelling on the left side of her
neck following a recent cold. On examination, there is a smooth, fluctuant, non-tender swelling anterior to the
sternocleidomastoid muscle. What is the most likely diagnosis?
a) Branchial cyst
b) Cervical rib
c) Cystic hygroma
d) Sternocleidomastoid tumour
e) Thyroglossal cyst
The answer is: Branchial cyst
6) A 27-year-old woman who suffers from rheumatic mitral stenosis develops atrial fibrillation. She is placed on
warfarin therapy. What is the most appropriate target international normalized ratio (INR) range?
a) <1.0
b) 1.0–2.0
c) 2.0–3.0
d) 3.0–4.0
e) >5
The answer is: 2.0–3.0
7) Primary amenorrhea with serum levels of FSH is 57 miu/ml can be
a) Polycystic ovarian disease
b) Ovarian endometrioma
c) Ovarian dysgenesis
d) Sheehan, s syndrome
e) Ovarian cyst
The answer is: Ovarian dysgenesis
8) Child 2.5 years old responds to his mum questions just with some vocal sounds what score will you give for that
according to Glasgow coma scale?
a) 2
b) 1
c) 3
d) 4
e) 5
The answer is: 3
9) Which of the following can be worn on hands during patient care? (Debatable)
a) A ring with a plain band
b) False nails
c) Discipline: infection control
d) Subject: hand washing
e) Difficulty level: easy
The answer is: A ring with a plain band
10) A 42-year-old married woman comes to the physician because of vaginal discharge for 3 days. She has no dysuria,
dyspareunia, pruritus, or burning. The patient is sexually active. She often douches between sexual intercourse.
Pelvic examination shows thin and off-white vaginal discharge. The pH of the discharge is 5.1. Wet mount exam
shows a quarter of her vaginal epithelial cells are covered with small coccobacilli. Which of the following is the
most appropriate next step in management?
175
EMLE Trial Exams October 2020 (1000 Questions) Version (1.0) 9/12/2020

a) Treat the patient with ceftriaxone and azithromycin


b) Treat the patient and partners with metronidazole
c) Treat patient with topical ketoconazole
d) Reassurance and follow-up in one week
e) Treat the patient with metronidazole
The answer is: Treat the patient with metronidazole
11) Bartholin abscess present with variety of symptoms that DO NOT include:
a) Impairment of ability to walk
b) Dyspareunia.
c) Local throbbing pain
d) Purulent discharge
e) Pruritus vulva
The answer is: Pruritus vulva
12) A 27-year-old man presents to the emergency department with a 6-hour history of upper abdominal pain
radiating to the back and associated vomiting. On examination, he has marked epigastric tenderness and you
notice a bluish discolouration around his umbilicus. His heart rate is 118/ min and blood pressure is 108/76
mmHg. Which of the following blood analytes would be most useful in identifying a diagnosis?
a) Amylase
b) C-reactive protein
c) Haemoglobin
d) Sodium
e) Urea
The answer is: Amylase
13) A town with 1000 citizens has a 10% prevalence of disease X. A screening test for disease X was just developed,
with a sensitivity of 80% and a specificity of 70%. How many people without disease X will be falsely diagnosed
positive by this screening test?
a) 20
b) 80
c) 100
d) 270
e) 630
The answer is: 270
14) A 75-year-old man develops bleeding per rectum and presents with blood pressure of 90/60 mm Hg and heart
rate of 120 beats/min. His vital signs improved slightly with crystalloid and packed red cells infusion. Which of the
following is considered the most appropriate next step in management?
a) CT scan with double contrast (IV&oral).
b) Bariumenema.
c) Upper GIT endoscopy and colonoscopy.
d) Radioactive labeled RBCs study.
e) Conservative medical treatment.
The answer is: Upper GIT endoscopy and colonoscopy.
15) Most common cause of Megaloblastic anemia is:
a) Folic acid deficiency
b) Zinc deficiency
176
EMLE Trial Exams October 2020 (1000 Questions) Version (1.0) 9/12/2020

c) Iron deficiency
d) Anemia of chronic disease
e) Spherocytosis
The answer is: Folic acid deficiency
16) Which of the following is considered a dangerous sign of headache?
a) Morning headache and vomiting
b) Being unilateral
c) Being bilateral
d) Being in the morning
e) Associated with aura
The answer is: Morning headache and vomiting
17) A 10-year-old girl presents with symptoms of an upper respiratory tract infection. She is incidentally noted to be
hypertensive on examination. Her systolic blood pressure (BP) is 116\75 mmHg, which is at the 95th centile for
her weight & height. What is the next best step in management?
a) Admission to hospital for anti-hypertensive treatment
b) None, as hypertension in childhood is defined as a BP>120 mmHg
c) BP measurement using the smallest cuff available
d) Renal biopsy
e) Ambulatory BP measurement
The answer is: Ambulatory BP measurement
18) Which one of the following is most likely to be found in a patient with thyrotoxicosis?
a) Raised ESR
b) Anti-TSH receptor stimulating antibodies
c) Anti-thyroid peroxidase antibodies
d) Decreased TSH
e) Co-existing type 2 DM
The answer is: Decreased TSH
19) A 41-year-old lawyer presents with a 3-month history of recurrent headaches which feel like a tight band around
the head. Which of the following is considered a dangerous sign of headache?
a) Morning headache and vomiting
b) Being unilateral
c) Being bilateral
d) Being in the morning
e) Associated with aura
The answer is: Morning headache and vomiting
20) A 10-yr-old child has intermittent symptoms of mild asthma. The most appropriate treatment option is:
a) Environmental control and patient education only when no medication is indicated
b) Oral theophylline
c) Cromolyn
d) Inhaled B2 agonist as needed for symptoms
e) Daily inhaled corticosteroid
The answer is: Inhaled B2 agonist as needed for symptoms

177
EMLE Trial Exams October 2020 (1000 Questions) Version (1.0) 9/12/2020

21) A 35–year-old banker presents with intermittent lower abdominal pain, gaseous distension with a tendency
toward diarrhea whenever he is “stressed out”. He is a heavy cigarette smoker. Physical examination shows only
diffuse abdominal distension. The most important criteria of abdominal pain of this disease is:
a) Associated with a diarrhea
b) Association with rectal bleeding
c) Relieved with defecation
d) Relieved with urination
e) Relieved with vomiting
The answer is: Relieved with defecation
22) Child 8 years old witnessed to have loss of conscious for 6 min after head trauma now he is good except of 4
episodes of vomiting what is you recommendation?
a) MRI
b) X ray
c) Urgent CT
d) Observe
e) Reassure
The answer is: Urgent CT
23) A 36-year-old woman comes to the physician to discuss contraceptive options. She is currently sexually active
with her husband, and they have not been using any contraception. She has no significant past medical history
and takes no medications. She has smoked one pack of cigarettes daily for 15 years. She is allergic to latex and
copper. A urine pregnancy test is negative. Which of the following contraceptive methods is absolutely
contraindicated in this patient?
a) Diaphragm with spermicide
b) Progestin-only pill
c) Intrauterine device
d) Condoms
e) Combined oral contraceptive pill
The answer is: Combined oral contraceptive pills
24) A 27-year-old G 1 PO at 1 5 week's gestation has a history of chronic hypertension. She presents for her first
prenatal visit. Vital signs: T 98°F, P 66 beats/min, R 1 6 breaths/min, BP 1 46/98 mmHg. She stopped taking her BP
medication 6 weeks ago when she found out that she was pregnant. Of the following medications, which is the
best choice for treating chronic hypertension in pregnancy?
a) Hydralazine
b) Hydrochlorothiazide
c) Labetalol
d) Lisinopril
e) Atenolol
The answer is: Labetalol
25) A 27-year-old man presents to the emergency department complaining of severe nasal pain and a blocked nose.
On examination, he has a bluish discoloured swelling over the bridge of the nose. His friend tells you he was
assaulted four days ago. What is the most likely cause of his symptoms?
a) Fracture of the orbit
b) Fracture of the cribriform plate
c) Intranasal foreign body

178
EMLE Trial Exams October 2020 (1000 Questions) Version (1.0) 9/12/2020

d) Maxillary sinusitis
e) Septal haematoma
The answer is: Septal haematoma
26) A 5-year-old boy presents to the Emergency Department with a fever of 39⁰ C, red eyes, strawberry tongue &
cracked lips, periungual desquamation of his fingers & toes, polymorphous rash & cervical lymphadenopathy. He
has been too unwell to attend school for a week due to these symptoms. Which of the following treatments will
afford him the best long-term prognosis?
a) A course of penicillin
b) Cyclophosphamide
c) IV immunoglobulin with aspirin
d) IV corticosteroids
e) Anti-INF-a immunotherapy
The answer is: IV immunoglobulins with aspirin
27) An otherwise well 2-year-old presents with extreme pallor. Growth & development are normal. The child is pale
but there is no lymphadenopathy or splenomegaly. Full blood count shows haemoglobin 4g\dl (40 g\l). MCV 59.
Other indicies are normal. What is the most appropiate management option?
a) Bone marrow aspirate
b) Chelation therapy for lead
c) Dietary review
d) Small bowel technetium scan
e) Start gluten-free diet
The answer is: Dietary review
28) A 50‐year old cachectic Korean woman with known stage 4 gastric cancer presents with progressive oral
intolerance and weight loss. An upper gastrointestinal series demonstrates high‐grade gastric outlet obstruction.
Which of the following statements about management of malignant gastric outlet obstruction is true?
a) Gastrojejunostomy is the current standard of care for patients with malignant gastric outlet obstruction
b) Endoscopic stenting is contraindicated in malignant gastric outlet obstruction due to the risk of biliary sepsis
from ampullary obstruction
c) Compared to endoscopic stenting, gastrojejunostomy is associated with a higher proportion of late
complications including recurrent obstruction and cholangitis
d) Compared to gastrojejunostomy, endoscopic stenting is associated with better short‐term outcomes as well as
shorter hospital length of stay
e) Nasogastric decompression and nil per os will resolve malignant gastric outlet obstruction in 60% of patient
The answer is: Compared to gastrojejunostomy, endoscopic stenting is associated with better short‐term outcomes as
well as shorter hospital length of stay
29) A 33-year-old female is diagnosed with a personality disorder by her community psychiatrist. She has struggled to
hold down a job as an assistant store manager as she often finds her colleagues to be lacking in morals or values
and is reluctant to delegate work to them. She feels that her colleagues are lazy and do not perform their duties
to a sufficiently high standard, as a result, she is often overwhelmed with outstanding tasks that she cannot
complete and ends up staying late to get things right. What personality disorder is she most likely to have been
diagnosed with? (Debatable)
a) Schizoid
b) Narcissistic
c) Borderline

179
EMLE Trial Exams October 2020 (1000 Questions) Version (1.0) 9/12/2020

d) Obsessive – compulsive
e) Dependant
The answer is: Obsessive – compulsive
30) An 80-year-old male presents to the Emergency Department experiencing numbness in his right arm which
resolves after 6 hours upon being admitted to the general medical ward. He has had three episodes of similar
symptoms within the last week. His past medical history includes hypertension, type 2 diabetes mellitus, stage 2
chronic kidney disease, and has suffered a myocardial infarction 10 years ago. On clinical examination you detect
a left carotid bruit and suspect he is embolizing from a probable atheromatous plaque. From the following list,
what is the most appropriate first line investigation to exclude carotid artery stenosis?
a) Magnetic resonance angiography
b) Plain X-ray neck
c) Carotid angiography
d) Ultrasound scan of neck
e) Carotid duplex scan
The answer is: Carotid duplex scan
31) An 8-year-old girl presents with fever, neck stiffness and photophobia. She is confused, with spontaneous eye-
opening and can obey motor commands. Calculate the Glasgow Coma Score (GCS) of this patient
a) 10
b) 11
c) 12
d) 13
e) 1
The answer is: 14
32) A 23-year-old man with very severe Crohn's disease has been put on complete bowel rest for an indefinite period
of time whilst he continues his in-patient recovery. He has been receiving total parenteral nutrition for the past
few weeks. Recently he has noticed his skin becoming dry and scaly with mouth ulcers and lip swelling.Which of
the following cause both anemia and mixed central and peripheral neurological signs?
a) Iron deficiency
b) Zinc deficiency
c) Vitamin B12 deficiency
d) Folate deficiency
e) Selenium deficiency
The answer is: Vitamin B12 deficiency
33) Which of the following statements about homeostasis are false?
a) It is defined as a stable state of the normal body.
b) The central nervous system, heart, lungs, kidneys and spleen are the essential organs that maintain homeostasis
at a normal level.
c) Elective surgery should cause little disturbance to homeostasis.
d) Emergency surgery should cause little disturbance to homeostasis.
e) Return to normal homeostasis after an operation would depend upon the presence of co-morbid conditions.
The answer is: Emergency surgery should cause a little disturbance to homeostasis
34) A 2 3-year-old.woman who is 10 weeks pregnant with a 3-week history of hyperemesis gravidarum is brought to
the emergency room after she collapsed and started fiting. what is your diagnosis?
a) Tonic-clonic seizures
180
EMLE Trial Exams October 2020 (1000 Questions) Version (1.0) 9/12/2020

b) Hyponatremia
c) Absence seizures
d) Hypomagnesemia
e) Hypocalcemia
The answer is: Hypomagnesemia
35) 56 years old female patient underwent mastectomy for invasive duct carcinoma grade 3, the following structure
can be injured during this procedure:
a) Long thoracic nerve
b) Intercostobrachial nerve
c) Intercostal nerves
d) A & B
e) A, B, & C
The answer is: A & B
36) Which of the following would be appropriate immediate step for a patient admitted to hospital with sepsis?
a) Blood cultures
b) CT brain
c) Immediate transfer to critical care unit
d) Maintenance of blood sugar less than 110 mg/dl
e) X-ray chest
The answer is: Blood culture
37) Successful management of allergic disorder can be started by:
a) reassurance
b) Avoidance of allergens or irritants which can be identified
c) Pharmacological therapy alone
d) Immunotherapy
e) Gene therapy
The answer is: Avoidance of allergens or irritants which can be identified
38) A Case of F4 you meet in the hospital and fall down in front of you seems to be a cyanotic spell what is the first
thing you will do ?
a) Morphine
b) Beta blocker
c) O2
d) Squatting position for the patient
e) Abs
The answer is: Squatting position for the patient
39) A 70-year-old man under palliative treatment for bronchogenic carcinoma is prescribed a diuretic for ankle
oedema. Over the next 10 days he I becomes more confused, then he develops a tonic—clonic seizure.what is
your diagnosis?
a) Tonic-clonic seizures
b) Hyponatremia
c) Absence seizures
d) Hypomagnesemia
e) Hypocalcemia
The answer is: Hyponatremia
181
EMLE Trial Exams October 2020 (1000 Questions) Version (1.0) 9/12/2020

40) A 3-day-old term infant who has not passed meconium since birth. On examination his abdomen is distended but
the remainder of the examination is normal. An X-ray of the abdomen shows distended loops throughout the
bowel, including the rectum. What is the most likely diagnosis?
a) Congenital hypothyroidism
b) Cystic fibrosis
c) Duodenal atresia
d) Hirschsprung disease
e) Malrotation
The answer is: Hirschsprung disease
41) Which of the following is not a reason for breaking patient’s confidentiality?
a) Suicide
b) Homicide
c) TB
d) HIV
e) Influenza
The answer is: Influenza
42) What is the preferred treatment for an anaphylactic shock?
a) Epinephrine
b) Placing the patient in sitting position and administer oxygen
c) Preventing the reaction from occurring through patient teaching
d) Placing a bag of ice on the area, administer antihistaminic and corticosteroids
e) Antibiotic therapy
The answer is: Epinephrine
43) A 64-year-old woman presents with a 2-day history of increasing left-sided abdominal pain with fever. On
examination, she has localized peritonism in the left iliac fossa. Her blood tests reveal a raised white cell count
and C-reactive protein. Which of the following is the most likely diagnosis?
a) Constipation
b) Diverticular disease
c) Diverticulitis
d) Diverticulosis
e) Irritable bowel syndrome
The answer is: Diverticulitis
44) Which of the following factors are essential for good wound healing?
a) Steroids
b) Haematoma
c) Infection
d) Tension
e) Blood supply
The answer is: blood supply
45) Folic acid deficiency during pregnancy is associated with the following EXCEPT:
a) Hypersegmented neutrophils
b) Megaloblastic erythropoiesis
c) Increased rate of systemic infections
d) Megaloblastic anemia
182
EMLE Trial Exams October 2020 (1000 Questions) Version (1.0) 9/12/2020

e) Abruptioplacentae
The answer is: Increased rate of systemic infections
46) A 13-year-old boy presents to the Emergency Department at 01:00 in the morning with a 2-hour history of acute
left-sided testicular pain and vomiting. He has had no recent history of a viral illness and has had a past medical
history of delayed testicular descent. On clinical examination, abdominal examination revealed no abnormality;
the left testicle is high riding, and is in a transverse lie. It is exquisitely tender on palpation. A urine dipstick
analysis is performed which shows no abnormality and no evidence of infection. From the following list, select the
next appropriate course of action:
a) Immediate out of hours ultrasound scan of scrotum
b) Wait until 08:00 and proceed to scrotal exploration
c) Immediate scrotal exploration
d) Urgent ultrasound scan of scrotum at 08:00
e) Discharge with outpatient ultrasound scan and urgent outpatient follow-up
The answer is: Immediate surgical exploration
47) What is the best drug to give in patients with hepatic encephalopathy?
a) Frusemide
b) Lactulose
c) Indapamide
d) Midodrine
e) Ciprofloxacin
The answer is: Lactulose
48) The COMMONEST site of occurrence ectopic pregnancy is:
a) Isthmic or interstitial portion of the Fallopian tube
b) Corneal end of the tube
c) Ampullary portion of the fallopian tube
d) Fimbrial end of the Fallopian tube
e) Ovary
The answer is: Ampullary portion of the fallopian tube
49) A primigravida, 36 weeks’ gestation is in labor with 3 cm dilated cervix and minimal uterine contraction. On
rupture of membranes, fresh bleeding is noted with late fetal deceleration reaching 50 beats/min. The patient
was taken for LSCS but fetus could not be saved. No abruptio or placenta previa was seen. The MOST LIKELY
diagnosis is:
f) Placenta previa
g) Revealed abruptio placentae
h) Mixed type of abruptio placentae
i) Circumvallate placenta
j) Vasa previ
The answer is: vasa previa
50) A 33-year-old man is admitted following an assault in a street fight. During the fight, he was hit by a blunt object
across the side of the head. On admission his Glasgow Coma Scale score is initially 13/15 but falls to 7/15 during
his evaluation. The decision is taken to perform a computed tomography head scan, which identifies a lens-
shaped space-occupying lesion within the cranial vault. The diagnosis is
a) Extradural haematoma
b) Subdural haematoma
183
EMLE Trial Exams October 2020 (1000 Questions) Version (1.0) 9/12/2020

c) Subarachnoid haemorrhage
d) Cerebral contusion
e) Intracerebral haemorrhage
The answer is: Extradural haematoma
51) All the following can be used to treat elevated intracranial pressure except:
a) Elevate the head of the bed.
b) Place a ventricular catheter.
c) Mannitol bolus IV
d) Hypertonic saline IV
e) Glutamate antagonists
The answer is: Glutamate antagonists
52) An 8-month-old infant, is brought to the ER by his parents. He is having episodes of abdominal pain and is just
recovering from an upper respiratory tract infection. He seems well in-between, but then suddenly seems to be in
pain and looks pale. He has vomited several times. On questioning he has not passed stools for 24 hours.What is
the most likely cause of his illness?
a) Appendicitis
b) Gastroenteritis
c) Malrotation
d) Pyloric stenosis
e) Intussusception
The answer is: Intussusception
53) A 70-year-old woman presents with a 2-month history of anal pain and itching. More recently she had been
having some fresh bleeding and mucous discharge per rectum. On examination, there is an irregular tender
ulceration at the anal margin which appears to be extending into the anal canal. What is the most likely
diagnosis?
a) Anal carcinoma
b) Anal fissure
c) Anal warts
d) Fistula-in-ano
e) Primary syphilis
The answer is: Anal carcinoma
54) A 25-year old with recently diagnosed ulcerative colitis is started on mesalazine after a recent tapering of high
dose steroids. Two weeks later, he develops severe pain in his epigastrium which radiates through to his back.
What is the most likely diagnosis?
a) Hepatitis
b) Acute pancreatitis
c) Primary sclerosing cholangitis
d) Duodenal ulceration
e) Acute coronary syndrome
The answer is: Acute pancreatitis
55) Which of the following statements about head injury and concomitant hyponatremia are true?
a) There are no primary alterations in cardiovascular signs.
b) Signs of increased intracranial pressure may be masked by the hyponatremia.
c) Oliguric renal failure is an unlikely complication.
184
EMLE Trial Exams October 2020 (1000 Questions) Version (1.0) 9/12/2020

d) Rapid correction of the hyponatremia may prevent central pontine injury.


e) This patient is best treated by restriction of water intake.
The answer is: There are no primary alterations in cardiovascular signs.
56) Clinical evidences of cervical cancer may include all of the following EXCEPT:
a) Perimenopausal bleeding
b) Uremia
c) Postmenopausal bleeding
d) Gastric upset
e) Severe pelvic pain
The answer is: Gastric upset
57) What is the level of HCO3 that diagnoses diabetes mellitus?
a) >20 mmol/L
b) <20 mmol/L
c) >18 mmol/L
d) <18 mmol/L
e) <10 mmol/L
The answer is: <18 mmol/L
58) Child 3 years with wet purpura and platelets 3000 what is the best action?
a) Packed RBC transfusion
b) Platelets transfusion
c) Plasma transfusion
d) Reassure
e) Observe
The answer is: Platelets transfusion
59) Child 5 years old presents with purpuric eruptions of one week duration and nose bleeds he is otherwise good and
active, what is the first investigation you will request?
a) CBC
b) PT
c) PTT
d) INR
e) CT
The answer is: CBC
60) The following statements about Trichomonas vaginalis are correct EXCEPT:
f) Trichomonas vaginalis (TV) may be a sexually transmitted disease
g) Most patients, having TV, have symptoms
h) The disease may cause vulval itching
i) Unlike bacterial vaginosis, metronidazole is ineffective therapy
j) It may cause a strawberry appearance of the cervix
The answer is: Unlike bacterial vaginosis, metronidazole is ineffective therapy
61) Which of the following statements about malignant transformation in cells is false?
a) Cells become immortal.
b) Cells acquire angiogenic competence.
c) Cells increase apoptosis.
d) Cells resist signals that inhibit growth.
185
EMLE Trial Exams October 2020 (1000 Questions) Version (1.0) 9/12/2020

e) Cells evade detection/elimination.


The answer is: cell increase apoptosis
62) Which of the following antecedent pregnancy will give the highest score in the WHO scoring system of gestational
trophoblastic neoplasia?
a) Hydatidiform mole.
b) Fulltermpregancy.
c) Spontaneous abortion.
d) Induced abortion.
e) Ectopic Pregnancy.
The answer is:fullterm pregnancy
63) A Culdocentesis is performed in a 19-year-old G1 P0 woman with lower abdominal pain and vaginal spotting. A
total of 3 cc of clotted blood is aspirated. Which one of the following is the best interpretation?
a) A hemoperitoneum is present.
b) No hemoperitoneum is present.
c) The blood probably came from a blood vessel.
d) The patient probably has an ectopic pregnancy.
e) None og the above
The answer is:the patient probably has an ectopic pregnancy
64) The medial boundary of the Hassell Bach triangle is
a) Lina alba
b) Lina semilunaris
c) Inferior epigastric artery
d) Inguinal ligament
e) Internal ring
The answer is:lina semi lunaris
65) A 24-year-old handball player presents to the fracture clinic with a comminuted, displaced fracture of the
clavicle.What is the most appropriate treatment option?
a) Hanging cast
b) External fixation
c) Manipulation and cast application
d) Plating
e) Dynamic hip screw
The answer is:hanging cast
66) The patient’s son is asking you about his father’s diagnosis. which is the most appropriate response
a) Discuss with the son full details of the diagnosis
b) Tell him that you have to take consent firstly from the patient
c) Tell him about the diagnosis but not the full details
d) Tell him that you are not allowed to tell because he has a bad prognosis.
e) None of the above
The answer is: tell him that you have to take consent firstly from the patient
67) A 55-year-old male with history of Diabetes and hypertension presents to the emergency department with chest
pain that started 1 hour ago and didn’t resolve with nitrates. ECG showed ST depression. Troponin is elevated.
Which is the most likely diagnosis?
a) Stable angina
186
EMLE Trial Exams October 2020 (1000 Questions) Version (1.0) 9/12/2020

b) Unstable angina
c) STEMI
d) NSTEMI
e) Pericarditis
The answer is:NSTEMI
68) Guillain-Barre syndrome (Post infectious polyneuropathy) is manifested by:
a) gradual onset and usually stationary course
b) Flaccid paralysis in an Descending manner
c) Unilaterall symmetrical distribution
d) Hypertonia and hyper reflexia
e) Flaccid paralysis ascending manner
The answer is: flaccid paralysis ascending manner
69) Carcinoid tumours most likely to produce carcinoid before syndrome metastasis originate from: (Debatable)
a) Appendix
b) Stomach
c) Bronchus and terminal ileum
d) Rectum
e) Liver
The answer is: bronchus and terminal ilium
70) The following are risk factors for development of placental abruption EXCEPT
a) Short umbilical cord
b) Folic acid deficiency
c) Pre-eclampsia
d) History of threatened abortion
e) Previous placental abruption
The answer is:history of threatened abortion
71) What is the duration of healthy stages of sleep?
a) 5-20 minutes
b) 20-50 minutes
c) 50-90 minutes
d) 90-110 minutes
e) 110-210 minutes
The answer is: 90-110 minutes
72) 72) Overwhelming infection resulting in vasodilatation can lead to:
a) Hypovolemic shock
b) Anaphylactic shock
c) Septic shock
d) Neurogenic shock
e) Cardiogenic shock
The answer is: Septic shock
73) A 60-year-old man develops shortness of breath and a cough productive of purulent sputum three days after a
right hemicolectomy. An arterial blood gas shows pH 7.42, pO2 54 mmHg, pCO2 40 mmHg, bicarbonate
25mmol/L. What blood gas picture does this represent?
a) Metabolic alkalosis
187
EMLE Trial Exams October 2020 (1000 Questions) Version (1.0) 9/12/2020

b) Respiratory acidosis
c) Respiratory alkalosis
d) Type I respiratory failure
e) Type II respiratory failure
The answer is: Type I respiratory failure
74) Non steroidal anti-inflammatory drugs are not generally used for long term tocolysis because they:
a) Are ineffective
b) Produce marked hypertension
c) Are too expensive
d) May cause premature closure of fetal ductus arteriosus
e) Are associated with lactic acidosis
The answer is: Produce marked hypertension
75) Side effects of progestins in treatment of endometriosis DO NOT include
a) Depression
b) Nausea
c) Weight gain.
d) Secondary amenorrhea
e) Breakthrough bleeding
The answer is: Secondary amenorrhea
76) What of the following combination of diuretics is used in ascites secondary to liver cirrhosis?
a) Chlorthalidone and frusemide
b) Metolazone and frusemide
c) Bumetanide and eplerenone
d) Metolazone and spironolactone
e) Frusemide and metolazone
The answer is: Bumetanide and eplerenone
77) A 2 9 -year-old G 1 P 00 1 0 woman with regular menses ( every 2 8 days lasting 5 days) is currently trying to
conceive. She has not used Chapter 9: Gynecology 1 93 contraception for 6 months . Her ovulation predictor kit
revealed an luteinizing hormone (Debatable)
a) Cystic teratoma
b) Ectopic pregnancy
c) Follicular cyst
d) Hemorrhagic corpus luteum cyst
e) Serous cystadenoma
The answer is: Follicular cyst
78) Which of the following about blood transfusion are false?
f) A hemoglobin level of 6 g/dL or less is now considered a typical indication.
g) Fresh frozen plasma (FFP) is considered as the first-line therapy in coagulopathic hemorrhage.
h) Cryoprecipitate is useful in low fibrinogen states and in factor VIII deficiency.
i) Platelets have a shelf life of 3 weeks.
j) Patients can pre-donate blood up to 3 weeks before surgery for autologous transfusion
The answer is: Platelets have a shelf life of 3 weeks.
79) Schistosomiasis causes which type of splenomegaly?
a) Immune hyperplasia
188
EMLE Trial Exams October 2020 (1000 Questions) Version (1.0) 9/12/2020

b) RES hyperplasia
c) Congestion
d) Infiltration
e) Neoplastic
The answer is: Congestion
80) Which one of the following statements is correct regarding the definition of neonatal deaths?
a) Neonatal deaths occur only up to the first 24 hours of life
b) Neonatal deaths occur only up to the first 72 hours of life
c) Neonatal deaths occur only up to the first 7 days of life
d) Neonatal deaths occur only up to the first 21 days of life
e) Neonatal deaths occur only up to the first 28 days of life
The answer is: Neonatal deaths occur only up to the first 28 days of life
81) A 24-year-old woman underwent a normal vaginal delivery of a term infant female. After the delivery, the
placenta does not deliver even after 30 minutes. Which of the following would be the next step for this patient?
a) Initiate oxytocin.
b) Wait for an additional 30 minutes.
c) Hysterectomy.
d) Attempt a manual extraction of the placenta.
e) Misoprostol intravaginally.
The answer is: Attempt a manual extraction of the placenta.
82) A 52-year-old man has several years’ history of intermittent bright red blood on the toilet paper when wiping
after defecation. There has been no mucus or change in bowel habit. Which is the simplest appropriate first
examination to be done at the outpatient clinic ?
a) Colonoscopy
b) CT colonography
c) Endoanal ultrasound
d) Flexible sigmoidoscopy
e) Proctoscopy.
The answer is: Proctoscopy.
83) A 79-year-old patient is admitted to the cardiology service and treated for acute CHF. He is started on a new
medication regimen including a diuretic which relieves his symptoms and improves his cardiac function. He is
discharged home, though he returns to the hospital 10 days later with another episode of CHF. During the
readmission, the team notices that the patient never filled his new prescriptions and was not taking the
prescribed diuretic while at home. What actions can be taken to prevent this from happening again?
a) Provide timely access to care following a hospitalization
b) Communication and coordination of care plan with patients is not a solution
c) Discharge planning and transition pr.ocesses cannot help
d) patient education cannot help
e) support to optimize home care is not the proper action
The answer is: Provide timely access to care following a hospitalization

189
EMLE Trial Exams October 2020 (1000 Questions) Version (1.0) 9/12/2020

84) A 34‐year‐old man was involved in a motorcycle accident. On arrival, his initial systolic blood pressure (SBP) was
80/50 mm Hg but subsequently improved to 120/65 mm Hg after 2 L of normal saline bolus. His Glasgow coma
scale (GCS) was 13. He was taken to the computerized scan (CT). His head CT showed depressed skull fracture with
a subdural hematoma. While being transported back to the trauma bay, he again became hypotensive with SBP
85/50 mm Hg. What would be the most appropriate treatment for this patient?
a) Administer 2 L of normal saline and continue to observe
b) Administer 2 units of O‐ and continue to observe
c) Arterial embolization by interventional radiologist
d) Administer 2 units of O‐ and take the patient directly to surgery
e) Type and cross and wait for radiologist report
The answer is: Administer 2 units of O‐ and take the patient directly to surgery
85) When is the age of weaning?
a) 3 months
b) 6 months
c) 9 months
d) 12 months
e) 15 months
The answer is: 6 months.
86) In post-streptococcal glomerulonephritis there is:
a) History of pancreatitis
b) Low ASO titer
c) High C3 level
d) HTN
e) Hypotension
The answer is: HTN
87) An 80-year-old man has been attending his GP for 3 years with an enlarging right-sided scrotal mass which
disappears on lying flat but worsens during the day. The GP cannot get above the mass, nor does it
transilluminate.
a) maldescended testis
b) femoral aneurysm
c) hydrocoele
d) lipoma of the cord
e) inguinal hernia
The answer is: inguinal hernia
88) Commonest Cyanotic heart in neonates is:
a) VSD
b) ASD
c) PDA
d) F4
e) TGA
The answer is: TGA

190
EMLE Trial Exams October 2020 (1000 Questions) Version (1.0) 9/12/2020

89) Child presented to you with severe abdominal pain and red current jelly stool and you diagnosed intussuseption
what is your recommendation
a) Go to ER
b) Reassure
c) Anti emetic
d) Analgesic
e) Antibiotic
The answer is: Go to ER
90) A 55-year-old male with history of Diabetes and hypertension presents to the emergency department with chest
pain that started 3 days ago after he had runny nose and sneezing for five days. ECG showed ST elevation in all
leads. Which is the most likely diagnosis?
a) Stable angina
b) Unstable angina
c) STEMI
d) NSTEMI
e) Pericarditis
The answer is: Pericarditis
91) All are true about sickle cell anemia except:
a) Valine substitute amino acid number six
b) Painful crisis
c) Absence of α chain
d) Hyperhemolytic crises can occur
e) None of the above
The answer is: Absence of α chain
92) Child one year old suddenly start to cough with stridor, his mum was watching TV while that happened where he
was at his room with his older 4 years old brother playing with small toys What is the best treatment?
a) Antibiotics
b) Steroids
c) Bronchodilator
d) Laryngoscope
e) Anti-viral
The answer is: laryngoscope
93) You are consulted by the emergency room to see a 40‐year‐old woman with peritonitis. She had a Roux‐en‐Y
gastric bypass 6 months ago. Her preoperative BMI was 45 (kg/m2), and her current BMI is 39. She has been
taking multivitamins, metformin, and takes ibuprofen regularly for joint pain. She has stopped drinking alcohol,
but continues to smoke. What is the most likely cause of her abdominal pain?
a) Internal hernia
b) Perforated ulcer
c) Perforation of a gastro‐gastric fistula
d) Obstruction of her jejuno‐jenunal anastomosis resulting in proximal perforation
e) Acute cholecystitis
The answer is: Perforated ulcer

191
EMLE Trial Exams October 2020 (1000 Questions) Version (1.0) 9/12/2020

94) The INCORRECT statement regarding pruritus vulva


a) Rarely to be idiopathic
b) Mightbeiatrogenic.
c) Might be associated with malignant disease
d) Might be associated with sexual frustration
e) Might be associated with pediculosis
The answer is: Rarely to be idiopathic
95) The most conclusive choice in a 55-year-old woman who has presented with postmenopausal bleeding is
a) Transvaginal ultrasound
b) Pap smear
c) CA-125
d) Fractional curettage
e) None of the above
The answer is: Fractional curettage
96) In cases of pruritus vulva
a) Recurrent symptoms are uncommon.
b) Hydrocortisone is contraindicated.
c) Common after menopause.
d) Radical vulvectomy is indicated.
e) Antihistaminics are ineffective for treatment.
The answer is: Common after menopause.
97) A primigravida at 26 weeks gestation and her CBC showing HB of 9.4 g/dL; MCV of 77fL and TIBC saturation < 15%.
What is the INCORRECT statement regarding her management?
a) The recommended oral dose of elemental iron is 150 – 300 mg/day.
b) Oral iron therapy may cause constipation.
c) Parentral iron therapy might induce teratogenic effects.
d) Adding folic acid to the iron formula is better than iron formula alone.
e) Cyanocobalamin is essential for normal erythropoiesis.
The answer is: Parentral iron therapy might induce teratogenic effects.
98) A 24-year-old college student recovers from a bout of severe pancreatitis. He has mild epigastric discomfort,
sensation of bloating, and loss of appetite. Examination reveals an epigastric fullness that on ultrasound is
confirmed to be a pseudocyst. The swelling increases in size over a 3-week period of observation. What should be
the next step in management?
a) Percutaneous drainage of the cyst
b) Laparotomy and internal drainage of the cyst
c) Excision of pseudocyst
d) Total pancreatectomy
e) Administration of pancreatic enzymes
The answer is: Percutaneous drainage of the cyst
99) A 27-year-old man who was the driver of a car involved in a high speed collision with a truck is brought into the
resuscitation room. On arrival, he is complaining of severe left-sided chest pain. You note that he is breathless,
tachycardic and hypotensive. He has reduced air entry on the left side of the chest and the trachea is deviated to
the right. What would you do next?
a) Insert a chest drain into the fifth intercostal space

192
EMLE Trial Exams October 2020 (1000 Questions) Version (1.0) 9/12/2020

b) Insert a wide bore cannula into the second intercostal space


c) Request an urgent chest X-ray
d) Request an urgent ECG
e) Perform a pericardiocentesis
The answer is: Insert a wide bore cannula into the second intercostal space
100) A 24-year-old woman complains of bothersome hirsutism and skipping periods, she does not have evidence of
voice changes, hair loss, or clitoromegaly. The pelvic examination does not reveal adnexal masses. The serum
DHEA-S, testosterone, and 17-hydroxyprogesterone levels are normal. The LH to FSH ratio is 2:1. Which of the
following is the MOST LIKELY diagnosis?
a) Polycystic ovarian syndrome
b) Familialhirsutism
c) Ovarian tumor
d) Adrenaltumor
e) Cushing’s syndrome
The answer is: Polycystic ovarian syndrome

193

You might also like